Skip to main content

Full text of "The Critical Reasoning Guide"

See other formats


MANHATTAN 



Guide 6 



Critical Reasoning 



Includes 
Free Online ^ 
Exams ^ 



& More! 



Demystifies Logical Analysis of Complex Arguments 
Teaches Effective GMAT Problem Solving Strategies 
Includes Practice Problems with Detailed Explanations 
Updated for The Official Guide for GMAT® Review, 13th Ed. 





Reg Fernandez. Manhattan GMAT Instructor 



99th Percentile Instructors • Content-Based Curriculum 

GMAT and GMAC are registered trademarks of the Graduate Management Admission Council which neither sponsors nor endorses this product. 



MANHATTAN GMAT 

Critical Reasoning 



GMAT Strategy Guide 



This unique guide illustrates how to deconstruct arguments using a four-step process 
designed to build speed and improve accuracy. Understanding the underlying structure 
of arguments and answer choices is the key to quick reading and accurate analysis. 




Critical Reasoning GMAT Strategy Guide, Fifth Edition 



10-digit International Standard Book Number: 1-935707-61-2 
13-digit International Standard Book Number: 978-1-935707-61-5 
elSBN: 978-1-937707-02-4 

Copyright © 201 2 MG Prep, Inc. 

ALL RIGHTS RESERVED. No part of this work may be reproduced or used in any form or 
by any means— graphic, electronic, or mechanical, including photocopying, recording, 
taping, web distribution — without the prior written permission of the publisher, 
MG Prep Inc. 

Note: GMAT, Graduate Management Admission Test, Graduate Management Admission 
Council, and GMAC are all registered trademarks of the Graduate Management Admission 
Council, which neither sponsors nor is affiliated in any way with this product. 

Layout Design: Dan McNaney and Cathy Huang 
Cover Design: Evyn Williams and Dan McNaney 
Cover Photography: Alii Ugosoli 




INSTRUCTIONAL GUIDE SERIES 



GMAT Roadmap 

(ISBN: 978-1 -935707-69-1) 

Fractions, Decimals, & Percents 

(ISBN: 978-1 -935707-63-9) 

Algebra 

(ISBN: 978-1-935707-62-2) 

Word Problems 

(ISBN: 978-1 -935707-68-4) 

Geometry 

(ISBN: 978-1-935707-64-6) 



0 



Number Properties 

(ISBN: 978-1-935707-65-3) 

Critical Reasoning 

(ISBN: 978-1-935707-61-5) 

Reading Comprehension 

(ISBN: 978-1 -935707-66-0) 

Sentence Correction 

(ISBN: 978-1 -935707-67-7) 

«^ Integrated Reasoning & Essay 

(ISBN: 978-1 -935707-83-7) 



Q 

0 
0 



SUPPLEMENTAL GUIDE SERIES 

Math GMAT Supplement Guides Verbal GMAT Supplement Guides 

Foundations of GMAT Math Foundations of GMAT Verbal 

(ISBN: 978-1 -935707-59-2) (ISBN: 978-1-935707-01-9) 

Advanced GMAT Quant 

(ISBN: 978-1 -935707-1 5-8) 

Official Guide Companion 

(ISBN: 978-0-9841 78-01 -8) 



MANHATTAN 

GMAT 

April 24th, 2012 
Dear Student, 

Thank you for picking up a copy of Critical Reasoning. I hope this book provides just the guidance you need to get the 
most out of your GMAT studies. 

As with most accomplishments, there were many people involved in the creation of the book you are holding. First 
and foremost is Zeke Vanderhoek, the founder of Manhattan GMAT. Zeke was a lone tutor in New York when he 
started the company in 2000. Now, 12 years later, the company has instructors and offices nationwide and contributes 
to the studies and successes of thousands of students each year. 

Our Manhattan GMAT Strategy Guides are based on the continuing experiences of our instructors and students. For 
this volume, we are particularly indebted to Dave Mahler, Ian Jorgeson, and Stacey Koprince. Dave deserves special 
recognition for his contributions over the past number of years. Dan McNaney and Cathy Huang provided their 
design expertise to make the books as user-friendly as possible, and Noah Teitelbaum and Liz Krisher made sure all 
the moving pieces came together at just the right time. And there's Chris Ryan. Beyond providing additions and edits 
for this book, Chris continues to be the driving force behind all of our curriculum efforts. His leadership is invaluable. 
Finally, thank you to all of the Manhattan GMAT students who have provided input and feedback over the years. This 
book wouldn't be half of what it is without your voice. 

At Manhattan GMLAT, we continually aspire to provide the best instructors and resources possible. We hope that 
you will find our commitment manifest in this book. If you have any questions or comments, please email me at 
dgonzalez@manhattanprep.com. I'll look forward to reading your comments, and I'll be sure to pass them along to 
our curriculum team. 

Thanks again, and best of luck preparing for the GMAT! 



Sincerely, 




Dan Gonzalez 
President 

Manhattan GMAT 



www.manhattangmat.com 138 West 25th St., 7th Floor NY, NY 10001 Tel: 212-721-7400 Fax: 646-514-7425 



HOW TO ACCESS YOUR ONLINE RESOURCES 

If you... 

® are a registered Manhattan GMAT student 

and have received this book as part of your course materials, you have AUTOMATIC 
access to ALL of our online resources. This includes all practice exams, question banks, 
and online updates to this book. To access these resources, follow the instructions in 
the Welcome Guide provided to you at the start of your program. Do NOT follow the 
instructions below. 

® purchased this book from the Manhattan GMAT online store 
or at one of our centers 

1. Go to: http://www.manhattangmat.com/practicecenter.cfm. 

2. Log in using the username and password used when your account was set up. 

® purchased this book at a retail location 

1. Create an account with Manhattan GMAT at the website: https://www.manhattangmat.com/createaccount.cfm. 

2. Go to: http://www.manhattangmat.com/access.cfm. 

3. Follow the instructions on the screen. 

Your one year of online access begins on the day that you register your book at the above URL. 

You only need to register your product ONCE at the above URL. To use your online resources any 
time AFTER you have completed the registration process, log in to the following URL: 
http://www.manhattangmat.com/practicecenter.cfm. 

Please note that online access is nontransferable. This means that only NEW and UNREGISTERED copies of the book 
will grant you online access. Previously used books will NOT provide any online resources. 

® purchased an eBook version of this book 

1. Create an account with Manhattan GMAT at the website: 
https://www.manhattangmat.com/createaccount.cfm. 

2. Email a copy of your purchase receipt to books@manhattangmat.com to activate 
your resources. Please be sure to use the same email address to create an account 
that you used to purchase the eBook. 







For any technical issues, email books@manhattangmat.com or call 800-576-4628. 
Please refer to the following page for a description of the online resources that come with this book. 



YOUR ONLINE RESOURCES 

Your purchase includes ONLINE ACCESS to the following: 

® 6 Computer-Adaptive Online Practice Exams 

The 6 full-length computer-adaptive practice exams included with the 
purchase of this book are delivered online using Manhattan GMAT's propri- 
etary computer-adaptive test engine. The exams adapt to your ability level by 
drawing from a bank of more than 1,200 unique questions of varying 
difficulty levels written by Manhattan GMAT's expert instructors, all of whom 
have scored in the 99th percentile on the Official GMAT. At the end of each 
exam you will receive a score, an analysis of your results, and the opportunity 
to review detailed explanations for each question. You may choose to take 
the exams timed or untimed. 




L 



oonaoocmnoa 
□□□□□□□□□□□a 
aoaanDonnoa 



The content presented in this book is updated periodically to ensure that 
it reflects the GMAT's most current trends and is as accurate as possible. 
You may view any known errors or minor changes upon registering for 
online access. 



Important Note: The 6 computer adaptive online exams included with the purchase of 
this book are the SAME exams that you receive upon purchasing ANY book in the 
Manhattan GMAT Complete Strategy Guide Set. 



® Critical Reasoning Online Question Bank 

The Bonus Online Question Bank for Critical Reasoning consists of 25 extra practice questions (with 
detailed explanations) that test the variety of concepts and skills covered in this book. These questions 
provide you with extra practice beyond the problem sets contained in this book. You may use our online 
timer to practice your pacing by setting time limits for each question in the bank. 




® Online Updates to the Contents in this Book 



The content presented in this book is updated periodically to ensure that it reflects the GMAT's most 
current trends. You may view all updates, including any known errors or changes, upon registering for 
online access. 



TABLE Of CONTENTS 



1 . Argument Structure 1 1 

Problem Set 23 

2. Methodology 29 

Problem Set 45 

3. Structure-Based Family 51 

Problem Set 67 

4. Assumptions 91 

Problem Set 117 

5. Strengthen and Weaken 1 41 

Problem Set 161 

6. Evidence Family 1 89 

Problem Set 207 

7. Complete the Argument 225 

Problem Set 233 

8. Wrong Answer Analysis 247 

Problem Set 257 

Appendix A: Official Guide Problem Sets 269 



6 



Chapter/1 




Critical Reasoning 



Argument Structure 



is 



The Core 

Building Blocks of an Argument 
Argument Structure 
Intermediate Conclusions and the Therefore Test 



Argument Structure 



Here is an example of a typical GMAT argument. 

SI: Background 




The expansion of the runways at the Bay City Airport will allow 
S2: 1 for larger planes to travel to and from Bay City. These new 
Counterpoint |\p|anes will create a large amount of noise, a nuisance for 



residents who live near the airport. However, many of the X $3: Premise 
residents in this neighborhood work in construction, and the ' ' 
contract to expand the runways has been awarded to a local 
construction company. Thus, the expansion of the runways will 
lead to an increased quality of life for the residents of this 
neighborhood. 



S4: Conclusion 



There are two broad things we need to study in order to answer Critical Reasoning questions effectively 
and efficiently. We need to understand the specific information given for that question, and we also 
need to know how to conduct the necessary reasoning to answer a question of that type. 

Lets begin first by understanding what we are given. What are the pieces of an argument, how do they 
fit together, and how do we categorize them properly? In later chapters, we'll talk about what we need to 
do with that information. 

On the GMAT: 

(1) All arguments contain at least one premise. A premise is information used by the author to support 
some claim or conclusion. That information may be a fact or an opinion. In the above example, sen- 
tence 3 is a premise because it helps to support the author s conclusion. 



13 



ter 1 



Argument Structure 



|1 (2) Most (though not all) arguments contain a conclusion, the primary claim the author is trying to 

y | 

prove. In the above example, sentence 4 is a conclusion. 

^| 

(3) Many arguments (though not all) contain background information, which provides context to al- 
low us to understand the basic situation. In the above example, sentence 1 provides background. 

(4) Some arguments contain a counterpoint or counterpremise — a piece of information that goes 
against the author s conclusion. In the above example, sentence 2 represents a counterpoint because it 
goes against the authors conclusion. 



Collectively, these categories represent the building blocks of an argument. How do we know which 
sentences fall into which categories? Try to articulate your own thought process for the above argument, 
then take a look at this example "decision process" of a fictional student: 



Argument 


Reader's Thoughts 


The expansion of the runways at the Bay City 
Airport will allow for larger planes to travel to 
and from Bay City. 


Hmm. This is a fact. It could be premise or it 
could just be background. I'm not sure yet. 


These new planes will create a large amount of 
noise, a nuisance for residents who live near the 
airport. 


Now we're moving into claim territory. Something 
negative will come from this project. Why are they 
telling me this? I cant figure that out until I know 
the conclusion. 


However, many of the residents in this neighbor- 
hood work in construction, and the contract to 
expand the runways has been awarded to a local 
construction company. 


The word "however" indicates a contrast between 
sentences two and three. What's the contrast? 
The noise is a negative consequence of the expan- 
sion, while winning a work contract is a positive 
consequence. Looks like I've got a premise and a 
counterpoint in these two sentences, but I don't 
know which one is which yet. 


Thus, the expansion of the runways will lead to 
an increased quality of life for the residents of 
this neighborhood. 


The word "thus" usually indicates a conclusion. 
And, yes, this does seem like a conclusion — this 
project will have a certain outcome (better quality 
of life in this neighborhood), and I can now see how 
the previous two sentences fit into this conclusion. 
Sentence 3 is a premise because it tells me one way 
in which the quality of life might be better for these 
people (they might make more money), and sentence 
2 is a counterpremise because it tells me a nega- 
tive consequence. 



MANHATTAN 
GMAT 



Argument Structure 



Notice how many times the reader thought Tm not sure yet" (or something along those lines). That 
will happen frequently while reading an argument. Were gathering information and trying to under- 
stand what each piece might be, but we won't really know how everything fits together until we know 
what the conclusion is — and that might not be until the end. 



The Core 

The premise (or premises) and conclusion represent the core of the argument. Remember that not all 
arguments will have a conclusion, but all will have at least one premise, so we will always have at least a 
partial core. The core represents what the author is trying to tell me or prove to me. 

Its important for us to be able to identify what specific information in an argument falls into which 
category, because that helps us to take our next step: conducting the necessary reasoning in order to 
answer the question. It turns out that different question types require us to perform different kinds 
of reasoning; we'll discuss this in much more detail in subsequent chapters. Let s take one step now, 
though, just to whet our appetites: how do the premises support the conclusion? In other words, how 
does the "core" actually function in this particular argument? 

In this problem, our core consists of these two pieces: 



However, many of the residents 

in this neighborhood work in . Thus, the expansion of the 

construction, and the contract I runways will lead to an increased 

to expand the runways has been ■ quality of life for the residents of 

awarded to a local construction this neighborhood, 

company. 



The conclusion, on the right, claims that the runway expansion project will have a good outcome (bet- 
ter quality of life for certain people). The premise, on the left, provides one piece of information to sup- 
port this claim: the people in question may make money as a result of this project. 

The premise provides one piece of evidence toward a positive outcome, but the argument is not air- 
tight. For example, do we know for sure that the residents of the neighborhood are the ones who work 
for the local construction company that won the contract? We don't. As we'll see, that kind of thinking 
will help us when we get to the question-answering stage. For now, remember this: when we have both 
a premise and a conclusion, it's critically important to understand how the premise supports the conclu- 
sion. 



Chapter 1 



Argument Structure 




Building Blocks of an Argument 

Lets fully define all of the building blocks we've discussed so far. 



Premise 

• Part of the core of the argument; present in every argument 

• Supports the authors conclusion 

• Can be a fact or an opinion; can be a description, historical information, statistical or 
numerical data, or a comparison of things 

• Often signaled by words or phrases such as because of, since, due to, and as a result of 

Conclusion 

• Part of the core of an argument; present in most arguments 

• Represents the authors main opinion or claim; can be in the form of a prediction, a judg- 
ment of quality or merit, or a statement of causality 

• Is supported by at least one premise 

• Often signaled by words such as therefore, thus, so, and consequently (though note that 
harder arguments might use such a word elsewhere in the argument in an attempt to 
confuse us) 

Background 

• Not part of the core; often present, but not always 

• Provides context to help understand the core 

• Almost always fact-based; can be in almost any form: historical information, numerical or 
other data, descriptions of plans or ideas, definitions of words or concepts, and so on 

Counterpoint 

• Not part of the core; only present occasionally 

• Opposes or goes against the authors conclusion in some way 

• Introduces multiple opportunities for traps: believing that the conclusion is the opposite 
of what it is, mistakenly labeling a counterpoint the premise (and vice versa), and so on 

• Often signaled by transition words such as however, yet, and but', typically, the transition 
word will be found somewhere between the counterpremise and the conclusion (though 
the two sentences may not be right next to each other) 



MANHATTAN 
GMAT 



Argument Structure 



Chapter 1 



Argument Structure 

The argument above used all four of our 1 

Background - Counterpoint - Premise - Conclusion 

We call that the structure because it shows the building blocks used and the order in which each ap- 
peared. The simplest possible argument will contain only premises; its structure might look like this: 

Premise - Premise 

The GMAT can vary the type of building blocks used in a particular argument, and it can also vary 
the order of those building blocks. If we can label the building blocks given in any particular argument, 
that helps us to understand the purpose of each step in the chain of information, and we'll be one good 
step closer to answering the question correctly. 

Lets try some sample arguments. You have two tasks. First, read the argument and try to identify the 
role of each sentence or major piece of information (note that one sentence could contain two different 
pieces of information). Use that information to write out the structure as we just did above. Second, try 
to articulate in your own words how the premises support the conclusion. 

1. Budget Fitness will grow its membership base by 10% in the next six 
months. Budget Fitness has recently crafted a clever ad campaign that it 
plans to air on several local radio stations. 

2. Last year, the Hudson Family Farm was not profitable. However, the farm will 
be profitable this year. The farm operators have planted cotton, rather than 
corn, in several fields. Because cotton prices are expected to rise dramati- 
cally this year, the farm can expect larger revenues from cotton sales than it 
previously earned from corn. 

Answers can be found on page 2L 

Intermediate Conclusions and the Therefore Test 

We have one more building block to introduce in this chapter. Try the below problem. 

The owner of a small publishing company plans to lease a new office space that 
has floor-to-ceiling windows and no internal walls, arguing that the new space 
will enhance worker productivity. The owner cites a recent study showing that 
workers exposed to natural light throughout the day tended to report, on aver- 
age, a higher level of job satisfaction than did those who worked in office spaces 



building blocks; its "structure" looks like this: 




MANHATTAN 
GMAT 



1 



Argument Structure 



that used fluorescent lighting. Thus, the owner concluded, exposure to natural 
light has a positive effect on workers' job satisfaction. 



The owner of a small publishing company plans 
to lease a new office space that has floor-to-ceil- 
ing windows and no internal walls, 


This is likely to be background information because 
it introduces a "plan" to do something. The argument 
is probably about the plan, or a result of the plan. 


arguing that the new space will enhance worker 
productivity. 


This might be the conclusion because it describes the 
predicted future benefit of the company's plan. 


The owner cites a recent study showing that 
workers exposed to natural light throughout the 
day tended to report, on average, a higher level 
of job satisfaction than did those who worked in 
office spaces that used fluorescent lighting. 


And this seems to be a premise in support of that 
conclusion. The workers will be more productive be- 
cause the new space will provide exposure to natural 
light through the floor-to-ceiling windows. 


Thus, the owner concluded, exposure to natural 
light has a positive effect on workers' job satisfac- 
tion. 


Hmm, this is strange. This appears to be the conclu- 
sion as well. It uses the word "thus, " it represents an 
explanation for the study's results, and it even says 
that "the owner concluded" this! 



This is a tough one! In this case, we have two claims that look like the conclusion. Now what? 



This brings us to another building block, the intermediate conclusion (also known as the secondary 
conclusion). What is an intermediate conclusion? Look at this simpler example: 

The burglar is clumsy and often makes a lot of noise while robbing homes. As a 
result, he is more likely to get caught. Thus, in the near future, he will probably 
end up in jail. 

The first sentence is a basic premise: it tells us some factual information about the robber. The second 
sentence is a claim made based upon that premise: because he makes noise, he is more likely to get 
caught. This is a conclusion. . . but, wait, there's a third sentence! That third sentence also contains a 
claim, and this claim follows from the previous claim: because he is more likely to get caught, there is a 
good chance he will end up in jail. 

Essentially, a premise supports a conclusion, and that conclusion then supports a further conclusion. 
The first conclusion is called the intermediate conclusion (also known as the secondary conclusion). 
The second conclusion can be called the final conclusion to distinguish it from the intermediate conclu- 
sion. 

In the example above, the three pieces were given in this order: Premise - Intermediate Conclusion 
- Final Conclusion. Arguments won't always do this, however; they might mix up the order and have 
additional information thrown in. When an argument contains more than one conclusion and we're not 
sure how to classify each, we can use the Therefore Test. 



MANHATTAN 
GMAT 



Argument Structure 



Chapter 1 



We have two conclusions; let's call them A (he's more likely to get caught) and B (he will probably end 
up in jail). All we need to do is plug the two conclusions into two sentences and ask which one is true: 

Is it the case that A (he's more likely to get caught) is true, THEREFORE B (he will 
probably end up in jail) is true? 

Or is it the case that B (he will probably end up in jail) is true, THEREFORE A (he's 
more likely to get caught) is true? 

What do you think? Right, the first scenario makes sense, but the second one doesn't. That tells us that 
B (he will probably end up in jail) is the final conclusion and A (he's more likely to get caught) is the 
intermediate conclusion. 

Let's return to the job satisfaction argument. We have two possible conclusions: 

04) . . .arguing that the new space will enhance worker productivity. 

(B) Thus, the owner concluded, exposure to natural light has a positive effect on work- 
ers' job satisfaction. 

Which scenario makes more sense? 

The new space will enhance worker productivity, THEREFORE exposure to natural 
light has a positive effect on workers' job satisfaction. 

OR 

Exposure to natural light has a positive effect on workers' job satisfaction, THERE- 
FORE the new space will enhance worker productivity. 

The second scenario seems to make sense. That means that (B) is the intermediate conclusion and (A) is 
the final conclusion. 

As is typical of arguments with an intermediate conclusion, the premise supports the intermediate con- 
clusion, which then supports the final conclusion. The premise (the second sentence) says that a study 
found a correlation between natural lighting and job satisfaction. The third sentence in that argument 
then makes a claim based on the study's results: the owner (not the study) concludes that exposure to 
natural light actually causes better job satisfaction. 

The owner claims that the new space will enhance productivity at her company because, first, a study 
showed a correlation between natural light and job satisfaction, and that study then led the owner to 
conclude that natural light results in better job satisfaction. So the first half of the first sentence is back- 
ground, and the second half is the final conclusion. 

The structure is Background - Conclusion - Premise - Intermediate Conclusion. 




MANHATTAN 
GMAT 



Chapter 1 



Argument Structure 




Takeaways 

A premise is a piece of evidence (fact or claim) that supports the authors conclusion. 



A (final) conclusion is the authors main claim. 

An intermediate conclusion is both a conclusion and a premise; it supports the final conclusion. 

Background information helps to set the context for an argument. 

A counterpoint or counterpremise goes against the authors conclusion. 

We can use these building blocks to understand the structure of an argument. Understanding the struc- 
ture will help us to answer the question. 

When we have more than one conclusion, we can use the Therefore Test to find the final conclusion. 
Either "A is true, THEREFORE B is true" or "B is true, THEREFORE A is true." 



MANHATTAN 
GMAT 



Argument Structure 



Chapter 1 



Answer Key 



i. 



Budget Fitness will grow its membership base by 
10% in the next six months. 

Budget Fitness has recently crafted a clever ad 
campaign that it plans to air on several local 
radio stations. 



This is a prediction about the future, so it is a 
claim, not a fact. This is a good candidate to be the 
conclusion. 

Budget Fitness already crafted the campaign — this 
is a fact. It is also a fact that the company currently 
"plans" to air the campaign (though whether it 
will actually air is uncertain, since that is a future 
event). This information supports the claim in the 
first sentence, so it is a premise. 



(Task 1) The structure is Conclusion - Premise. (Task 2) The author claims that the gym will increase 
its membership in the future because the company will implement a strategy (ad campaign) that may 
help attract new customers. 



2. 



Last year, the Hudson Family Farm was not 
profitable. 

However, the farm will be profitable this year. 



The farm operators have planted cotton, rather 
than corn, in several fields. 

Because cotton prices are expected to rise dra- 
matically this year, the farm can expect larger 
revenues from cotton sales than it previously 
earned from corn. 



This is a fact; it already occurred in the past. This 
may be background info, though it may also be a 
premise or counterpoint. 

The word "however" indicates a change in di- 
rection. This prediction is the opposite of what 
happened last year. This future prediction is a good 
candidate to be the conclusion, in which case the 
previous sentence would be a counterpoint. 

Hmm, why do we care which crop the farm is 
planting? 

Okay, now we can see that planting cotton will 
lead to more revenue than last year. The author is 
using this information to support his conclusion. 



(Task 1) The structure is Counterpoint - Conclusion - Premise - Premise. (Task 2) The argument 
predicts that an unprofitable farm will become profitable because a change in crops will result in higher 



revenues. 



Did you spot any flaws in the authors reasoning? There are several, but the biggest one is the fact that 
revenues and profits are not the same thing! A company can have lots of revenue and zero profit — or 
even lose money. 



■T:'X\ 



MANHATTAN 21 
GMAT * 



Argument Structure 



Chapter 1 



Problem Set 




Read the argument and try to identify the role of each sentence or major piece of information. Use that 



1. A program instituted by a state government to raise money allows homeowners to 
prepay their future property taxes at the current rate. Even if the government were 
to raise the tax rate in a subsequent year, any prepaid taxes would allow the home- 
owner to maintain taxes at the lower rate, lowering the overall property tax burden 
over time. For this reason, homeowners should participate in the program. 

2. Tay Sachs disease, a usually fatal genetic condition caused by the build-up of gan- 
gliocides in nerve cells, occurs more frequently among Ashkenazi Jews than among 
the general population. The age of onset is typically six months and generally results 
in death by the age of four. 

3. Some critics have argued that the price of food and drink at Ultralux, a restaurant, is 
too high, given its quality. However, Ultralux features a beautiful interior and com- 
fortable seating, and research has shown that consumers actually enjoy food and 
drink more in such a setting, even when the food and drink is of comparable quality 
to that served elsewhere. Thus, the food and drink at Ultralux is reasonably priced. 

4. Editorial: To stem the influx of illegal immigrants, the government is planning to 
construct a wall along our entire border with Country Y. This wall, however, will do 
little to actually reduce the number of illegal immigrants. Because few economic op- 
portunities exist in Country Y, individuals will simply develop other creative ways to 
enter our nation. 

5. The cutback in physical education is the primary contributing factor to North High 
School's increasing failure rate on the year-end physical fitness examination. Last 
year, when students participated in gym class on a daily basis, 85 percent of the 
school's seniors passed the exam. This year, students had gym class twice weekly, 
and only 70 percent of seniors passed the test. Clearly, fewer sessions of gym class 
lead to reduced fitness. 



information to write out the building block structure. 



MA 



N 



H 



ATTAN 
GMAT 



23 



Argument Structure 



Chapter 1 



Solutions 



A program instituted by a state government to 
raise money allows homeowners to prepay their 
future property taxes at the current rate. 



Even if the government were to raise the tax rate 
in a subsequent year, any prepaid taxes would 
allow the homeowner to maintain taxes at the 
lower rate, lowering the overall property tax bur- 
den over time. 

For this reason, homeowners should participate in 
the program. 



This is a fact. It sounds like background, though it 
could be a premise — Vm not sure yet. People can 
choose to pay future taxes right now at the current 
tax rate. [I'd only want to do this if it saved me 
money.] 

Ah, heres how it could save me money. This is a 
premise. If taxes go up butVve already pre-paid, 
Idont have to pay more; I got to pay at the lower 
rate. [What if tax rates go down? What if I sell my 
house?] 

Conclusion: people should participate. Vve already 
thought of a couple of reasons why it could NOT be 
a good idea. 



The structure is Background - Premise - Conclusion. The author concludes that people should par- 
ticipate because they would save money if taxes go up. 



Tay Sachs disease, a usually fatal genetic condi- 
tion caused by the build-up of gangliocides in 
nerve cells, occurs more frequently among Ashke- 
nazi Jews than among the general population. 
The age of onset is typically six months and gen- 
erally results in death by the age of four. 



This is a fact. Its so general that it sounds like back- 
ground info, though it could be a premise. 



This is also a fact — just more information about 
this disease. Thats interesting. There s no conclusion 
here, just two facts. Both are premises. 



The structure is Premise - Premise. The argument concludes nothing. (Note: two types of questions 
lack conclusions: Inference and Explain a Discrepancy. We'll discuss these later in the book.) 



mi 



3. 

Some critics have argued that the price of food 
and drink at Ultralux, a restaurant, is too high, 
given its quality. 



However, Ultralux features a beautiful interior 
and comfortable seating, 



"Some critics" criticize the restaurant Ultralux for 
being too expensive. The language "some critics" is 
often used in counterpoints; later, the author will 
often tell us something else that the author or others 
believe instead. 

This seems to be pointing out a good thing about 
Ultralux. 



MANHATTAN 
GMAT 



25 



Argument Structure 



and research has shown that consumers actually 
enjoy food and drink more in such a setting, even 
when the food and drink is of comparable quality 
to that served elsewhere. 
Thus, the food and drink at Ultralux is reason- 
ably priced. 



And this tells us why the beautiful interior and 
comfortable seating are beneficial. If we enjoy the 
food and drink more, then perhaps we're willing to 
pay more money? 

This looks like a conclusion. In fact, it directly 
contradicts the critics' argument in the first sentence, 
which we now are sure is a counterpoint. 



The structure is Counterpoint - Premise - Premise - Conclusion. The author concludes that Ultra- 
lux is reasonably priced because research demonstrates that certain beneficial aspects provided by the 
restaurant are valuable to the consumer. 



4. 

Editorial: To stem the influx of illegal immi- 
grants, the government is planning to construct a 
wall along our entire border with Country Y. 



This wall, however, will do little to actually re- 
duce the number of illegal immigrants. 



Because few economic opportunities exist in 
Country Y, individuals will simply develop other 
creative ways to enter our nation. 



The government plans to construct a wall and 
claims that this will reduce the number of illegal 
immigrants. This could be the conclusion, but the 
sentence also starts with the word "Editorial," imply- 
ing that someone with a point of view is writing this 
argument. Til have to see whether that person gives 
a different opinion or claim. 
"However!" Okay, whoever s writing the editorial 
thinks that the government's plan is not going to 
achieve its objective. This is the conclusion, so the 
previous sentence must be a counterpremise. 
"Because" — and here's the reason why the Editorial 
writer thinks this: these illegal immigrants have no 
real opportunities in their own country, so they will 
just search for other ways to get into the neighboring 
country. 



The structure is Counterpoint - Conclusion - Premise. The author concludes that the government s 
plan won't work because the people trying to immigrate illegally will just search for other ways to do so, 
since they don't have many opportunities in their home country. 



The cutback in physical education is the pri- 
mary contributing factor to North High School's 
increasing failure rate on the year-end physical 
fitness examination. 

Last year, when students participated in gym class 
on a daily basis, 85 percent of the school's seniors 
passed the exam. 



This is an opinion, so it could be the conclusion. The 
school isn't offering as much physical education as it 
used to, and the author claims that this is causing 
more students to fail a physical fitness exam. 
Fact. Last year, they had gym class daily, and the 
vast majority of students passed the exam. 



Argument Structure 



Chapter 1 



This year, students had gym class twice weekly, 
and only 70 percent of seniors passed the test. 



Fact. This year, they had gym class less frequently, 
and a smaller percentage of students passed the 



exam. 




Clearly, fewer sessions of gym class lead to re- 
duced fitness. 



Here's another claim. Having fewer gym classes 
causes reduced fitness levels. Is this the conclusion: 
What about the first sentence? 



I need to use the Therefore Test. A = cutback in gym is causing more kids to fail the fitness exam. B = cutback 



Is it the case that cutbacks in gym are causing kids to fail the exam, THEREFORE those cutbacks are causing 



Or is it the case that cutbacks in gym are causing reduced fitness, THEREFORE those cutbacks are causing 
more kids to fail the fitness exam? 

Its the second option — first, the kids have reduced fitness, and then that causes them to fail the fitness exam. 
So the first sentence is the final conclusion, and the last sentence is just an intermediate conclusion. 

The structure is Final Conclusion - Premise - Premise - Intermediate Conclusion. The author con- 
cludes that gym cutbacks are causing kids to fail the fitness exam because this year s seniors had fewer 
gym classes, leading to reduced fitness levels which, in turn, caused more kids to fail the exam. 



in gym causes reduced fitness. 



reduced fitness? 




11 



GMAT 



Critical Reasoning 



Methodology 



see 



Step 1: Identify the Question 
Step 2: Deconstruct the Argument 
Step 3: State the Goal 
Step 4: Work from Wrong to Right 
How to Abbreviate 



Methodology 



In Chapter 1, we introduced arguments, discussed their building blocks, and examined how to "de- 
construct" an argument in order to understand how the pieces of information are related. These tasks 
represent the first two steps of our overall 4-step approach for any Critical Reasoning problem. 

Before we dive into our 4-step process, lets discuss what we don't want to do. While there is a lot of 
flexibility in how different people can work their way through the same problem, there are some ap- 
proaches that are downright bad, such as this one: 

1. Read the argument pretty quickly, don't take notes, don't understand the "big picture" 

2. Read the question 

3. Realize need to read the argument again in order to answer; re-read argument 

4. Re-read question 

5. Examine answers, eliminating one or several 

6. Read the argument yet again 

7. Eliminate another answer 

8. Start checking each answer against the argument and re-reading argument 

9. Repeat until one answer is left 

What's the problem? That's incredibly inefficient! Inefficiency both wastes time and makes it harder for 
us to answer the question correctly. There's too much going on, and that can distract us from our goal. 
So what do we do instead? 

Here's our 4-step approach for all CR questions: 

Step 1: Identify the question. 
Step 2: Deconstruct the argument. 
Step 3: State the Goal. 
Step 4: Work from wrong to right. 



31 



Chapter 2 



Methodology 



Step 1: Identify the Question 

Most arguments are followed by a question (there is actually one exception; we'll discuss this later). 
There are several different types of CR questions, and the wording of the question stem (the part below 
the argument itself) allows us to identify which type of CR question we re about to have to answer. It's 
critically important to identify that question type right away because we need to employ different kinds 
of reasoning depending upon the type of question we have. We want to know, right from the start, how 
best to work through the current problem. 

There are three broad categories of CR questions: the Structure-based family, the Assumption-based 
family, and the Evidence-based family. Each of these families contains a few distinct question types. We 
also have one minor category, the Complete the Argument question type. 

The Structure-Based Family 



These questions all depend upon a solid understanding of the structure of the argument, similar to 
what we discussed in Chapter 1. What pieces do we have and how do they fit together? There are two 
types of Structure questions, both of which we'll discuss in Chapter 3: 



Question Type 


Sample Question Phrasing 


Goal 


Describe the 
Role 


In the argument given, the two boldface por- 
tions play which of the following roles? 


Identify the roles of the bold- 
face portions. 


Describe the 
Argument 


In the passage, the mayor challenges the coun- 
cilmember's argument by doing which of the 
following? 


Describe the structure of the 
argument. 



The Assumption-Based Family 

These questions all depend upon an understanding of the assumptions made by the author to reach a 
certain conclusion. What is an assumption? 

First, an assumption is something that the author does not state in the argument; for this reason, we call 
assumptions unstated. An assumption is, however, something that the author must believe to be true in 
order to draw the given conclusion. 

We'll go into much more detail on assumptions in chapter 4 but let's look at a short example: 

That car is green. Therefore, that car cannot belong to Dan. 

If we re only told that the car is green, how can we know for sure that it doesn't belong to Dan? Clearly, 
there's some information missing. What is the author assuming here? 

The assumption: Dan does not have a green car. 



MANHATTAN 

GMAT 



Methodology Chapter 2 

If we were to insert the assumption into the argument, it would make the argument stronger: 

That car is green. Dan does not have a green car. Therefore, that car cannot belong to 
Dan. 

In this case, it not only makes the argument stronger, it makes the argument "air tight" — we can't 
argue with it! That wont always happen, but the assumption should make the argument significantly 
stronger. 



There are five types of Assumption questions, which we'll cover in chapters 4 and 5. 



| Question Type 


Sample Question Phrasing 


Goal 


| Assumption 


The argument depends on which of the 
following assumptions? 


Identify an unstated assumption. 


Evaluate 


Which of the following must be studied in 
order to evaluate the argument above? 


Identify a piece of information 
that would help to determine the 
soundness of the conclusion. 


Flaw 


Which of the following indicates a flaw in 
the reasoning above? 


Identify something illogical in 
the argument. 


Strengthen 


Which of the following, if true, provides 
the most support for the argument above? 


Strengthen the authors conclu- 
sion. 


Weaken 


Which of the following, if true, most seri- 
ously weakens the argument? 


Attack the authors conclusion. 



The Evidence-Based Family 

These questions all lack conclusions; they consist entirely of premises! Were then asked to find some- 
thing that must be true or something that eliminates a discrepancy in order to answer the question. We'll 
discuss both of these question types in Chapter 6. 



Question Type 


Sample Question Phrasing 


Goal 


Inference 


Which of the following can be logically 
concluded from the passage above? 


Identify something that must 
be true based upon the given 
information 


j Explain a 
Discrepancy 


Which of the following, if true, most helps 
to explain the surprising finding? 


Identify something that 
eliminates some discrepancy or 
paradox given in the argument. 



There is also a minor type called Complete the Argument. We'll discuss this type in its own separate 
chapter; for now, know that you want to prioritize the three major families during your study. 



MANHATTAN 
GMAT 



Chapter 2 



Methodology 



As we go through each of the families and their question types, we will learn what kind of language 
signals specific question types — and that's our first big step in our 4-step approach. 

Step 2: Deconstruct the Argument 

Now that we've identified the family and question type, we can use that to help us deconstruct the 
argument. We began to learn how to do this in the previous chapter when we labeled arguments using 
the building block components. We'll learn even more about how to do this in later chapters, when we 
begin discussing each question type in detail. 

In order to accomplish this argument deconstruction, many people take some light notes. Some people 
are able to deconstruct the argument and remember the pieces without taking notes, but most people 
do take some notes. If you aren't sure which way is best for you, try taking notes for a couple of weeks; 
you need some time to develop a good method and learn to work efficiently. Most people find that, the 
more they practice, the less they have to write, and some people even get to the point where they only 
have to write notes on the longest, most convoluted arguments. 

In other words, you don't have to take notes, but don't underestimate the value of learning to take good 
notes — this is a powerful tool that can help us accomplish our main goal: deconstructing arguments 
efficiently and effectively. 

These notes need to be neat enough to read quickly and easily, but they are also going to be heavily 
abbreviated. These are not the kind of notes you take during a class, when you have to write everything 
down thoroughly so that you can study for the test 3 weeks from now. 

Rather, these notes will actually help us to think our way through the problem — we want to use them to 
understand the structure and flow of the information. By the time we're done reading and taking notes, 
we have maybe 60 to 90 seconds left. We can abbreviate extremely heavily and still remember what 
those abbreviations mean in another 60 to 90 seconds. 

Lets revisit the first argument that we did in Chapter 1. What might the notes look like? 

The expansion of the runways at the Bay City Airport will allow for larger planes 
to travel to and from Bay City. These new planes will create a large amount of 
noise, a nuisance for residents who live near the airport. However, many of the 
residents in this neighborhood work in construction, and the contract to expand 
the runways has been awarded to a local construction company. Thus, the expan- 
sion of the runways will lead to an increased quality of life for the residents of this 
neighborhood. 



34 MANHATTAN 

GMAT 



Methodology 



Chapter 2 



Here's one method, idea by idea: 

BC rnwy T -> bigger planes -* t noise, bad for res 

BUT res = constr work, local com doing work 

(C) rnwy f -> better life for res 

The first line encompasses the first two sentences of the argument. Most people would probably write 
down only the first sentence first: 

BC rnwy T -> bigger planes 

Then, as we continue reading, we realize that the second sentence followed on 
from the first: those bigger planes then cause more noise. As a result, we can 
just continue that same line, even though the additional information is given 
in a new sentence. 

If someone who hasn't read the original argument looks at our notes, then 
our notes would look like nonsense — and, in fact, they should be abbreviated 
enough that, if we were to re-read just the notes in a week or two (after forget- 
ting the argument), we should not be able to tell what the full argument was. 
If, a week later, we can reconstruct the entire argument just from our notes, 
then we wrote too much down. 

Let's try two more. Give yourself about 30 to 45 seconds to create notes for 
the below arguments that we saw in chapter 1, incorporating the techniques 
you've learned in this chapter. 

1. Budget Fitness will grow its membership base by 10% in the next six 
months. Budget Fitness has recently crafted a clever ad campaign that it 
plans to air on several local radio stations. 

2. Last year, the Hudson Family Farm was not profitable. However, the farm will 
be profitable this year. The farm operators have planted cotton, rather than 
corn, in several fields. Because cotton prices are expected to rise dramati- 
cally this year, the farm can expect larger revenues from cotton sales than it 
previously earned from corn. 



TIP 

When first learning this method, most 
people do write too much. As part of your 
review of problems, ask yourself, "Did I 
write this down in the most efficient and 
effective way? Did my notes make sense for 
short-term use? Did I write down something 
that I could've just skipped, or did I use too 
many words to write something down when 
I could've abbreviated more?" If you were 
really off the mark, make yourself write out 
the notes again in a more ideal way — and 
ask yourself why this new way is better than 
the old way. Now you're learning how to do 
a better job on the next new problem! 



MANHATTAN 

GMAT 



er 2 Methodology 
Answer Key 



Below are sample representations of notes for the two given arguments. Your notes might differ quite a 
bit from the samples shown below. That's fine as long as your notes accomplish the following purposes: 

• clearly delineate a conclusion (if there is one) 

• demonstrate the "flow" of information (how one piece of info relates to the next, where 
applicable) 

• indicates contrasts or changes of direction 

1. Budget Fitness will grow its membership base by 10% in the next six months. Budget 
Fitness has recently crafted a clever ad campaign that it plans to air on several local radio 
stations. 



Sample 1 BF new ad camp to air BF member t 10% in 6 mo. © 

Sample 2 © BF mbrs > 10% 6 mos. 

BF to put new ads on radio 

In this argument, the conclusion was in the first sentence, so we may write down that info before we 
know that it is the conclusion. The second sentence actually leads to the first sentence, so if we have 
room to do so on our scrap paper, we could just write that information to the left of the conclusion. If 
so, we might end up with something that looks like Sample 1. Alternatively, we might write down each 
"big idea" on its own line, and then use an arrow to show that the second line leads to the first one, 
similar to Sample 2. 

In both cases, we label the conclusion clearly once we've found it (and, again, you can use any "this is 
my conclusion" label that you want, as long as you consistently use the same label every time). 

2. Last year, the Hudson Family Farm was not profitable. However, the farm will be profit- 
able this year. The farm operators have planted cotton, rather than corn, in several fields. 
Because cotton prices are expected to rise dramatically this year, the farm can expect 
larger revenues from cotton sales than it previously earned from corn. 

Sample 1 - B4, HFF not prof 

✓--►© BUT will be now [why?] 
+ cotton, not corn 
> + cot $ 1 1 so > rev from cot than corn 
[cost to grow corn?] 



Sample 2 



now 



not cot $ > cot rev © will 

prof t than corn be prof 



MANHATTAN 
GMAT 



Methodology 



In Sample 1, we ve used some pluses and minuses, along with the usual conclusion symbol, to classify 
each line. A "plus" indicates a premise: something the author is using to support the conclusion. A "mi- 
nus" indicates a counterpremise: something that does not support the conclusion. 

Sample 2 shows us a timeline. If we ve got a future prediction, along with some past background info, 
this can be an effective way to show the sequence of events. The first two sentences tell us that we ve got 
a past/future situation in this argument, so we can tell at the beginning that a timeline might work. 

Notice that we also added one new type of note to Sample 1: the bracketed notes [why?] and [cost to 
grow corn?]. As we take notes on the argument itself, we might also want to jot down notes about what 
were thinking. It wasn't profitable before, but it will be now? Why? So we're already thinking about 
that as we continue to read the argument. Later, the argument says the farmers can earn more revenue 
from the cotton, but the conclusion said something about profits. Profit equals revenue minus costs. 
We've been given some evidence that we may be able to make more money from cotton (and even that's 
debatable), but we've been told nothing about costs, so how can the argument conclude anything about 
profits? 

The argument told us that cotton prices are going up; it follows then, that farmers will make more 
money on the same amount of cotton this year than they did on the same amount of cotton last year. 
How do the prices of cotton and corn compare? We have no idea. It's entirely possible that cotton prices 
have increased but are still lower than corn prices. That's a subtle point, but if you noticed that, you 
might have wanted to jot down a note so that you wouldn't forget as you continued through the prob- 
lem. 



Chapter 2 



Methodology 




Step 3: State the Goal 



This is a short but often overlooked step: what exactly am I trying to do when I answer this question? 
What's my goal? I know what kind of question I have, I understand the argument and how it fits to- 
gether now, I know my conclusion (if there is one). . . now what? 

At this stage, we need to remind ourselves what it is were actually trying to do when we start attacking 
the answers, and this goal depends upon the type of question that we have. Each question type requires 
a certain kind of reasoning and demands certain characteristics from the correct answer. There are also 
common types of wrong answer traps. Before we dive into the answers, we want to remind ourselves 
(briefly) of our goal and any traps that we want to avoid. We'll learn all about these things in later 
chapters. 



Step 4; Work from Wrong to Right 

Finally, the answer choices! On verbal in general, we're asked to find the "best" answer. We're going to 
use a two-step process in order to accomplish this. First, we look through all five answers and eliminate 
as many "definitely wrong" answers as we can. On this first pass through the answers, we're not actu- 
ally trying to decide which is the right one, only which ones are definitely wrong. 

If we only have one answer left, great; we're done. If we have two or more answers left, then we compare 
those remaining answers. 

Why do we do it this way? By definition, finding the best answer is a comparison; if I spot a tempting 
answer, I can't know whether it's the best one until I've seen all of the others. It's most efficient to dump 
all of the "no way" answers as fast as we can, and then directly compare the remaining, more tempting 
answers. 

Finally, we have one last important rule to remember for verbal questions: when we've narrowed down 
to two answers, we should look at each answer and compare the two once more, but then we should 
pick and move on. Going back and forth multiple times is a waste of time — either we know it after 
comparing the first time or we don't. 

As we go through and assess these answers, it's critical to keep track of our thinking — we're actually 
going to track what we think about each of the five answers as we go. There are two big decisions to 
make in terms of how you choose to do this. 



MANHATTAN 
GMAT 



Methodology 



Chapter 2 



Decision #1: How do I write down ABCDE? 



What to Do 


Pros 


Cons 


Write ABCDE for each 
question 


Can write directly on each letter; 
can keep letters with notes about 
argument 


Have to write 41 separate times 


Write ABCDE at the top 
of the page, then move to a 
new line for each question 


Only have to write once for each 
page (several times for entire test) 


Have to keep track "below" each 
letter; notes for problem might not 
be right next to answer tracking 
row 



Option 1 (write for each question) might look like this: 



WA 

notes 
notes 



SAB^DE 



notes 
notes 




Option 2 (write once per page) might look like the below, where the first question (a Weaken) is an- 
swered in the first row and the second question (a Strengthen) is answered in the second row. Remem- 
ber that the scrap paper will be graph paper, so there will already be lines built-in to separate the five 
answer choices. 



A 


B 


C 


D 


E 




/ 


/ 


/ 


W 








notes 








\ 


notes 










\ 










notes 










notes 



MANHATTAN 
GMAT 



Methodology 



Decision #2: What symbols will I use to keep track of my thoughts? 

We need four symbols in order to keep track of our thoughts on the answers; you can use any symbols 
you prefer as long as you consistently use the same symbols to mean the same things: 



x or/ 


Definitely wrong 




Maybe 


p 


I have no idea 


o 


This is it! 



Let s try all of this on an actual problem. 

Over the past decade, many companies have begun using automated telephone 
services; callers hear a machine-generated voice and are able to select options us- 
ing the numbers on the telephone keypad. Research shows that callers are more 
patient when the machine-generated voice is that of a woman. Thus, smaller 
companies that cannot afford an automated service should consider hiring 
women, rather than men, to interact with customers by phone. 

Which of the following, if true, would be most damaging to the conclusion above? 

(A) Automated telephone services are becoming cheaper and cheaper every 
year. 

(B) Patient customers tend to order more products and return fewer products 
than impatient customers. 

(C) A separate study indicated that the extra patience exhibited by callers is 
limited to interactions with an automated system. 

(D) Some customers prefer automated systems to talking with a live person. 

(E) On average, callers are only slightly more patient when interacting with a 
female voice, rather than a male voice, in an automated telephone system. 

How did you do with each step? Did you identify the question type? Do you feel comfortable with your 
notes, and did you identify the conclusion (if there is one)? Did you remember to state the goal (briefly) 
before looking at the answers? Did you use the 2-pass process to assess the answer choices? 

Here's how someone might work through the above problem. We'll show each of the four steps sepa- 
rately. The first column will show the relevant text from the problem. The second column will show 
what we might write on the scrap paper. The third column will show what we might be thinking while 
working on the problem. 



Methodology 



Chapt 



Step 1 : Identify the Question 

Which of the following, if true, 
would be most damaging to the 
conclusion above? 



W 



A B C D E 



Step 2: Deconstruct the argument 

Over the past decade, many 
companies have begun using 
automated telephone services; 
callers hear a machine-generated 
voice and are able to select op- 
tions using the numbers on the 
telephone keypad. 
Research shows that callers are 
more patient when the machine- 
generated voice is that of a 
woman. 

Thus, smaller companies that 
cannot afford an automated 
service should consider hiring 
women, rather than men, to in- 
teract with customers by phone. 



lOy: corns use auto 
phone 



R: female = 
t patience 



Small com -» use 
women phone 



"most damaging to the conclusion" 
means this is a Weaken. I need to find 
the conclusion, and I need to think 
about what flaws or gaps might exist be- 
tween the premises and the conclusion. 



Sounds like background, but V 11 jot 
down a note anyway. 

This is describing what an automated 
phone system is; I probably dont need to 
write that down. 

This is a fact, not a claim, so it has to be 
either a premise or counterpremise. Its 
probably a premise, since there s only one 
sentence left. 

This is the only claim, so its the con- 
clusion. Now I can go back and add a 
© to the conclusion in my notes and a 
+ to the premise. 



The final notes might look something like this: 

W ABCDE 

lOy: corns use auto phone 

+ R: female = t patience 

© Small com use women phone 

Your notes might look very different from the above notes. That's perfectly fine as long as your notes 
convey the basic flow of information clearly and concisely. Your notes also need to identify the question 
type, and you need some mechanism by which to track your answers. 



GMAT 



er2 



Methodology 



Step 3: State the Goal. 

The conclusion is that small companies should hire women to answer the phones, because callers are more 
patient when hearing automated female voices. 

I need to weaken the conclusion, so that would mean there's some reason why companies might not be better 
off hiring women to answer the phones. 

[Hmm. The evidence is about automated female voices, while the conclusion is about real women. Is there 
any kind of disconnect there?] 

First, we briefly restate the core of the argument — the conclusion and the main reasoning that supports 
that conclusion. Then we articulate what kind of answer would accomplish our goal — in this case, to 
weaken the conclusion. We may also happen to notice significant discrepancies, and we can articulate 
those at this stage as well. 

Step 4: Work from wrong to right 

Now, were ready to attack the answers. 



(A) Automated telephone ser- ABCDE 
vices are becoming cheaper and 
cheaper every year. 



(B) Patient customers tend to ABCDE 
order more products and return 

fewer products than impatient 
customers. 

(C) A separate study indicated ABCDE 
that the extra patience exhibited 

by callers is limited to interac- 
tions with an automated system. 



(D) Some customers prefer auto- A 8 C B E 
mated systems to talking with a 
live person. 



The conclusion discusses what companies should 
do when they can t afford automated services. If 
the service becomes cheap enough that a particu- 
lar company can buy it, that company no longer 
has to worry about whether to hire women or 
men to answer the phones. 
This is a good reason for the company to do 
whatever it can to keep its customers in a patient 
mood. If anything, that would strengthen the 
argument. 

Hmm. This creates a distinction between auto- 
mated and live voices. . . I was wondering earlier 
whether that might be the disconnect. There 
doesnt seem to be any evidence now that a live 
female voice will make callers more patient. Keep 
this one in. 

Presumably these customers would be more 
patient with an automated system. . . oh, but 
this argument is only about those companies who 
cant afford the system and are using real people. 
Nope, this isn't it. 



MANHATTAN 
GMAT 



Methodology 



Chapter 2 



(E) On average, callers are only 
slightly more patient when 
interacting with a female voice, 
rather than a male voice, in an 
automated telephone system. 



A fi @ © E This one seems to be telling me there isn't a huge 
difference between male and female voices — but 
there is still a small positive effect for female voic- 
es. If anything, this strengthens the argument; 
after all y as a small business owner, I'll take any 
necessary steps that will get me more business! I 
only have one answer left, so C is the answer. 



lilii 



How to Abbreviate 

A clear, consistent shorthand (abbreviation) method will help us to take notes efficiently and spend 
more of our mental energy focused on how the argument works (rather than how to write down a par- 
ticular piece of information). 



The chart below contains some symbols and abbreviations that are especially useful for Critical Reason- 
ing. As you study, make sure to develop your own. 



Increase / more / high 


t 


Decrease / less / low 


I 


Causes / leads to / results in 


-> 


Was caused by 




Greater than / more than / 


> 


Less than / smaller 


< ; 


majority 




than / minority 




Equals / correlates with 




Number 


# 


Price / dollar amount 


$ 


Percent 


% \ 


Change 


A 


Women / Men 


W/M 


Best / most effective 


★ 


Worst / least effective 


X 


Attribution (e.g. the Mayor 
said. . .) 


e.g. M: 


Like / dislike 


©/© 


Future / prediction (something 
will happen, someone plans to 
do something) 


F 


Century (e.g., 20th 
century) 


c 

e.g. 20c 


Time 


t 


However / altnough 
/ etc 


BUT 


years 


y 


conclusion 


© 


your own thoughts (not in the 
argument) 


[your own thoughts 
in brackets] 


Profit, Revenue, Cost 


P = R-C 


premise 


+ (plus) 


counterpremise 


- (minus) 



MANHATTAN 
GMAT 



Methodology 



For very large increases or decreases, a very large majority or very small minority, and so on, double the 
symbol. For example, for a very large increase in the number of employees, write 1 1 # emp. 

For the profit formula, do write out the whole formula even if the argument mentions only profit, or 
only profit and either revenues or costs. All three variables go together (and that fact is often the weak 
point for a question that mentions profit). 

For any names, unfamiliar "big" words, or other unusual words, simply use the first letter of the name 
or word. In traditional note-taking, that wouldn't be adequate, but we only need to remember for about 
90 seconds, and a single-letter abbreviation is sufficient to remember for 90 seconds. 

Takeaways 

Our 4-step approach for all CR Questions is: 

Step 1 ; Identify the question, 

• we'll learn how to do this in later chapters 

• the question type tells us what kind of information we expect to find in the argument and 
what kind of reasoning help to answer the question 

Step 2: Deconstruct the argument 

• break the argument down into its building blocks 

• take very abbreviated notes showing both the details and the "flow" of the information 

Step 3: State the Goal, 

• very briefly articulate your goal based upon this question type (again, we'll learn the goals 
for each type in later chapters) 

Step 4: Work from wrong to right 

• plan to go through the answers twice 

• on the first pass, focus on eliminating anything that is definitely wrong; leave everything 
else in 

• on the second pass, compare any choices that remain, then pick 

Know how you're going to keep track of your answers on your scrap paper. First, decide whether to have 
a separate ABCDE grid for each problem or whether to use the "write once per page" method described 
earlier in the chapter. Second, make sure you have four consistent symbols for these four labels: defi- 
nitely wrong; maybe; I have no idea; and this is it! 



Methodology 

Problem Set 



Read the argument and try to identify the role of each sentence or major piece of information. Take ab- 
breviated notes for the argument. Use that information to write out the building block structure. 

1. A series of research studies has reported that flaxseed oil can have a beneficial effect 
in reducing tumor growth in mice, particularly the kind of tumor found in human 
postmenopausal breast cancer. Thus, flaxseed oil should be recommended as an ad- 
dition to the diets of all postmenopausal women. 

2. During the past thirty years, the percentage of the population that smokes ciga- 
rettes has consistently declined. During the same time period, however, the number 
of lung cancer deaths attributed to smoking cigarettes has increased. 

3. The Chinese white dolphin is a territorial animal that rarely strays far from its habitat 
in the Pearl River Delta. In recent years, increasing industrial and agricultural runoff 
to the Delta's waters has caused many white dolphins to perish before they reach 
breeding age. Unless legislation is enacted to ensure there is no further decline in 
the Delta's water quality, the Chinese white dolphin will become extinct. 

4. Most doctors recommend consuming alcohol only in moderation, since the ex- 
cessive intake of alcohol has been linked to several diseases of the liver. Drinking 
alcohol is no more dangerous for the liver, however, than abstaining from alcohol 
entirely. Last year, more nondrinkers than drinkers were diagnosed with liver failure. 

5. To increase the productivity of the country's workforce, the government should 
introduce new food guidelines that recommend a vegetarian diet. A study of 
thousands of men and women revealed that those who stick to a vegetarian diet 
have IQs that are around five points higher than those who regularly eat meat. The 
vegetarians were also more likely to have earned advanced degrees and hold high- 
paying jobs. 



Methodology 



Chapter 2 



Solutions 



Note: the sample notes show in the answer key represent only one example of how someone might take 
ntoes. Just make sure that your notes are legible, very concise, and convey the main points in a manner 
that makes sense to you. 



1. 



Argument 

A series of research studies has reported 
that flaxseed oil can have a beneficial ef- 
fect in reducing tumor growth in mice, 
particularly the kind of tumor found in 
human postmenopausal breast cancer. 

Thus, flaxseed oil should be recom- 
mended as an addition to the diets of all 
postmenopausal women. 



Notes 



R: Flax helps 1 tumor mice 
esp PM BC 



R: flax helps 1 tumor mice 
esp BC 

1 PM women shd take flax 



Thoughts 

This is a fact. It's 
either background or a 
premise. 



Definitely the conclu- 



sion. 




The structure of this argument is Premise - Conclusion. 



2. 



Argument 

During the past thirty years, the 
percentage of the population that 
smokes cigarettes has consistently 
declined. 

During the same time period, 
however, the number of lung cancer 
deaths attributed to smoking ciga- 
rettes has increased. 



Notes 

30y: % pop smoke cig I steady 



30y: % pop smoke cig I steady 
same P: # LC dead from cig t 



Thoughts 

This is a fact. Its 
either background or a 
premise. 

Another fact, so an- 
other premise. There 
isn't a conclusion. 



The structure of this argument is Premise - Premise. 



MANHATTAN 47 
GMAT 



Chapter 2 



Methodology 



3. 




Argument 

The Chinese white dolphin is a ter- 
ritorial animal that rarely strays far 
from its habitat in the Pearl River 
Delta. 

In recent years, increasing indus- 
trial and agricultural runoff to the 
Delta s waters has caused many 
white dolphins to perish before they 
reach breeding age. 

Unless legislation is enacted to 
ensure there is no further decline 
in the Deltas water quality, the 
Chinese white dolphin will become 
extinct. 



Notes 

CWD stays in PRD 



The structure of this argument is Premise - Premise - Conclusion. 



Argument 

Most doctors recommend con- 
suming alcohol only in modera- 
tion, since the excessive intake 
of alcohol has been linked to 
several diseases of the liver. 

Drinking alcohol is no more 
dangerous for the liver, howev- 
er, than abstaining from alcohol 
entirely. 

Last year, more nondrinkers 
than drinkers were diagnosed 
with liver failure. 



Notes 



Drs rec I ale be t ale liver 
dis 



Drs rec I ale be T ale liver 
dis 

@ drink not worse than abstain 

Drs rec I ale be T ale liver 
dis 

drink not t bad than abstain 
ly: >nondrink had liv dis 



CWD stays in PRD 

rent: ind + ag in PRD 
CWD die b4 breed 



CWD stays in PRD 

rent: ind + ag in PRD -> 
CWD die b4 breed 

IF govt doesn't fix H20 -* 
CWD extinct 



Thoughts 

This is a fact. It's either 
background or a premise. 



This is also a fact but 
is more like a premise 
because it feels like it could 
build to a conclusion. 

And here's the conclusion. 

[Note: H20 here is an ab- 
breviation for water, based 
on the chemical formula 
H 2 O.J 



Thoughts 

This is a fact. Its either back- 
ground or a premise. 



Oh, this has the word "how- 
ever!" The last sentence was a 
counterpremise, and this one 
sounds like the conclusion. 

This supports the previous 
sentence; it's a premise. [It also 
seems pretty flawed. What 
percentage of nondrinkers vs. 
drinkers had liver disease?] 



The structure of this argument is Counterpremise - Conclusion - Premise. 



48 



GMAT 



Methodology 



Chapter 2 



5. 



Argument 

To increase the productivity of 
the country's workforce, the 
government should introduce 
new food guidelines that rec- 
ommend a vegetarian diet. 

A study of thousands of men 
and women revealed that those 
who stick to a vegetarian diet 
have IQs that are around five 
points higher than those who 
regularly eat meat. 

The vegetarians were also more 
likely to have earned advanced 
degrees and hold high-paying 
jobs. 



Notes 



govt shd rec veg to t wrkr 
prod 



govt shd rec veg to T wrkr 
prod 

S: veg t IQ than non-veg 



govt shd rec veg to t wrkr 
prod 

S: veg t IQ than non-veg 
veg > better schl and high pay 



Thoughts 

This is definitely a claim. 
It sounds like a conclusion, 
though Idont know for sure 
yet. 

This is a fact — the results of a 
study. It also supports the claim 
above, so it's a premise. 




This is another premise sup- 
porting the first sentence. 



The structure of this argument is Conclusion - Premise - Premise. 



GMAT 



49 



Critical Reasoning 



Structure-Based Family 



On 7& Chapter. . . 

Describe the Role 
Describe the Argument 



Structure-Based Family 



In the first two chapters, we introduced arguments, examined the building blocks used to construct 
them, and learned the overall 4-step approach to tackling any Critical Reasoning question. We also 
introduced the main types of questions found on the test. Here's our 4-step approach: 

Step 1: Identify the question. 

Step 2: Deconstruct the argument. 

Step 3: State the Goal. 

Step 4: Work from wrong to right. 

Now, we're going to begin tackling the first of our three main Critical Reasoning Families: the Struc- 
ture-Based questions. As the name implies, these questions depend upon our ability to understand the 
structure of the argument. What kinds of building blocks are present in the argument? Which piece 
leads to which piece? What is the purpose of each piece of info — what role does it play? 

There are two main Structure question types: Describe the Role and Describe the Argument. 

Describe the Role 

Of the two types, Describe the Role (or Role, for short) is more common. These questions present a 
standard argument, but one or two portions of that argument are presented in boldface font. We are 
asked to describe the role each portion of boldface font plays. 

What does "role" mean? We actually already studied this. The "role" is just another name for building 
block. A particular bolded portion could be a premise, a conclusion, a counterpremise, an intermediate 
conclusion, or background information. It could also be a counter-conclusion or opposing conclusion, 



53 



er 3 



Structure-Based Family 



something we didn't discuss earlier. An opposing conclusion is simply a conclusion that goes against the 
authors main conclusion. 

These question types are easy to identify because one or two statements (usually two) will be presented 
in bold font, and the question stem will include the word "boldface." 

Our task here is to determine the role that each boldface statement plays in the argument. We're going 
to discuss two possible methods. The Primary Method will always work, but it's more complicated and 
time-consuming to use. The Secondary Method will allow us to narrow down answer choices more 
easily but may not allow us to get all the way to one answer — that is, we may have to guess from a nar- 
rowed set of answers. 



Primary Method 

There are three possible roles: 

(C) The statement in boldface is the author's CONCLUSION. 

(P) The statement in boldface is a PREMISE (it supports the author's 
conclusion) 

(X) The statement in boldface is SOMETHING ELSE (this might be a 
counterpremise, background information, acknowledgement of a weak- 
ness in the argument. . .) 

Strategy Tip: Labels C and P are considered to be "on the same side" 
(because both are part of the author's argument). 

Strategy Tip: Label X is considered to be "on the opposite side" of labels 
C and P, because label X does not support the author's argument. 

In our notes, we'll classify each statement using the labels C, P, or X, as described above. When we 
evaluate the answer choices, we'll look for matching language based upon our labels. 

How would that work? Let's say that we've decided to label the first boldface statement with an X and 
the second boldface with a C. Then we check our answers for an XC pattern. 

The answer choices are the most difficult part of Structure questions in general because they are written 
in an abstract form. For example, an answer might read: 

(A) The first [boldface statement] is evidence that has been used to weaken 
a claim made by the argument; the second [boldface statement] is that 
claim. 

The first half of that sentence is quite convoluted. Let's start with the most basic piece: a building block. 
There is a claim made by the argument; the claim is the conclusion. This first half says that the first 



MANHATTAN 
GMAT 



Structure-Based Family 



Chapters 



boldface weakens the conclusion. Something used to weaken the conclusion is a counterpremise. If we 
labeled the first boldface statement with an X, then this might be the right answer. 

The second half of the sentence is more straightforward but includes a structure that is commonly used 
to try to confuse us. It refers back to something that was said in the first half of the answer choice. In 
this case, the second half refers to "that claim." It s not just talking about any claim here; its talking 
about the same claim that was mentioned in the first half of the sentence. The second half is describing 
the conclusion; if we labeled the second boldface statement a C, then this might be the right answer. 

Great! We wanted an XC combo (in that order), and we just found an answer choice that gives us an 
XC combo. We re done! 

If we can use the above method accurately, we will be able to eliminate the 4 wrong answers and get to 
the right answer. We might struggle to do that, though, or it might take too much time. Our Second- 
ary Method allows us to get rid of some answers more quickly before taking a guess from among the 
remaining answers. 

Secondary Method 

There are three possible roles: 

(C) The statement in boldface is the authors CONCLUSION. 
(F) The statement in boldface is a FACT. 

(O) The statement in boldface is an OPINION (but not the conclusion). 

Strategy Tip: Check for the conclusion first. Only label something an O 
if it is NOT the conclusion. 



How would this work on our answer choice from above? This time, lets say that we have labeled the 
first boldface from our argument with an F and the second boldface with a O (opinion but not conclu- 
sion). Next, we check the answers. 

(A) The first [boldface statement] is evidence that has been used to weaken 
a claim made by the argument; the second [boldface statement] is that 
claim. 

The word "evidence" typically indicates a fact, not an opinion, so the first half is likely describing an F 
label. The second half is still describing the conclusion, so it would receive a label of C. 

That doesn't match. We re looking for an FO combo, but this answer gives us an FC combo. Eliminate it. 



MANHATTAN 
GMAT 



Chapter 3 



Structure-Based Family 



Common Trap Answers 

The most tempting trap answers on Role questions tend to be "off by just one word, and that word is 
usually at the end of the sentence. For instance, let s imagine that we've decided the first boldface is a 
premise in support of the author's conclusion — in other words, a P. A tempting wrong answer might 
read: 




(A) The first [boldface statement] provides evidence in support of the position 
that the argument seeks to reject. 

Every word of that answer matches what we want to find with the exception of the very last word, "re- 
ject." In fact, if we changed that one word, the answer would be correct: 

(A) The first [boldface statement] provides evidence in support of the position 
that the argument seeks to establish. 

The first version of the answer choice says that the first boldface is a premise in support of some 
counterconclxxsion. That's an X label, not a P. The second version says that the first boldface is a premise 
in support of the author's conclusion, and that is, indeed, a P label. If we're not reading every word very 
carefully, we may pick the first version without even realizing that it's an X, not a P! 



MANHATTAN 
GMAT 



Structure-Based Family 



Chapter 3 



Putting It All Together 



Let s try a full example: 

Mathematician: Recently, Zubin Ghosh made headlines when he was recognized 
to have solved the Hilbert Conjecture. Ghosh posted his work on the internet, 
rather than submitting it to established journals. In fact, he has no job, let alone 
a university position; he lives alone and has refused all acclaim. In reporting on 
Ghosh, the press unfortunately has reinforced the popular view that mathemati- 
cians are antisocial loners. But mathematicians clearly form a tightly knit com- 
munity, frequently collaborating on important efforts; indeed, teams of research- 
ers are working together to extend Ghosh's findings. 

In the argument above, the two portions in boldface play which of the following 
roles? 

(A) The first is an observation the author makes to illustrate a social pattern; the 
second is a generalization of that pattern. 

(B) The first is evidence in favor of the popular view expressed in the argument; 
the second is a brief restatement of that view. 

(C) The first is a specific example of a generalization that the author contradicts; 
the second is a reiteration of that generalization. 

(D) The first is a specific counterexample to a generalization that the author as- 
serts; the second is that generalization. 

(E) The first is a judgment that counters the primary assertion expressed in the 
argument; the second is a circumstance on which that judgment is based. 




M 



N 




57 



GMAT 



Chapter 3 



Structure-Based Family 



Step 1 : Identify the question. 




In the argument above, the two R 
portions in boldface play which 
of the following roles? 



A B C D E 



This is a Role question. The argu- 
ment contains bold font, and the 
question stem contains the words 
"boldface" and "role." 



Step 2; Deconstruct the argument 



Mathematician: Recently, Zubin 
Ghosh made headlines when he 
was recognized to have solved 
the Hilbert Conjecture. 
Ghosh simply posted his work 
on the internet, rather than 
submitting it to established 
journals. 

In fact, he has no job, let alone 
a university position; he lives 
alone and has refused all ac- 
claim. 

In reporting on Ghosh, the press 
unfortunately has reinforced the 
popular view that mathemati- 
cians are antisocial loners. 



But mathematicians clearly 
form a tightly knit commu- 
nity, frequently collaborating 
on important efforts; indeed, 
teams of researchers are work- 
ing together to extend Ghosh s 
findings. 



R A B C D E 
M: ZG slvdHC 



R 



A B C D E 



M: ZG slvdHC 
ZG pub Int 



R A B C D E 

M:ZG slvdHC 

ZG pub Int 

no job, not math guy 

*r~ aTcTd e 

M: ZG slvdHC 
ZG pub Int 
no job, not math guy 
R: ZG, math = loners 



R 



A B C D E 



A past fact — this is likely back- 
ground. Still, F 11 jot down a note. 



Sounds like more background. 



Here's the first boldface. He's not 
\ a mathematician; that's surpris- 
| ing. Still, I don't know what the 
j conclusion is, so I don't know 

what role this sentence ^ playing. 



j M: ZG slvdHC 
ZG pub Int 
no job, not math guy 
R: ZG, math = loners 
!) BUT math = commun, 
collab 

R A B C D E 

M: ZG slvdHC 
ZG pub Int 
(^)no job, not math guy 
R: ZG, math = loners 
©BUT math = commun, collab 



So the first boldface is "evi- 
dence" of "the popular view" that 
mathematicians are loners. . . but 
the sentence also uses the word 
"unfortunately" so it sounds like 
the author doesn't agree. . . 



/ was right; the author disagrees. 
The author's conclusion is this 
second boldface statement, so I can 
label it with a ©. 



Now, what about that first 
boldface statement? It's not the 
conclusion, and it doesn't support 
the conclusion, so it must be an X: 
\ Something Else. 



MANHATTAN 
GMAT 



Structure-Based Family 



Chapter 3 



Step 3: State the Goal 



The first boldface statement is an X; that is, it is neither the conclusion nor a premise. In this case, it sup- 
ports the alternate point of view, so we can call it a counterpremise. It goes against the conclusion. The second 
boldface statement is a C; it is the authors conclusion. 

Whatever answer I find should describe the first statement as something consistent with an X label and should 
describe the second statement as something consistent with a C label. Tm looking for an XC combo, and those 
two labels are on "opposite sides. " 

Step 4: Work from wrong to right. 




(A) The first is an obser- 
vation the author makes 
to illustrate a social 
pattern; the second is a 
generalization of that 
pattern. 



(B) The first is evidence 
in favor of the popular 
view expressed in the 
argument; the second 
is a brief restatement of 
that view. 



R A B C D E 

M: ZG slvd HC 

ZG pub Int 
)no job, not math guy 

R: ZG, math = loners 
)BUT math = commun, 

collab 



R 



A © C D E 



M: ZG slvdHC 

ZG pub Int 
)no job, not math guy 

R: ZG, math = loners 
)BUT math = commun, 

collab 



(C) The first is a specific R A8€ D E 



example of a general- 
ization that the author 
contradicts; the second 
is a reiteration of that 
generalization. 



M: ZG slvd HC 

ZG pub Int 
@no job, not math guy 

R: ZG, math = loners 
©BUT math = commun, 

collab 



Hmm. I'm not 100% sure what they mean 
by "illustrate a social pattern, " but I can tell 
that the description of the two statements here 
makes them sound like they're on the same 
"side" — the first illustrates something, and 
the second generalizes that same thing. I want 
an "opposite sides" answer. 

; The first supports a popular view... okay, 
maybe. You could call the press view the 
popular view. Oh, but the second doesnt 
restate that view; the second goes against that 
view. These two are on the same side again, 
and I want an "opposite sides" answer. 



"The first is a <something> that the author 
contradicts. " The <something> part confuses 
me, but I agree that the author contradicts 
the first one; this is a good description of 
this " label X" statement. Hmm. The second 
repeats "that generalization" — the same one 
mentioned for the first statement? No, Tm 
looking for opposite sides. 



GMAT 



59 



Structure-Based Family 



(D) The first is a spe- 
cific counterexample 
to a generalization that 
the author asserts; the 
second is that general- 
ization. 



R 



AB6DE 



(E) The first is a judg- 
ment that counters 
the primary assertion 
expressed in the argu- 
ment; the second is a 
circumstance on which 
that judgment is based. 



M: ZG slvdHC 

ZG pub Int 
(^)no job, not math guy 

R: ZG, math = loners 
©BUT math = commun, 

collab 

R AB€®E 

M: ZG slvd HC 

ZG pub Int 
(^)no job, not math guy 

R: ZG, math = loners 
©BUT math = commun, 

collab 



The first is a counterexample to something 
the author says? Yes, that accurately describes 
a " label X." The second is "that generaliza- 
tion?" I crossed off the last one for this same 
language. But wait. . . which generalization is 
this referring to this time? Oh, a generaliza- 
tion that the author asserts: that's the conclu- 
sion, which is a "label C." Leave this answer 



tn. 



| "Counters" language — yes, the first statement 
does counter the conclusion, which is consis- 
tent with the label X. "That judgment" = the 
first boldface. The second is not something on 
which the first one is based — that would be 
same side, and I want opposite sides. 



Takeaways for Describe the Role Questions 

We recognize this question type by the boldface font in the argument and the use of the word "bold- 
face" in the question stem. The question stem will also typically use the word "role" or a synonym. We 
will usually have two boldface statements, but sometimes there will be only one. 

Our goal is to identify the specific role, or building block category, of each boldface statement. Our 
primary method involves splitting the building blocks into three categories: 

C: The conclusion 

P: A premise supporting the conclusion 
X: Something other than C or P 

If needed, we can also try a secondary method that will allow us to make an educated guess if were 
short on time or get stuck: 

C: The conclusion 
F: A fact 

O: An opinion that is not the conclusion 



The most tempting trap answers will be "off" by just one or two words, often at the end of the sentence 
or phrase. We have to read very carefully all the way to the end in order not to fall for this trap. 



Structure-Based Family Chapter 3 

Describe the Argument 

Describe the Argument questions can be similar to Role questions in that Describe the Argument ques- 
tions usually also offer "abstract" answer choices that explicitly discuss the structure of the argument, 
including referring to the various building blocks (conclusions, premises, and so on). The majority of 
these Argument questions will offer two competing points of view and ask us, for example, how one 
person responds to the argument made by the other person. 

Important note: other question types can also be presented in this "two people speaking" format — the 
mere existence of two speakers does not make the problem a Describe the Argument problem. Its always 
necessary to identify the question type based upon the question stem. 

A minority of these questions will instead offer just one point of view and ask us how the author of that 
argument develops his or her point of view. 

Common question formulations include: 

Bill responds to Sally s argument by 

Bill challenges Sally's argument by 

The author develops the argument by doing which of the following? 
These all indicate that we have a Develop the Argument question. 

Our task here is to determine the manner in which a particular part of the text was constructed. If the 
second person disagrees with the first person, for example, we may be asked to explain how the second 
person disagrees, and possible answers might involve providing alternate evidence that contradicts the 
first person's claim, demonstrating that some evidence used by the first person is invalid or flawed (or 
simply questioning the accuracy of that evidence), introducing a new piece of information that the first 
person failed to consider, and so on. 

If the argument is presented in two parts, with one person presenting an argument and the second 
replying, then the first person's text is a complete argument that we need to read and diagram just as we 
do any argument. It's critically important to label each piece of information in the first speaker's argu- 
ment. Next, we examine the response and figure out which piece of the argument the response attacks. 
Ultimately, the attack is designed to find fault with the conclusion, but don't assume that the second 
person is attacking the conclusion directly. Tearing down any piece of the argument would ultimately 
undermine the conclusion, so find the piece that the second person most directly attacks. 



MANHATTAN 
GMAT 



Chapter 3 



Structure-Based Family 



Here's an example: 



Bill: I need to learn the names of 100 muscles for the anatomy exam in two hours. 
I've just memorized 5 of them in 5 minutes, so I only need 95 more minutes to 
study. Therefore, 111 have plenty of time to memorize everything and get a perfect 
score on the test. 




Sally: Are you sure? Perhaps the more you memorize, the harder it gets. 



Sally responds to Bill by 



Were not going to look at answer choices yet for this one. What is Bill s argument? What is his conclu- 
sion, and how does he support it? 

must learn 100 names in 2h 

mem 5 in 5m, so need 95m 

[have >95m] 

© will get 100% 

Which part does Sally attack? Does she attack the conclusion directly? No, but her words certainly cast 
doubt on Bill's eventual conclusion. She attacks Bills assumption that he can maintain the same rate of 
learning, 1 name every minute, for all 100 words. He doesn't explicitly state that he can maintain that 
rate, but he clearly believes it to be true in order to say that he needs only 95 more minutes. The correct 
answer might read something like: 

Sally calls into question an assumption Bill makes about the efficacy of his plan. 

This answer addresses the appropriate part of the argument — an assumption that Bill makes about his 
plan. An incorrect answer might look something like: 

Sally introduces new evidence that contradicts one of Bill's premises. 

Sally does say something new, but does it rise to the level of evidence? She only suggests that his memo- 
rization rate might not be constant; she doesn't prove that it is not. While we might be able to argue 
that the word "evidence" is okay, the word "contradicts" clearly takes things too far. Sally does not de- 
finitively contradict Bill's premise that he will need only 95 more minutes; rather, she raises a question 
as to whether he really can memorize the words in only 95 minutes. 

We probably won't be able to anticipate the exact abstract language of the correct answer, but if we can 
identify the part of the argument addressed, then we are in a much better position to find the appropri- 
ate "matching" language in the correct answer. 



62 



MANHATTAN 
GMAT 



Structure-Based Family 



Chapter 3 



Common Trap Answers 

The most tempting trap answers on Describe the Argument questions will be similar to those on Role 
questions: most of the answer is fine, but one or two words will throw the answer "off." 

In addition, because most of these arguments will consist of a second person objecting to something the 
first person says, it will always be tempting to choose an answer that indicates that Sally rejects Bills 



conclusion. It is the case, ultimately, that Sally s comment is going to weaken Bill s conclusion somehow, 
but she may not directly attack the conclusion — and the question asks us to articulate what she attacks 
directly. 




Let s try a full example: 

Mayor: The recycling program costs us nearly $1 million to operate every year, 
and our budget shortfall this year is projected to be $5 million. We need to cut the 
recycling program in order to help balance the budget. 

Consumer Advocate: It costs the city more to throw something out than to recycle 
it. 

The consumer advocate responds to the mayor by 

(A) establishing that the mayor's figures were incorrectly calculated 

(B) accepting the mayor's conclusion but questioning the legality of the plan 

(C) interpreting the mayor's evidence in a way that reduces the validity of the 
mayor's claim 

(D) introducing a new piece of information that calls into question the validity of 
the mayor's conclusion 

(E) pointing out that the mayor has not adequately considered the potential 
causes and effects of the budget shortfall 



MANHATTAN 
GMAT 



Chapter 3 



Structure-Based Family 



Step 1 : Identify the question. 




The consumer advocate re- 
sponds to the mayor by 



Step 2: Deconstruct the argument 




This is a Describe the Argument 
question. Two people are talking, 
and I have to explain how one 
responds to the other. 



Mayor: The recycling program 
costs us nearly $1 million to op- 
erate everv vear. and our budget 
shortfall this year is projected to 
be $5 million. 


DA A B C D E 

M: R cost $lm; this yr $5m 
short 


The mayor is stating a couple of 
facts — recycling costs $lm and 
they're voinv to miss their budget 
by $5m. 


We need to cut the recycling 
program in order to help bal- 
ance the budget. 


DA A B C D E 

M: R cost $lm; this yr $5m 
short -» cut R -> bal budg © 


So the mayor suggests that they 
should cut the R program in or- 
der to help balance the budget. 


Consumer Advocate: It costs the 
city more to throw something 
out than to recycle it. 


DA A B C D E 

M: R cost $lm; this yr $5m 
short -» cut R -* bal budg © 

CA: throw away costs > R 


That's interesting The CA 
says that it costs even more to 
throw something out. Why does 
this matter? If you cant recycle 
something, what are you going 
to do with it instead? Probably 
throw it out. 



Step 3: State the Goal 



For Describe the Argument questions I have to address how some part of the argument is made. In this case, 
I have to describe how the CA responds to the M. First, it sounds like the CA thinks that the Ms plan isn't 
going to work. The CA doesn't say so directly, but does say that throwing stuff out is more costly than recycling 
it. If that's true, then the plan to cut the recycling program just got a bit worse — it might not actually achieve 
the ultimate goal, which is to save money (to help balance the budget). 

The answer I find should indicate that the CA disagrees with the M, and specifically the CA disagrees as to 
whether the suggested action (cutting the R program) will result in the desired outcome (saving money, helping 
to balance the budget). 



MANHATTAN 
GMAT 



Structure-Based Family 



Chapters 



Step 4: Work from wrong to right 



(A) establishing that 
the mayors figures were 
incorrectly calculated 



(B) accepting the 
mayor s conclusion but 
questioning the legality 
of the plan 

(C) interpreting the 
mayor s evidence in a 

| way that reduces the 
i validity of the mayors 
claim 

(D) introducing a new 
piece of information 
that calls into ques- 

j tion the validity of the 
mayors conclusion 

(E) pointing out that 
the mayor has not 
adequately considered 
the potential causes and 
effects of the budget 
shortfall 



DA 



A B C D E 



M: R cost $lm; this yr 
$5m short -> cut R -» bal 
budg© 

CA: throw away costs > R 
DA AfiCDE 

M: R cost $lm; this yr 
$5m short -» cut R -> bal 
budg© 

CA: throw away costs > R 
DA AfiCDE 

M: R cost $lm; this yr 
$5m short -> cut R -» bal 
budg© 

CA: throw away costs > R 
DA A fi € D E 

M: R cost $lm; this yr 
$5m short -> cut R bal 
budg© 

CA: throw away costs > R 
DA Afi۩E 

M: R cost $lm; this yr 
$5m short -» cut R -» bal 
budg© 

CA: throw away costs > R 



The CA doesn't say anything about the 
mayors figures — in fact, the CA doesn't 
dispute the mayors evidence at all. Rather, 
the CA attacks the mayors assumption that 
cutting the program will lead to balancing 
the budget. 

The CA doesn't accept the conclusion, nor 
does the CA say anything about legality. 
Rather, the CA questions whether the plan 
will really lead to saving money. 



Hmm. Maybe. The CA does reduce the valid- 
ity of the mayors claim. Fm not 100% sure 
what "interpreting the evidence" means. Til 
leave this in for now. 



The CA does call the mayors conclusion into 
question, yes. Oh, I see — this one is better 
than answer C because the CA does introduce 
a new piece of info (that it costs more to throw 
something away). 

This one is tricky. It's true that the mayor 
hasn 't fully considered the potential effects of 
the plan to cut the recycling program — but 
that's not what this choice says. It talks about 
the causes and effects of the budget shortfall. 



til 



GMAT 



65 



Chapter 3 



Structure-Based Family 




Takeaways for Describe the Argument Questions 

We recognize this question type by the question stem (most commonly asking us how one person 
"responds" or "objects" to something that another person said), and by the "abstract" answer choices 
that address the role of the information (claim or conclusion, evidence or premise, and so on). We will 
be asked to address the role of a particular sentence or statement within the conversation (usually the 
respondent s statement, if there are two people talking). 

Our goal is to identify the specific role played by the statement about which were asked. Most of the 
time, that role will have something to do with calling into question a premise, assumption, or conclu- 
sion made by the first person. That can be done by directly attacking what the first person said, or by 
introducing new information that undermines the first person s argument. 



MANHATTAN 
GMAT 



Structure-Based Family 



Chapter3 



Problem Set 



1. Ad Revenues 

Media Critic: Network executives allege that television viewership is decreasing 
due to the availability of television programs on other platforms, such as the in- 
ternet and mobile devices. These executives claim that declining viewership will 
cause advertising revenue to fall and networks will thus be unable to spend 
the large sums necessary to produce high-quality programming. That devel- 
opment in turn, will lead to a dearth of programming for the very devices that 
cannibalized television's audience. However, research shows that users of alterna- 
tive platforms are exposed to new programs and, as a result, actually increase 
the number of hours per week that they watch television. This demonstrates 
that alternative platforms will not prevent networks from increasing advertising 
revenue. 




The portions in boldface play which of the following roles in the media critic's 
argument? 

(A) The first is an inevitable trend that weighs against the critic's claim; the sec- 
ond is that claim. 

(B) The first is a prediction that is challenged by the argument; the second is a 
finding upon which the argument depends. 

(C) The first clarifies the reasoning behind the critic's claim; the second demon- 
strates why that claim is flawed. 

(D) The first acknowledges a position that the network executives accept as 
true; the second is a consequence of that position. 

(E) The first opposes the critic's claim through an analogy; the second outlines a 
scenario in which that claim will not hold. 



MANHATTAN 
GMAT 



Chapters 



Structure-Based Family 




Renaissance Masters 

Many people praise High Renaissance painting for creating very realistic images 
from observation, but scholars have documented that some High Renaissance 
painters used pinhole cameras to project the likeness of their subjects onto 
the canvas and painted from there. Thus, people who credit High Renaissance 
painters with superior artistic skills are misguided. Painting from a projected 
image requires only an insignificant amount of additional skill beyond that 
needed to copy a picture outright. 

In the argument given, the two boldfaced portions play which of the following 
roles? 

(A) The first is a finding that has been used to support a conclusion that the 
argument rejects; the second is a claim that supports that conclusion. 

(B) The first is a finding that has been used to support a conclusion that the 
argument rejects; the second is that conclusion. 

(C) The first is a claim put forth to support a conclusion that the argument re- 
jects; the second is a consideration that is introduced to counter the force of 
that evidence. 

(D) The first is evidence that forms the basis for the position that the argument 
seeks to establish; the second is a claim presented to solidify that position. 

(E) The first is evidence that forms the basis for the position that the argument 
seeks to establish; the second is that position. 

Democracy 

As the United States demonstrated during its early development, it is not enough 
for citizens simply to have rights; the successful functioning of a democracy 
requires that they also know how to exercise those rights. Access to formal educa- 
tion was one necessary component that helped the U.S. citizenry to learn how to 
exercise its rights. Therefore, in order for a democracy to function successfully, its 
citizens must have access to a formal education. 

The author develops the argument by 

(A) using an analogy to establish a precedent for a planned future event 

(B) illustrating differences in the requirements for the functioning of a 
democracy depending upon the democracy in question 

(C) introducing an example that illustrates a common principle 

(D) forming a hypothesis that explains apparently contradictory pieces of 
evidence 

(E) supplying an alternate explanation for a known phenomenon 



68 




H 



ATTAN 



GMAT 



Structure-Based Family 



Chapter 3 



4. Malaria 

In an attempt to explain the cause of malaria, a deadly infectious disease, early 
European settlers in Hong Kong attributed the malady to poisonous gases sup- 
posedly emanating from low-lying swampland. In the 1880s, however, doctors 
determined that Anopheles mosquitoes were responsible for transmitting the 
disease to humans after observing that the female of the species can carry a 
parasitic protozoan that is passed on to unsuspecting humans when a mos- 
quito feasts on a person's blood. 

What function does the statement in boldface fulfill with respect to the argument 
presented above? 

(A) It provides support for the explanation of a particular phenomenon. 

(B) It presents evidence that contradicts an established fact. 

(C) It offers confirmation of a contested assumption. 

(D) It identifies the cause of an erroneous conclusion. 

(E) It proposes a new conclusion in place of an earlier conjecture. 

5. Digital Marketing 

Sania: The newest workers in the workforce are the most effective digital market- 
ing employees because they are more likely to use social networking websites 
and tools themselves. 

Carlos: But effective digital marketing also requires very technical expertise, such 
as search engine optimization, that is best learned on the job via prolonged expo- 
sure and instruction. 

Carlos responds to Sania by 

(A) demonstrating that Sania's conclusion is based upon evidence that is not 
relevant to the given situation 

(B) questioning the accuracy of the evidence presented by Sania in support of 
her conclusion 

(C) reinforcing Sania's argument by contributing an additional piece of evidence 
in support of her conclusion 

(D) pointing out differences in the qualifications desired by different employers 
seeking digital marketing employees 

(E) providing an additional piece of evidence that undermines a portion of 
Sania's claim 




MANHATTAN 
GMAT 



Chapter 3 Structure-Based Family 

6. Innovative Design 

Products with innovative and appealing designs relative to competing products 
can often command substantially higher prices in the marketplace. Because 
design innovations are quickly copied by other manufacturers, many con- 
sumer technology companies charge as much as possible for their new designs 
to extract as much value as possible from them. But large profits generated by 
the innovative designs give competitors stronger incentives to copy the designs. 
Therefore, the best strategy to maximize overall profit from an innovative 
new design is to charge less than the greatest possible price. 

In the argument above, the two portions in boldface play which of the following 
roles? 

(A) The first is an assumption that supports a described course of action; the 
second provides a consideration to support a preferred course of action. 

(B) The first is a consideration that helps explain the appeal of a certain strategy; 
the second presents an alternative strategy endorsed by the argument. 

(C) The first is a phenomenon that justifies a specific strategy; the second is that 
strategy. 

(D) The first is a consideration that demonstrates why a particular approach is 
flawed; the second describes a way to amend that approach. 

(E) The first is a factor used to rationalize a particular strategy; the second is a fac- 
tor against that strategy. 




MANHATTAN 
GMAT 



Structure-Based Family 



7. Gray Wolf Population 

Government representative: Between 1996 and 2005, the gray wolf popula- 
tion in Minnesota grew nearly 50 percent; the gray wolf population in Montana 
increased by only 13 percent during the same period. Clearly, the Minnesota gray 
wolf population is more likely to survive and thrive long term. 

Environmentalist: But the gray wolf population in Montana is nearly 8 times the 
population in Minnesota; above a certain critical breeding number, the popula- 
tion is stable and does not require growth in order to survive. 

The environmentalist challenges the government representative's argument by 
doing which of the following? 

(A) introducing additional evidence that undermines an assumption made by 
the representative 

(B) challenging the representative's definition of a critical breeding number 

(C) demonstrating that the critical breeding number of the two wolf popula- 
tions differs significantly 

(D) implying that the two populations of wolves could be combined in order to 
preserve the species 

(E) suggesting that the Montana wolf population grew at a faster rate than 
stated in the representative's argument 



Structure-Based Family 



Solutions 

1. Ad Revenues: The correct answer is B. 



Step 1: Identify the question. 

The portions in boldface play 
which of the following roles in 
i the media critics argument? 

Step 2: Deconstruct the argument. 

Media Critic: Network ex- 
ecutives allege that television 
viewership is decreasing due 
to the availability of television 
programs on other platforms, 
such as the internet and mobile 
devices. 

These executives claim that de- 
clining viewership will cause 
advertising revenue to fall and 
networks will thus be unable 
to spend the large sums neces- 
sary to produce high-quality 
programming. 

That development, in turn, will 
lead to a dearth of programming 
for the very devices that canni- 
balized televisions audience. 



However, research shows that 
users of alternative platforms are 
exposed to new programs and, 
as a result, actually increase the 
number of hours per week that 
they watch television. 



R A B C D E 



R A B C D E 

MC: NE say TV I b/c use 
other Ps 



R A B C D E 

MC: NE say TV 4 b/c use 
other Ps 

NE: TV I -» ad I -» no $ 
for qual prog 

R A B C D E 

MC: NE say TV I b/c use 
other Ps 

NE: TV I -* ad I -» no $ 
for qual prog no prog for 
other Ps 

R A B C D E 

MC: NE say TV I b/c use 
other Ps 

NE: TV I -» ad I -» no $ 
for qual prog no prog for 
other Ps 



This is a Role question. The ques- 
tion contains the word "boldface," 
and I'm asked to find the "role" of 
each bold statement. 



The word "allege" tells me this is 
a claim. Also, the critic is talking 
about what other people claim, so 
Vm guessing the critic is going to 
contradict what they claim — so 
this is probably a counterpremise. 

More from the NEs. More claims 
about bad things happening. Is 
the last thing the NEs conclusion? 
This is the 1st boldface. If the MC 
does contradict the NEs later, 
then this first boldface will be 
labeled an X. 

Ah, I see. Ironic. The fact that 
people are watching on other plat- 
forms will eventually lead to not 
having enough programming for 
those other platforms. Conclusion 
of the NEs. 

Here's the contradiction! Til wait 
till I find the conclusion for sure, 
but the first boldface is probably 
anX. 



BUT alt P users watch 
MORE TV 



Chapter 3 



Structure-Based Family 



This demonstrates that alterna- 
tive platforms will not prevent 
networks from increasing adver- 
tising revenue. 



R 



A B C D E 




MC:NEsayTV4b/cuse 
other Ps 

©NE: TV 4- -* ad I -» no $ 
for qual prog -» no prog for 
other Ps 

©BUT alt P users watch 
MORE TV 

©ad rates 

I want this combo: X P 



Hmm. I didn't think of that. The 
recyclable materials still have to 
go somewhere. Okay, the MC is 
concluding the opposite: that ad 
rates will go up. And if that's my 
conclusion, then the first boldface 
is indeed an Xand the second one 
supports the conclusion, so it's a P. 



Step 3: State the Goal. 

The question asks me to find the role of two boldface statements. The MC's conclusion is in the last line, and 
the second boldface, right before it, supports that conclusion. The second boldface is a P. The first boldface is 
part of the NE's argument, which is the opposite of the MC's argument, so the first boldface is an X. I want to 
find the combo XP (in that order) in an answer choice. 



Step 4: Work from wrong to rig ht. 



(A) The first is an inevi- 
table trend that weighs 
against the critic's claim; 
the second is that claim. 



R 



A B C D E 



MC: NE say TV 4 b/c use 
other Ps 

©NE: TV I -> ad I -» no $ 
for qual prog -» no prog for 
other Ps 

©BUT altP users watch 
MORE TV 

©ad rates 

I want this combo: X P 



"Weighs against the MC's claim" — yes, 
that's consistent with an X label. The 
second is "that" claim, meaning the MC's 
claim. No. the second one is a P, not a C 

(Note: the word " inevitable" can also be 
considered incorrect. The trend described in 
the first statement says that ad rates will go 
down, but the MC provides a reason why 
ad rates wouldn't go down. . . so the trend 
j isn't necessarily inevitable.) 



74 



GMAT 



Structure-Based Family 



Chapter 3 



MC:NEsayTV4< b/cuse 
other Ps 



(B) The first is a predic- S R ABCDE 
tion that is challenged 
by the argument; the 
second is a finding upon 
which the argument ©NE: TV I - ad I -+ no $ 

depends. for <l ual P ro g ~* no P ro S for 

other Ps 

©BUT alt P users watch 
MORE TV 

©ad rates 

I want this combo: X P 
R A B € D E 



(C) The first clarifies 
the reasoning behind 
the critic s claim; the 
second demonstrates 
why that claim is 
flawed. 



(D) The first acknowl- 
edges a position that the 
network executives ac- 
cept as true; the second 
is a consequence of that 
position. 



MC: NE say TV I b/c use 
other Ps 

©NE: TV I -» ad I -> no $ 
for qual prog no prog for 
other Ps 

©BUT altP users watch 
MORE TV 

©ad rates 

I want this combo: X P 
R A B € B E 

j MC: NE say TV I b/c use 
other Ps 

©NE: TV I -> ad I -> no $ 
for qual prog -» no prog for 
other Ps 

j 

©BUT altP users watch 
| MORE TV 

I ©ad rates 

I 

I want this combo: X P 



That's true, the MC does challenge the first 
one. That's an X. And the second one is 
a P, so we can describe that as something 
upon which the MC's argument depends. 
Keep this one in. 




Clarifies the MCs claim? No. The first one 
is something the NE's claim. I dont even 
need to read the second half of the answer. 



j Yes, the NEs do accept the first boldface as 
true — it's their premise. And they're on 

j the opposite side of the MC, so something 
they think isanX. Okay, that's fine. "The 
second is a consequence of that position. " 
What position? Oh, they use "position" in 

j the first half of the sentence. . . the NE's 
position. The second isn't something about 
the NE's position. It goes against the NE's 
position. No. 



MANHATTAN 75 
GMAT 



Chapter 3 



Structure-Based Family 




(E) The first op- 
poses the critic s claim 
through an analogy; 
the second outlines a 
scenario in which that 
claim will not hold. 



R A(B)€ B E 

MC: NE say TV I b/c use 
other Ps 

©NE: TV I -> ad 1 -» no $ 
for qual prog -» no prog for 
other Ps 

(P)BUT altP users watch 
MORE TV 

©ad rates 

I want this combo: X P 



| The first one does oppose what the MC con- 
| eludes. I'm not quite sure whether it does 
| so "through an analogy" Let's look at the 
second half. A scenario in which theMC's 
claim wont hold — meaning something 
that's on the opposite side of what the MCs 
say. No! The second one outlines a scenario 
in which the NE's claim won't hold, not 
I the MCs claim. 



2. Renaissance Masters: The correct answer is D. 



Step 1: Identify the question. 

In the argument given, the two 
boldfaced portions play which 
of the following roles? 



Step 2: Deconstruct the argument. 

Many people praise High 
Renaissance painting for creat- 
ing very realistic images from 
observation, 

but scholars have documented 
that some High Renaissance 
painters used pinhole cameras 
to project the likeness of their 
subjects onto the canvas and 
painted from there. 
Thus, people who credit High 
Renaissance painters with supe- 
rior artistic skills are misguided. 



R 



A B C D E 



R A B C D E 

ppl like HR pntg b/c realstc 

R A B C D E 

ppl like HR pntg b/c realstc 

BUT some HR pntrs just 
project / trace 



R A B C D E 

ppl like HR pntg b/c realstc 

BUT some HR pntrs just 
project / trace 

ppl who like HR = 
misguided 



The word " boldfaced," along with 
the boldface font in the argument, 
indicates that this is a Role ques- 
tion. 



j The "many people" intro feels like 
there's a contrast coming. . . and 
there is! Okay, let's just get this 
j piece down first. 
| Okay, so people think the HR 
j painters can paint realistically just 
\ by observing, but actually some 
were just projecting the images 
onto a canvas and sort of tracing 
the image. 



The word "thus" might mean 
this is the conclusion. Hmm. The 

previous sentence only said that 
"some"HR painters did the trac- 

j ing thing, not all of them. But this 

j sentence seems to be condemning 

j all of them. 



76 



GMAT 



Structure-Based Family 



Chapters 



Painting from a projected image R A B C D E 



requires only an insignificant 
amount of additional skill 
beyond that needed to copy a 
picture outright. 



n/a 



ppl like HR pntg b/c realstc 

BUT some HR pntrs just 
project / trace 

©ppl who like HR = 
misguided 

project / trace = 4 skill 
R A B C D E 

ppl like HR pntg b/c realstc 

BUT some HR pntrs just 
project / trace 

©ppl who like HR = 
misguided 

project / trace = I skill 



Step 3: State the Goal. 

I need to identify the role of the two boldfaced statements as they relate to the conclusion — which was that 
people who think HR painters are really skilled are misguided. The first one is a fact, and the second one is an 
opinion. I'm going to try to fact I opinion strategy and see how that works. 



Okay, the last sentence was 
definitely the conclusion. This sen- 
tence is supporting the conclusion. 
If this is true, then yes, painters 
who use this technique arent that 
great. 



Vm not 100% sure how to label 
the first boldface, but I did notice 
that the first one was a fact and 
the second one was a claim. I 
could use that alternate conclusion 
I fact I opinion strategy. 



Step 4: Work from wrong to right. 



R A B C D E 
ppl like HR pntg b/c realstc 
BUT some HR pntrs just 



(A) The first is a find- 
ing that has been used 
to support a conclu- 
sion that the argument 
rejects; the second is a project / trace 

claim that supports that Q pp i w h G like HR = 

misguided 

project / trace = I skill 



A " finding" could be a fact, and a claim is 
an opinion, so this one is okay so far. 



conclusion. 




MANHATTAN 
GMAT 



er3 



Structure-Based Family 



(B) The first is a finding 
that has been used to 
support a conclusion 
that the argument re- 
jects; the second is that 
conclusion. 


R A8CDE 

ppl like HR pntg b/c realstc 

BUT some HR pntrs just 
project / trace 

©ppl who like HR = 
misguided 

project / trace = I skill 


A "finding" could be a fact, but the conclu- 
sion is the conclusion; it doesn't fit into the 
"opinion" category. (Recall that an opinion 
is any claim that is NOT the conclusion.) 
This one's wrong. 


(C) The first is a claim 
put forth to support 
a conclusion that the 
argument rejects; the 
second is a consideration 
that is introduced to 
counter the force of that 
evidence. 


R A B € D E 

ppl like HR pntg b/c realstc 

BUT some HR pntrs just 
project / trace 

©ppl who likeHRR- 
misguided 

project / trace = 1 skill 


A "claim" is not a fact. I can eliminate this 
one. 


(D) The first is evidence 
that forms the basis 
for the position that 
the argument seeks to 
establish; the second 
is a claim presented to 
solidify that position. 


R A B € D E 

ppl like HR pntg b/c realstc 

BUT some HR pntrs just 
project / trace 

©ppl who likeHRR = 
misguided 

project / trace = I skill 


"Evidence" can be a fact, and a claim is a 
claim. This one has to stay in, too. 


(E) The first is evidence 
that forms the basis for 
the position that the 
argument seeks to estab- 
lish; the second is that 
position. 


R A B € D E 

ppl like HR pntg b/c realstc 

BUT some HR pntrs just 
project / trace 

©ppl who like HR = 
misguided 

project / trace = I skill 


"Evidence" can be a fact, but the second 
boldface is a claim, while this choice says 
that the second boldface is the "position, " or 
conclusion. I can eliminate this one. 



MANHATTAN 

GMAT 



Structure-Based Family 



Chapt 



compare A and D 



R A fi ۩E 

ppl like h-R pntg b/c realstc 

BUT some h-R pntrs just 
project / trace 

@ppl who like h-R = 
misguided 

project / trace = I skill 



3. Democracy: The correct answer is C. 
Step 1: Identify the question. 

The author develops the argu- DA 
ment by 



A B C D E 



Step 2: Deconstruct the argument. 



As the United States demon- 
strated during its early devel- 
opment, it is not enough for 
citizens simply to have rights; 
the successful functioning of a 
democracy requires that they 
also know how to exercise those 
rights. 

Access to formal education was 
one necessary component that 
helped the US citizenry to learn 
how to exercise its rights. 



DA 



A B C D E 



Based on the fact I opinion guessing tech- 
nique, I cant get any further; I just have to 
guess between B andD. Using the regular 
technique, both boldfaces are Premises used 
to support the authors conclusion. Answer 
A says that the first boldface is used "to 
support a conclusion that the argument 
rejects. "Eliminate answer A. 



Ex: US Cit have rights and exer- 
cise rights success democ 



DA 



A B C D E 



Ex: formal edu -» US Cit have 
rights and exercise rights -» suc- 
cess democ 



The wording is similar to a 
Describe the Argument question, 
though it doesnt have the "two 
people talking" feature. This 
might be one of the rare variants 
that doesnt have two people talk- 
ing. A quick glance at the abstract 
wording of the answer choices 
confirms: this is a Describe Arg 
question. 



Okay, specific example of a prin- 
ciple: the US showed that citizens 
need to have rights AND know 
how to exercise those rights. 



More detail on the US example. 
Formal education was needed to 
know how to exercise those rights. 



MANHATTAN 
GMAT 



Chapters 



Structure-Based Family 




Therefore, in order for a democ- 
racy to function successfully, its 
citizens must have access to a 
formal education. 



DA A B C D E Conclusion. The author s just sort 

Ex: formal edu -> US Cit of putting together the two "end" 

i . r j . , I pieces of the argument here. 

nave rights and exercise rights r J * 

success democ 

©formal edu nec for success 
democ 



Step 3: State the Goal. 

The author concludes that formal education is necessary in general for a democracy to be successful. The evi- 
dence: it happened this way in one country (the US). 

Step 4: Work from wrong to rig ht. 



(A) using an analogy to 
establish a precedent for 
a planned future event 



(B) illustrating differ- 
ences in the require- 
ments for the function- 
ing of a democracy 
depending upon the 
democracy in question 

(C) introducing an 
example that illustrates 
a common principle 



(D) forming a hy- 
pothesis that explains 
apparently contradictory 
pieces of evidence 



DA A B C D E 

Ex: formal edu US Cit 
have rights and exercise 
rights -» success democ 

©formal edu nec for success 
democ 

DA ABC D E 

Ex: formal edu US Cit 
have rights and exercise 
rights success democ 

©formal edu nec for success 
democ 

DA AfiCDE 

Ex: formal edu ~* US Cit 
have rights and exercise 
rights -> success democ 

©formal edu nec for success 
democ 

DA A B CBE 

Ex: formal edu ~* US Cit 
have rights and exercise 
rights -» success democ 

©formal edu nec for success 
democ 



The argument used an example. Is that they 
same thing as an analogy? Maybe. Oh, but 
what's the "planned future event"? There isn't 
anything; rather, the author concluded with 
a general statement, not a discussion of an 
event. 

I can imagine that it would be true that 
there are different requirements for differ- 
ent governments. . . but that's not what this 
argument says. The author only mentions the 
US and then concludes something in general 
about that. 

This looks decent. The argument did in- 
troduce an example and then used that to 
conclude a general principle. 



It would be reasonable to describe the conclu- 
sion as a hypothesis. . . but there aren't any 
contradictory things in the argument. Rather, 
the example given does illustrate the conclu- 



sion. 



80 



GMAT 



Structure-Based Family 



(E) supplying an alter- 
nate explanation for a 
known phenomenon 



| DA A B©B E 

Ex: formal edu -* US Cit 
j have rights and exercise 
rights success democ 

©formal edu nec for success 
democ 



The author doesrit supply an "alternate' 
explanation; he isn't arguing against anyone. 
He just concludes something from the US 
example. 



4. Malaria: The correct answer is A. 



Step 1: Identify the question. 


What function does the state- 


R ABCDE 


ment in boldface fulfill with 




respect to the argument pre- 




sented above? 





Step 2: Deconstruct the argument. 



In an attempt to explain the cause of 
malaria, a deadly infectious disease, 
early European settlers in Hong Kong ; 
attributed the malady to poisonous 
gases supposedly emanating from low- 1 
lying swampland. 
In the 1880s, however, doctors de- 
termined that Anopheles mosquitoes 
were responsible for transmitting the j 
disease to humans after observing that j 
the female of the species can carry 
a parasitic protozoan that is passed 
on to unsuspecting humans when a 
mosquito feasts on a person's blood. 



R ABCDE 
ES: Pgas -* M 



R ABCDE 

ES: Pgas -» M 

1880: mosq -» M by 
blood 

I want this combo: P 



This is a Role question. The ques- 
tion contains the word "boldface," 
and I'm asked to find the "func- 
tion" of each bold statement. 



This is a fact. Likely either back- 
ground or premise. 



Okay, this is still a fact, but it's 
the conclusion of the story. They 
used to think it was one thing, 
and then they figured out it was 
really the mosquitoes. The bold- 
face language, in particular, is the 
evidence used to show that it was 
mosquitoes. That's a Premise. 



(Note: our "combo" in this case consists of only one label because we only have one boldface statement.) 
Step 3: State the Goal. 

The question specifically asks me what role this information plays: "the female carries a PP that is passed to 
humans when a mosquito <bites someone>. "Because of that, the scientists decided that the mosquitoes were 
transmitting the disease. That's the most like a P — a premise that supports some further conclusion. 



I need to find the abstract language that indicates some kind of premise or support. 



Chapter 3 



Structure-Based Family 



Step 4: Work from wrong to right. 




(A) It provides support 
for the explanation of a 
particular phenomenon. 



(B) It presents evidence 
that contradicts an 
established fact. 



(C) It offers confirma- 
tion of a contested 
assumption. 



(D) It identifies the 
cause of an erroneous 
conclusion. 



(E) It proposes a new 
conclusion in place of 
an earlier conjecture. 



R ABCDE 

ES: Pgas -» M 

1880: mosq -» M by blood 

I want this combo: P 
R ABCDE 

ES: Pgas M 

1880: mosq -> M by blood 

I want this combo: P 

R A66DE 
ES: Pgas -» M 
1880: mosq -» M by blood 
I want this combo: P 



R A8€BE 

ES: Pgas -* M 

1880: mosq -» M by blood 

I want this combo: P 
R <g)B € B E 

ES: Pgas -* M 

1880: mosq M by blood 

I want this combo: P 



"Support" — that's good — for a "phenom- 
enon" Okay, that's just fancy-speak for: 
provides support for something that hap- 
pened. That sounds okay. Leave it in. 



"Evidence" — that's also good. And that 
evidence does "contradict" what the earlier 
settlers thought! Oh, wait — was that an 
established fact? Let me look at the first sen- 
\ tence again. No, they thought that, but the 
\ argument doesn't say it was an "established 
fact. " Cross this one off 
"Confirmation" is also good. . . of a "con- 
\ tested assumption." I'm not quite sure what 
they're referring to when they say "assump- \ 
\ Hon, " but nothing was contested here. First, I 
I some people thought one thing, and later, 
\ new evidence led some doctors to conclude 
something else. No. 

No — the only thing we might be able to 
j describe as an erroneous conclusion is what 
\ the early settlers thought. But the bold stuff 

supports the doctors' conclusion. 



Oh, yes, a new conclusion. Yes, that's ex- 
actly what the argument says! Oh, wait — / I 
labeled the boldface stuff a P, not a C. Why 
was that? Oh, I see — tricky. The first half 
of the sentence, the non-bold part, is the 
new conclusion. The bold part is the evi- 
dence supporting that. This isn't it after all! 



82 



GMAT 



Structure-Based Family 



5. Digital Marketing: The correct answer is E. 
Step 1: Identify the question. 



Carlos responds to Sania by 



DA 



A B C D E 



Step 2: Deconstruct the argument. 

Sania: The newest workers in 
the workforce are the most 
effective digital marketing em- 
ployees because they are more 
likely to use social networking 
websites and tools themselves. 



DA 



A B C D E 



S: new empl use soc nw 
most eff dig mktg empl © 



The "two person" structure and 
the focus on how Carlos responds 
indicate that this is a Describe the 
Argument question. 



Sania claims that the workers who 
use certain online tools are also 
the most effective at digital mar- 
keting, and that those people are 
the newest workers. In order to 
conclude that they re the MOST 
effective, she must be assuming 
that this is the most important 
criterion by which to judge such 
employees. 

Carlos doesnt dispute Sania s 
evidence, but he brings up a new 
point: you also need these other 
skills to be a good digital market- 
er. . . and those skills are learned 
on the job over a long time ("pro- 
longed"). . . which hurts Sania's 
claim that the newest workers are 
the most effective. 



Step 3: State the Goal. 

/ need to articulate how Carlos responds to Sania. He doesnt say that she's wrong about the newest workers 
using social networking tools. Rather, he says that digital marketers also need this other skill that takes a long 
time to learn on the job. If that's the case, then this weakens Sania 's claim that the newest workers are the 
most effective. 



Carlos: But effective digital 
marketing also requires very 
technical expertise, such as 
search engine optimization, that 
is best learned on the job via 
prolonged exposure and instruc- 



DA 



A B C D E 

S: new empl use soc nw -> 
most eff dig mktg empl © 

C: SE opt learn on job over 
long time eff dig mktg 



tion. 



Chapter 3 



Structure-Based Family 



Step 4: Work from wrong to rig ht. 



(A) demonstrating that 
Sania's conclusion is 
based upon evidence 
that is not relevant to 
the given situation 

(B) questioning the ac- 
curacy of the evidence 
presented by Sania in 
support of her conclu- 
sion 

(C) reinforcing Sania's 
argument by contribut- 
ing an additional piece 
of evidence in support 
of her conclusion 

(D) pointing out differ- 
ences in the qualifica- 
tions desired by differ- 
ent employers seeking 
digital marketing 
employees 

(E) providing an addi- 
tional piece of evidence 
that undermines a por- 
tion of Sania's claim 



DA 



A B C D E 



S: new empl use soc nw ~> 
most eff dig mktg empl © 

C: SE opt learn on job over 
long time eff dig mktg 
DA AfiCDE 

S: new empl use soc nw -» 
most eff dig mktg empl © 

C: SE opt learn on job over 
long time eff dig mktg 
DA ABGDE 

S: new empl use soc nw 
most eff dig mktg empl© 

C: SE opt learn on job over 
long time ~* eff dig mktg 
DA A B € B E 

S: new empl use soc nw 
most eff dig mktg empl © 

C: SE opt learn on job over 
long time eff dig mktg 

DA A B € B(E) 

S: new empl use soc nw -» 
most eff dig mktg empl © 

C: SE opt learn on job over 
long time -» eff dig mktg 



6. Innovative Design: The correct answer is C. 
Step 1; Identify the question. 



In the argument above, the two 
portions in boldface play which 
of the following roles? 



R 



Carlos doesn't say anything negative about 
Sania's evidence; rather, he introduces new 
evidence that attacks Sania's assumption 
that her piece of evidence is the most impor- 
tant thing to consider. 

Carlos doesn't attack Sania's evidence; 
rather, he introduces new evidence that 
attacks Sania's assumption that her piece 
of evidence is the most important thing to 
consider. 

Carlos does contribute an additional piece 
of evidence, but his new evidence hurts 
Sania's argument. Carlos doesn't support 
Sania's conclusion. 



Carlos does point out a different way to 
assess the effectiveness of digital marketing 
employees, but he doesn't mention employ- 
ers at all or differences among different 
employers. 



Bingo. This is exactly what Carlos does — a 
new piece of information that hurts the 
"newest workers" portion of Sania's claim. 



A B C D E 



; This is a Role question. The ques- 
\ Hon contains the word "boldface," \ 
j and I'm asked to find the "role" of 
\ each bold statement. 



84 



GMAT 



Structure-Based Family 



Step 2: Deconstruct the argument. 



Products with innovative and 
appealing designs relative to 
competing products can often 
command substantially higher 
prices in the marketplace. 
Because design innovations 
are quickly copied by other 
manufacturers, many consum- 
er technology companies charge 
as much as possible for their 
new designs to extract as much 
value as possible from them. 
But large profits generated by 
the innovative designs give 
competitors stronger incentives 
to copy the designs. 



Therefore, the best strategy to 
maximize overall profit from 
an innovative new design is to 
charge less than the greatest 
possible price. 



R A B C D E 
ID -» tt $ 

R A B C D E 
ID ^ Tt $ 

others copy, so COs charge 
Tt $ 

R A B C D E 
ID -» TT $ 

others copy, so COs charge 
TT $ 

BUT TT prof -» faster copy 
R A B C D E 

ID -» TT $ 

others copy, so COs charge 
TT $ 

BUT TT prof -» faster copy 

)to max prof, charge < than 
max price 

I want this combo: P C 



Sort of between a fact and a 
claim. Probably a premise. 



Getting more towards claim-based 
material, but I'm not sure this is 
the conclusion. 



BUT signals a contrast. Oh, so 
there s actually a drawback to 
making a lot of money: competi- 
tors will copy even faster so I guess 
that could hurt market share. 
That's interesting. 

Here we go, the conclusion. The 
person s claiming that companies 
actually shouldn't charge the 
largest possible price and this will 
actually help maximize profits in 
the end. The second boldface is the 
conclusion; that gets a C. And the 
first is a Premise that supports this 
conclusion. 



Step 3: State the Goal. 

The question asks me to determine the role played by each of 2 boldface statements. I've decided the second 
one is the conclusion and the first is a premise supporting that conclusion, so I want to find an answer that 
gives this combo: P C (premise, then conclusion). 



Chapters 



Structure-Based Family 



Step 4: Work from wrong to right. 




(A) The first is an 
assumption that sup- 
ports a described course 
of action; the second 
provides a consideration 
to support a preferred 
course of action. 



(B) The first is a con- 
sideration that helps 
explain the appeal of a 
certain strategy; the sec- 
ond presents an alterna- 
tive strategy endorsed by 
the argument. 



(C) The first is a phe- 
nomenon that justifies 
a specific strategy; the 
second is that strategy. 



(D) The first is a consid- 
eration that demon- 
strates why a particular 
approach is flawed; the 
second describes a way 
to amend that approach. 



R A B C D E 
ID-> tt$ 

others copy, so COs charge 
tt $ 

BUT tt prof -» faster copy 

)to max prof, charge < than 
max price 

I want this combo: P C 
R A B C D E 

ID ^ tt $ 

others copy, so COs charge 
tt $ 

BUT tt prof -* faster copy 

)to max prof, charge < than 
max price 

I want this combo: P C 
R ABC D E 

ID tt $ 

others copy, so COs charge 
tt $ 

BUT tt prof -» faster copy 

)to max prof, charge < than 
max price 

I want this combo: P C 
R A B C B E 

ID tt $ 

others copy, so COs charge 
tt $ 

BUT tt prof -> faster copy 

)to max prof, charge < than 
max price 

I want this combo: P C 



Hmm, they call the first an assumption, not 
a premise, but I suppose that's okay; they 
do say it "supports" something. The second, 
though, is the actual conclusion — but this 
answer choice makes the second sound like 
another premise. Idont think so. 



The wording for the first statement is a 
little strange, but I suppose that could be 
considered a premise. Okay. The word- 
ing for the second definitely doesnt work, 
though — the second isn't discussing an 
"alternative" strategy compared to the first. 
The first one actually supports the second! 



Something that " justifies" something else. . . 
yes, this could be a way to describe some- 
thing that supports something else. That 
part's okay. And the second part "is that 
strategy" — yes, strategy could be a synonym 
for conclusion. Keep this one in. 



No, the first supports the conclusion — it 
doesnt illustrate a flaw. I don't even need 
to read the second half of this choice. 



86 



GMAT 



Structure-Based Family 



(E) The first is a factor 
used to rationalize a 
particular strategy; the 
second is a factor against j 
that strategy. j 



R AB©BE 
ID -» tt $ 

others copy, so COs charge 
tt $ 

BUT 1 1 prof -» faster copy 

©to max prof, charge < than 
max price 

I want this combo: P C 



Something used to "rationalize" a "strate- 
gy"? Yes, that could be describing a premise 
that supports a conclusion. Oh, but the 
second goes against the strategy? No! The 
second is actually the strategy, or conclu- 
sion. 



7. Gray Wolf Population: The correct answer is A. 
Step 1: Identify the question. 



The environmentalist challenges 
the government representatives 
argument by doing which of the 
following ? 



DA A B C D E 



Step 2: Deconstruct the argument. 

Government representative: Be- \ 
tween 1996 and 2005, the gray j 
wolf population in Minnesota 
grew nearly 50 percent; the gray l 
wolf population in Montana 
increased by only 13 percent 
during the same period. 
Clearly, the Minnesota gray 
wolf population is more likely to \ 
survive and thrive long term. 



DA 



A B C D E 



GR: 96-05, Min GW t 
50%, Mon GW t 13% 



DA 



A B C D E 



GR: 96-05, Min GW t 
! 50%, Mon GW t 13% 

©Min > likely to survive/thrive 



We have a 2~person-talking struc- 
ture, and we're asked how the 
second person responds; this is a 
Describe the Argument question. 



This is just a straight fact. The 
Minnesota wolf population grew a 
lot faster in that time period than 
the Montana wolf population. 



Conclusion! Claiming that Min- 
nesota wolves are more likely to 
survive and thrive. Certainly, the 
Minnesota wolf population grew 
more. . . but does that automati- 
cally mean they re more likely to 
survive and thrive? 



Structure-Based Family 



GR: 96-05, Min GW T 
50%, Mon GW t 13% 



DA A B C D E Ah, okay. The environmentalist 

is pointing out that they're not 

I necessarily the same thing. Once 

\ the population is large enough, 
©Min > likely to survive/thrive \ it > s a i ready stMej so isn > t 

E: BUT Mon 8x Min; when I necessarily critical to survival. 
t enough, already stable 



Environmentalist: But the gray 
wolf population in Montana is 
nearly 8 times the population 
in Minnesota; above a certain 
critical breeding number, the 
population is stable and does 
not require growth in order to 
survive. 

Step 3: State the Goal 



The GR concludes that the Min wolves are more likely to survive and thrive because the growth rate was a lot 
higher, but the E responded that the Mon population was already a lot larger, so growth might not have been 
necessary to keep the population thriving. The Mon population might already have been stable in the first 
place. 

I need to find something that explains this is a more abstract way: a new piece of evidence changes the way we 
think about the issue addressed in the conclusion (surviving and thriving). 



Step 4: Work from wrong to right. 



(A) introducing ad- 
ditional evidence that 
undermines an as- 
sumption made by the 
representative 



DA 



A B C D E 



(B) challenging the rep- 
resentative s definition 
of a critical breeding 
number 



GR: 96-05, Min GW T 
50%, Mon GW T 13% 

©Min > likely to survive/ 
thrive 

E: BUT Mon 8x Min; 
when t enough, already 
stable 

DA AfiCDE 

GR: 96-05, Min GW t 
50%, Mon GW T 13% 

©Min > likely to survive/ 
thrive 

E: BUT Mon 8x Min; 
when t enough, already 
stable 



This sounds pretty good. The Es statement is 
a new piece of evidence, and it does under- 
mine the Rs assumption that growth is a good 
indicator of likelihood to survive and thrive. 



The E does challenge the Rs assumption 
about what it takes to survive and thrive, but 
the E can't challenge the R on "critical breed- 
ing number, " because the R never mentions 
this concept. 



Structure-Based Family 



Chapter 3 



(C) demonstrating 
that the critical breed- 
ing number of the two 
wolf populations differs 
significantly 



DA 



A B € D E 



(D) implying that 
the two populations 
of wolves could be 
combined in order to 
preserve the species 



(E) suggesting that the 
Montana wolf popula- 
tion grew at a faster rate 
than stated in the repre- 
sentative s argument 



GR: 96-05, Min GW T 
50%, Mon GW t 13% 

©Min > likely to survive/ 
thrive 

E: BUT Mon 8x Min; 
when T enough, already 
stable 

DA A B € B E 

GR: 96-05, Min GW t 
50%, Mon GW t 13% 

])Min > likely to survive/ 
thrive 

E: BUT Mon 8x Min; 
when t enough, already 
stable 

DA ®B € & E 

GR: 96-05, Min GW t 
50%, Mon GW t 13% 

^)Min > likely to survive/ 
thrive 

E: BUT Mon 8x Min; 
when t enough, already 
stable 



The E does mention the concept of "critical 
breeding number" but establishes only that 
the number of wolves in each population 
differs significantly, not that the number of 
wolves needed to achieve the "critical breed- 
ing number' is different. 



This might be an interesting strategy, but the 
E doesnt actually mention it. 



This is tricky if we're not reading very care- 
fully. The E does introduce a new figure, 
but that figure has to do with the size of the 
two populations, not the rate of growth. The 
E does not dispute the R's figures for rate of 
growth. 




GMAT 



89 



Chapter/ 4 




Critical Reasoning 



Assumptions 



Assumption Family Questions 
Find the Assumption (FA) Questions 
Evaluate the Argument Questions 

Flaw Questions 



Assumptions 



We briefly introduced assumptions in chapter 2, but we haven't done much yet with this concept. As- 
sumptions are the key to the largest family of questions, the Assumption Family; all questions in this 
family require us to identify and conduct some reasoning using some assumption made by the author. 
(When we refer to the "author," were referring to a hypothetical person who is "speaking" the argument 
and believes the argument to be valid. Were not referring to the test writer.) 

An assumption is something that the author must believe to be true in order to draw a certain conclusion; 
however, the author does not state the assumption in the argument. The assumption itself might not nec- 
essarily be true in the real world; the only requirement is that the author has to believe its true in order 
to make his or her claim. 

For example, what is the author of the below argument assuming must be true? 

Amy got an A on the test. Therefore, Amy must have studied for a long time. 

therefore 

Amy got an A |. Amy must have studied 

on the test. I ^ for a long time. 



(premise) (conclusion) 

The author assumes that, in order to get an A on the test, it is absolutely required to study for a long 
time. Note that the author is not just assuming that studying for a long time is one way to get an A on 
the test. The author concludes that Amy must have studied for a long time, so that is the only way. 

The diagram above represents the core of the argument; we previously discussed the core in Chapter 1. 
The core consists of the conclusion and the main premise or premises that lead to that conclusion, as 
well as the assumption. 



93 



Chapter 4 



Assumptions 



Assumptions fill a gap in the argument; the gap is represented by the arrow in the diagram above. If we 
insert a correct assumption into the argument, it makes the argument stronger: 

Amy got an A on the test. Studying for a long time is the only way to get an A. Therefore, 
Amy must have studied for a long time. 



Amy got an A 
on the test. 

(premise) 



therefore 

Studying for a long 
time is the only way 
to get an A 



Amy must have studied 
for a long time. 

(conclusion) 



(assumption) 

Most GMAT arguments will contain multiple assumptions. Any one assumption will not automatically 
make the argument air-tight, but it will make the conclusion more likely to be true. Brainstorm some 
assumptions for this argument: 

Thomas's football team lost in the championship game last year. The same two 
teams are playing in the championship game again this year, and the players on 
Thomas's team have improved. Therefore, Thomas's team will win the champion- 
ship game this year. 

The author is making multiple assumptions here. Thomas's team has improved enough to be better 
than last year s winner. Last years winning team has not also improved enough to keep them ahead of 
Thomas's team. 



This brings us to a couple of important strategies for dealing with assumptions on the test. 



Do 




Don't 


do brainstorm assumptions you 
can think of relatively easily 


but 


don't spend more than about 20 
seconds brainstorming up front 


do look for your brainstormed 
assumptions in the answers 


but 


don't eliminate answers because 
they don't match any of your 
brainstormed assumptions 


do find something that the 
author must believe to be true in 
order to draw the conclusion 


but 


don't hold out for something that 
makes the conclusion "perfect" or 
definitely true 



MANHATTAN 
GMAT 



Assumptions 



Chapter 4 



Let s insert an assumption into our football argument and see how it works: 

Thomas's football team lost in the championship game last year. The same two 
teams are playing in the championship game again this year, and the players on 
Thomas's team have improved enough to be better than the players on the defend- 
ing champion team. Therefore, Thomas's team will win the championship game 
this year. 

That assumption does make the argument stronger, but we could easily argue that Thomas's team still 
might not win. There are too many other potential factors involved; the author is making many as- 
sumptions, not just one, in order to draw this conclusion. 

Let's try another. On your scrap paper, draw out the core for the following argument, then try to brain- 
storm some assumptions. 

Charles is a sculptor. Therefore, he does not work in a practical field. 

The first sentence offers a fact; the second offers a conclusion. Those are the two halves of our core, but 
what is the gap in between? 




Charles is a sculptor. 



He does not work 
in a practical field. 



therefore 

^> 

Sculpting is not practical. 
Sculpting is not just a hobby 

done in his spare time. 
He does not hold a different 
job in a practical field. 

The author is making a number of assumptions here; three are shown in the diagram above. Differ- 
ent people will brainstorm different assumptions; any are valid assumptions as long as the author must 
believe them to be true in order to draw that conclusion. 



GMAT 



95 



Chapter 4 Assumptions 
Drill: Brainstorm Assumptions 



i l llll I 



Draw out the "core," and brainstorm at least one assumption that must be true in order to draw each 
conclusion. 

1 . Chocolate is Prabha's favorite flavor of ice cream. Therefore, she also likes chocolate 
candy bars. 

2. The employees of Quick Corp's accounting department consistently show a sig- 
nificant jump in productivity in the two weeks before taking vacation. Clearly, the 
knowledge that they are about to go on vacation motivates the employees to maxi- 
mize their productivity. 

3. Mayor: The Acme Factory has developed a new manufacturing process that uses 
chemical Q, the residue of which is toxic to babies. In order to protect our children, 
we need to pass a law banning the use of this chemical. 



Answer Key for Drill: Brainstorm Assumptions 



The assumption is noted in italics below the arrow. You may brainstorm different assumptions from the 
one shown. Other assumptions are acceptable as long as they represent something that MUST be true 
in order to draw the given conclusion. 



The author assumes that Prabha will definitely like chocolate in at least one other form (candy bars). 
The author assumes that its not true that she likes chocolate only in the form of ice cream. 



therefore 



Choc fav flav 



ice cream 




also likes 
choc candy 



She likes choc in other 
forms than ice cream 



96 



MANHATTAN 
GMAT 



Assumptions 



Chapter 4 



2 wks b4 vaca: 
I prod 



emp choose to max 
prod b4 vaca 



2. 

therefore 

. ^> 

jump" is the same thing 
as "max prod" 
They didn't plan vacation to 
occur right after a big dead- 
line or other busy time. 

The author assumes a couple of things here. First, the premise mentions only a "significant jump" in 
productivity; it does not say that this productivity represents the employees' maximum productivity. So 
the author is assuming that this significant jump does, in fact, represent the maximum productivity. 

Second, the author concludes that employees decide to be more productive because they'll be taking 
vacation soon. Perhaps its the case, instead, that the employees choose to take vacation right after they 
know they'll be forced to work harder for some other reason. This author is assuming that this is NOT 
the case — that employees are NOT choosing to take vacation after what they know will be a busy time 
at work. 




3. 



Acme using Q, 
toxic baby 



therefore 



to protect kids, 
banQ 



If Acme uses Q, then kids will 
somehow come into contact 
with Q 

The author assumes that use of chemical Q in the production process will somehow eventually expose 
babies to the chemical residue. Maybe it's the case that the chemical is used only for something that 
never comes into contact with the final product and will never come into contact with kids. 



GMAT 



97 



Chapter 4 Assumptions 

Assumption Family Questions 

As we discussed earlier, there are 5 different types of assumption questions. We'll cover three, Find the 
Assumption, Evaluate, and Flaw, in this chapter. In the next chapter, we'll learn about Strengthen and 
Weaken questions. (Note: Find the Assumption Questions are one type of question in the overall As- 
sumption Family.) 

Each type of question has its own key characteristics and goals, but there are some commonalities across 
all five types. There will be a conclusion, so we need to make sure to find it. In addition, while we read, 
we also try to brainstorm any gaps, or assumptions, that we can in a short amount of time (but we don't 
take very much longer than we take to read the argument itself). 

Find the Assumption (FA) Questions 

Find the Assumption questions ask us to find an assumption that the author must believe to be true in 
order to draw the conclusion. The correct answer should make the argument stronger. In addition, if 
the correct answer were not necessarily true, that would significantly harm the argument. 

Our task is to figure out which answer choice represents something that must hold true according to 
the author. Note one especially tricky aspect of these problems: the assumption itself might be flawed. 
We might think, "Well, is that really true in the real world? I don't think that's necessarily true." Don't 
ask that question! The only issue is whether the author must believe it to be true in order to arrive at his 
or her conclusion. In our chocolate ice cream example from the last problem set, the author had to be- 
lieve that Prabha liked at least one other form of chocolate (other than ice cream), but that assumption 
could be false. Perhaps Prabha really does like chocolate only in the form of ice cream. 

Identifying the Question 

Most of the time, these questions are easy to identify because the question stem will use some form of 
the noun "assumption" or the verb "to assume." Occasionally, the question will be worded differently. 
It may ask for a new premise, or piece of information, that is "required" or a new premise that will help 
the conclusion to be "more properly drawn" (or similar language). Here are a couple of examples: 

Which of the following is an assumption on which the argument depends? 

The conclusion above would be more properly drawn if it were established that 

Let's look at a sample argument: 

When news periodicals begin forecasting a recession, people tend to spend less 
money on non-essential purchases. Therefore, the perceived threat of a future re- 
cession decreases the willingness of people to purchase products that they regard 
as optional or luxury goods. 




MANHATTAN 
GMAT 



Assumptions 



Chapter 4 



Which of the following is an assumption on which the argument depends? 
We'll look at the answers in a moment. Lets do our first couple of steps: 

Step 1 ; Identify the question, 

FA A B C D E 



Which of the following is an 
assumption on which the argu- 
ment depends? 



Step 2: Deconstruct the argument. 



When news periodicals begin 
forecasting a recession, people 
tend to spend less money on 
non-essential purchases. 
Therefore, the perceived threat 
of a future recession decreases 
the willingness of people to pur- 
chase products that they regard 
as optional or luxury goods. 



FA 



A B C D E 



news Ps fore rec -> ppl spend 
1$ 



FA 



A B C D E 



news Ps fore rec -» ppl spend 
1$ 



n/a 



@prcvd fut thrt ppl spend I 
$lux 

FA A B C D E 

news Ps fore rec -» ppl spend 
4 $ 

©prcvd fut thrt ppl spend I 
$lux 



The question stem uses the word 
"assumption" so it is the Assump- 
tion type. Write "FA" on the scrap 
paper and then the 5 answer 
choice letters. 



This sounds like a premise, though 
I suppose it could be a conclusion. 
The news Ps predict a recession, 
and then people spend less money. 
This is the conclusion. The 
premise just tells us what people 
do — spend less money. The con- 
clusion tries to claim why they do 
it — a perceived future threat. 

What is the author assuming? 
That people are actually read- 
ing or hearing about the news P 
forecasts. That the recession hasn't 
already started and that's why 
people are spending less money 
— maybe the news Ps are just 
slow in " forecasting" something 
that has already started. 



Did you come up with any other assumptions? The key is to get our brains thinking about these things, 
but there are almost always multiple possible assumptions; we may not be able to brainstorm the exact 
one that will show up in the answers. 




MANHATTAN 
GMAT 



99 



Chapter 4 



Assumptions 



Step 3; State the Goal. 

My core is: 



News Ps: 



rec! 



Prcvd threat spend $ I 



I spend & 4 




therefore 

Ppl reading I hearing info 

from News Ps 
Threat only prcvd today; 
hasn't actually started 

I'll look for the assumptions I brainstormed, but I'll also be flexible; I might not have thought of the assump- 
tion in the correct answer. On FA questions, traps often involve going too far out of scope (the answer is not 
tied to the conclusion), using reverse logic (the answer makes the argument weaker, not stronger), or making 
an irrelevant distinction or comparison. (Note: we'll discuss more about trap answers a bit later in the 
chapter.) 

Lets take a look at the full problem now. 

When news periodicals begin forecasting a recession, people tend to spend less 
money on non-essential purchases. Therefore, the perceived threat of a future re- 
cession decreases the willingness of people to purchase products that they regard 
as optional or luxury goods. 

Which of the following is an assumption on which the argument depends? 



(A People do not always agree as to which goods should be considered luxury 
goods. 

(B) People are more likely to have read a news periodical recently because more 
and more periodicals are being published. 

(C) Most people do not regularly read news periodicals. 

(D) The consumer perception of the threat of recession increases when news 
periodicals begin forecasting a recession. 

(E) At least some of the biggest-spending consumers prior to the recession were 
among those who curtailed their spending after the recession began. 



100 



MANHATTAN 

GMAT 



Assumptions 



Step 4: Work from wrong to right 



(A) People do not always 
agree as to which goods 
should be considered luxury 
goods. 



(B) People are more likely to 
have read a news periodical 
recently because more and 
more periodicals are being 
published. 



FA 



A B C D E 



news Ps fore rec ppl 
spend I $ 

)prcvd fut thrt -> ppl 
spend i> $ lux 
FA A B C D E 

news Ps fore rec ppl 
spend I $ 

)prcvd fut thrt -> ppl 
spend I $ lux 



(C) Most people do not regu- FA A B € D E 

larly read news periodicals. n c _^ , 

J r news rs tore rec -» ppl 

spend I $ 

©prcvd fut thrt ppl 
spend l $ lux 

(D) The consumer perception FA A B € D E 

of the threat of recession in- ^ c _^ i 

news Ps fore rec -> ppl 

creases when news periodicals S p enc l l $ 
begin forecasting a recession. 

©prcvd fut thrt ppl 
spend A $ lux 



FA 



A B €(D)E 



(E) At least some of the 
biggest-spending consumers 
prior to the recession were 
among those who curtailed 
their spending after the reces- ©P^vd fut thrt -» ppl 
sion began. s P end 1 $ lux 



news Ps fore rec ~> ppl 
spend I $ 



I can believe that this is true in the real 
world, hut this is irrelevant to the con- 
clusion. The argument is not based upon 
whether people agree as to how to classify 
certain goods. 

This sounds a little bit like one of my 
brainstormed assumptions — the argu- 
ment is assuming that people are actually 
reading those periodicals. I'm not so sure 
about the "more periodicals are be- 
ing published" part, though. You don t 
absolutely have to believe that in order 
to draw that conclusion. I'll keep it in 
for now, but maybe I'll find something 
better. 

This is also about reading the peri- 
odicals. . . but it's the opposite of what 
I want! The argument needs to assume 
that people DO read the Ps; if they don't, 
then how can they be influenced by what 
the Ps forecast! 

let's see. News Ps begin forecasting, then 
perception of threat increases, then people 
spend less $. If that's true, then it IS 
the case that the perception of the threat 
leads people to spend less $. This one is 
looking better than B. I can cross offB 
now. 

Hmm. This one sounds good, too. Maybe 
if the biggest spenders keep spending 
during the recession, then the overall 
amount of money being spent won't go 
down that much. . . although the argu- 
ment doesn't really seem to depend on 
how much it goes down. Oh, and this 
says "after the recession began" — that 
doesn't make sense with this conclusion. 
The conclusion is about a "perceived 
threat of a future recession. " Nice trap! 



Chapter 4 



Assumptions 



We had a couple of good brainstormed assumptions ahead of time, but it turns out that we didn't brain- 
storm the assumption contained in the correct answer, D. We did, however, see an "opposite" answer in 
choice C; it contained information that was the opposite of one of our brainstormed assumptions, and 
that made it easier for us to know that C was wrong. 

The Negation Technique 

On harder questions, we might find ourselves stuck between two answer choices. What can we do to 
distinguish between tempting wrong answers and the right answer? We can try the Negation technique. 




On Find the Assumption questions, the correct answer will be something that the author must believe 
to be true in order to draw his or her conclusion. Because that is the case, if we were to negate the cor- 
rect answer — say that it isn't true — then the authors argument should be harmed. Negating the cor- 
rect answer should actually weaken the author's conclusion. 



The Negation technique takes a bit of time to implement, so we don't want to use it on all five answer 
choices, but the extra time needed might be worth it when we're stuck between two answers. 

How do we do this? Let's try it out on the News Periodicals problem we just finished. Let's say that we 
narrowed our answers to B and D: 



(B) People are more likely to have read a news periodical recently because more 
and more periodicals are being published. 

(D) The consumer perception of the threat of recession increases when news 
periodicals begin forecasting a recession. 

Recall the argument itself: 

news Ps fore rec ppl spend l $ 

@ prcvd fut thrt -> ppl spend I $ lux 

That is, the author claims that the news Ps forecast a recession, which causes people to perceive a future 
threat, and so people choose to spend less money on luxury goods. 

What if answer choice B were NOT true? It would say something like: 

(B) People are NOT more likely to have read a news periodical recently 

(and) more periodicals are NOT being published. 

Does this tear down the author's conclusion? Not really. The number of periodicals being published is 
irrelevant to the argument. 

Try negating answer D: 

MANHATTAN 
GMAT 



Assumptions 



Chapter 4 



(D) The consumer perception of the threat of recession DOES NOT increase 
when news periodicals begin forecasting a recession. 

Hmm. If the news Ps forecast a recession, but the consumers don't actually perceive any threat of reces- 
sion. . . then how can the author claim that they change their buying behavior based on perceiving this 
threat? Negating this answer does weaken the author's conclusion, so this one is the right answer. 

Don't use this technique on every answer choice, or you'll be in danger of spending too much time on 
CR questions — but, when you're stuck, try to unstick yourself using the Negation technique. 

Common Trap Answers 

Let's take a look at some of the trap answers we saw in that last question and discuss the kinds of traps 
that they typically try to set for us on Assumption questions. 

On many FA questions, a trap answer will go too far out of scope: it won't actually address the conclu- 
sion. Answer A in the above problem is a good example. The issue is not whether different people would 
agree to classify the same item as a luxury good. Rather, the conclusion is about what causes someone to 
spend less money on anything that that individual believes to be a luxury good. 

Trap answers can also use reverse logic, which we see in answer choice C. Reverse logic does the op- 
posite of what we want to do: in this case, answer C actually makes the argument worse, when we're 
trying to articulate an assumption that would make the argument a bit stronger. 

Answer E is an example of another trap: making an irrelevant distinction or comparison. The argument 
does not hinge upon whether the highest-spending consumers do something different from the rest. 
Rather, all consumers are lumped together in the argument. 

Takeaways for Find the Assumption Questions 

Most of the time, an FA question stem will contain some form of the word "assumption." Occasion- 
ally, the question may ask for a new premise that is "required" to draw the conclusion or will help the 
conclusion to be "more properly drawn." 

Our goal is to find the core (conclusion plus major premises) and brainstorm assumptions while laying 
out the core. 

Correct answers will represent something that the author must believe to be true in order to draw his or 
her conclusion. 

If we get stuck between two answers, we can try the Negation technique: negate each answer and see 
whether doing so weakens the conclusion; the one that weakens the conclusion is the right answer. 




MANHATTAN 
GMAT 



Chapter 4 



Assumptions 



Trap answer types include: 

Out of Scope: goes beyond the scope of the argument, doesn't address or affect the 
conclusion 

Reverse Logic: does the opposite of what we want (on FA questions, a reverse logic trap 
would make the conclusion weaker, not stronger) 

Irrelevant Distinction or Comparison: makes a distinction or comparison that doesn't 
matter between two groups 

Evaluate the Argument Questions 

For Evaluate questions, our first step is still to find an assumption, but we have to do a little more work 
after finding the assumption. At heart, we are asked what additional information would help us to try 
to determine whether the assumption is valid or invalid. 

Most Evaluate question stems will contain one of the following: 

• some form of the word "evaluate" 




some form of the word "determine" 



• language asking what would be "useful to know (or establish)" or "important to know" 

For example, an Evaluate question stem might ask: 

"Which of the following must be studied in order to evaluate the argument?" 

"Which of the following would it be most useful to know in determining whether the 
mayor's plan is likely to be successful?" 

Occasionally, an Evaluate question will use different wording from the above, but the question will still 
get at the same overall idea — what is relevant to consider or important to research or understand in 
making some decision or evaluating some claim? 

The "Two Paths" Strategy 

Evaluate answers will often be in the form of a question or in the form "Whether <a certain thing is one 
way or the other>." For example, let's say we're given this argument: 

In order to increase its profits, MilICo plans to reduce costs by laying off any non- 
essential employees. 



MANHATTAN 
GMAT 



Assumptions 



Chapter 4 



Let's see. According to the argument: 

MillCo will lay off non-essential employees reduce costs -» increase profits 

Does that sound like a good plan? What is MillCo assuming in claiming that laying off non-essential 
employees will result in increased profits? 

Profits are a measure of revenues minus costs so, for one thing, MillCo is assuming that revenues wont 
drop a lot as a result of these layoffs. If revenues dropped as much as or more than the expected cost 
savings, then MillCo's profits wouldn't increase. 

An Evaluate question would ask us what would be most important to know in order to evaluate Mill- 
Co's plan. A correct answer might read: 

Whether revenues will be affected adversely enough to threaten MillCo's profit 
structure. 

There are two possible paths to examine, yes or no: 

Yes, MillCo's revenues will be affected adversely enough. In this case, MillCo's argu- 
ment is weakened — the plan to increase profits is less likely to work. 

No, MillCo's revenues won't be affected adversely enough. In this case, MillCo's argu- 
ment is strengthened — the plan to increase profits might work. 

This answer choice, then, is designed to test the assumption; it helps to determine whether the assump- 
tion is valid. The correct answer should be structured in such a way that there are at least two possible 
"paths" — one path will strengthen the argument, and the other will weaken it. 

The incorrect answers will also be presented in this "two paths" format, with one key distinction: the 
two paths won't lead in two different directions (strengthening and weakening the argument). What if 
we had this answer choice? 

Whether MillCo might reduce its costs by eliminating some health insurance benefits. 

Let's evaluate the two paths: 

Yes, MillCo can reduce costs by eliminating some health benefits. How will this affect 
the given plan to lay off employees? It doesn't — it isn't part of the plan at all. 

No, MillCo cannot reduce costs by eliminating some health benefits. Again, this 
doesn't affect the plan given in the argument. 

This incorrect answer choice is trying to distract us by offering a different way to increase profits. . . but 
we aren't asked to find alternate ways to increase profits. We're asked to evaluate whether the existing 




MANHATTAN 
GMAT 



Assumptions 



argument is valid. We don't know any more than we did about whether MillCos plan (reduce employees 
reduce costs -» increase profits) will work. 

On Evaluate questions, we're going to do what we do on all Assumption Family questions: 

(1) find the core (conclusion plus major premises), noting this on our scrap paper 

(2) brainstorm any assumptions we can 

Then were going to look for an answer that addresses one of our assumptions (if we've been able to 
brainstorm the right one!). The correct answer should offer at least two different "paths," one that 
would make the argument stronger and one that would make the argument weaker. 

Let's try a full example; set your timer for 2 minutes and pick an answer before you read the explanation! 

Editorial: In order to preserve the health of its local economy, Metropolis should 
not permit a CostMart warehouse department store to open within city limits. 
It has been demonstrated that when CostMart opens a warehouse department 
store within a city, the bankruptcy rate of local retailers increases in that city by 
twenty percent over the next several years. 

Which of the following questions would be most useful for evaluating the conclu- 
sion of the Editorial? 

(A) Does the bankruptcy rate of local retailers in a city generally stabilize several 
years after a CostMart warehouse department store opens? 

(B) Do most residents of Metropolis currently do almost all of their shopping at 
stores within the city limits of Metropolis? 

(C) Have other cities that have permitted CostMart warehouse department 
stores within city limits experienced any economic benefits as a result? 

(D) Is the bankruptcy rate for local retailers in Metropolis higher than in the aver- 
age city that has permitted a CostMart warehouse department store within 
city limits? 

(E) Does CostMart plan to hire employees exclusively from within Metropolis for 
the proposed warehouse department store? 

Step 1 : Identify the question. 

Which of the following ques- Ev A B C D E The language "most useful" and 

tions would be most useful for "evaluating tells me that this is 

evaluating the conclusion of the an Evaluate question. I'll write 

Editorial? down "Ev" to indicate that. 



Assumptions 



Chapter 4 



Step 2: Deconstruct the argument 



Editorial: In order to preserve 
the health of its local economy, 
Metropolis should not permit 
a CostMart warehouse depart- 
ment store to open within city 
limits. 



It has been demonstrated that 
when CostMart opens a ware- 
house department store within a 
city, the bankruptcy rate of local 
retailers increases in that city 
by twenty percent over the next 
several years. 



(brainstorm assumptions) 



Ev 



A B C D E 



)M ban CM in city -» help loc 
econ 



Ev A B C D E 

)M ban CM in city help loc 
econ 

if new CM store bnkrpt 
locals t 20% for years 



Ev A B C D E 
©M ban CM in city help loc 



econ 



if new CM store -> bnkrpt 
locals t 20% for years 

[any good results?] 



"In order to" means that some- 
thing is going to cause this. Okay, 
the author is saying that M 
shouldn't let CM into the city so 
that M can preserve the health of 
the local economy. That's causa- 
tion and kind of sounds like a 
conclusion. 

Okay, there's a bad economic 
outcome for local retailers when a 
new CM store opens. So certainly 
this is evidence that supports the 
authors claim that preventing 
CM from opening a store will 
preserve the local economy. This is 
a premise, so the previous sentence 
was the conclusion. 
Are there any good economic 
results when CM opens a store? 
Maybe there are some bad and 
good results. . . and maybe the 
good results could outweigh the 
bad. 



Step 3: State the Goal. 

When CM opens a new store, there's at least one bad economic outcome. The author concludes that prevent- 
ing CM from opening a new store will preserve a good local economy. 

I need to find an answer that will have two possible paths — one way will strengthen the author's claim, and 
the other way will weaken it. Possibly, the answer might have something to do with the assumption I brain- 
stormed — could there be both good and bad possible results from a new CM store? 



';Z r : : V'--- ■• - - 



GMAT 



107 



Chapter 4 



Assumptions 



Step 4: Work from wrong to right 




(A) Does the bankrupt- 
cy rate of local retailers 
in a city generally sta- 
bilize several years after 
a CostMart warehouse 
department store opens? 

(B) Do most residents of 
Metropolis currently do 
almost all of their shop- 
ping at stores within the 
city limits of Metropo- 
lis? 

(C) Have other cities 
that have permitted 
CostMart warehouse 
department stores 
within city limits expe- 
rienced any economic 
benefits as a result? 

(D) Is the bankruptcy 
rate for local retailers in 
Metropolis higher than 
in the average city that 
has permitted a Cost- 
Mart warehouse depart- 
ment store within city 
limits? 

(E) Does CostMart 
plan to hire employees 
exclusively from within 
Metropolis for the 
proposed warehouse 
department store? 



Ev 



A B C D E 



)M ban CM in city -> help 
loc econ 

if new CM store -> bnkrpt 
locals t 20% for years 

[any good results?] 
Ev A B C D E 

M ban CM in city help 
loc econ 

if new CM store bnkrpt 
locals t 20% for years 

[any good results?] 
Ev A B C D E 

©M ban CM in city -> help 
loc econ 

if new CM store bnkrpt 
locals t 20% for years 

[any good results?] 
Ev A B C B E 

©M ban CM in city -» help 
loc econ 

if new CM store bnkrpt 
locals T 20% for years 

[any good results?] 



If yes, then the bad result wouldn't continue 
to happen over time. . . but it would still hap- 
pen in the first place. If no, then the bad re- 
sult would keep happening over time. Either 
way, there is a bad result for at least a few 
years, so both "paths" strengthen the authors 
conclusion. 

If yes, then. . . I'm not sure what this has to 
do with the conclusion. If some stores go out 
of business, then people will have to switch 
stores? Okay, but that doesn't impact the city's 
overall economic situation — either there are 
local retailers or there's the CM store in the 
city (or both). 

If yes, then that would be a reason to let 
CM open a store (because economic benefits 
would help to "preserve the health of the local 
economy"); that weakens the author's argu- 
ment. If no, there are no benefits to a CM 
store, then this strengthens the author's claim. 
This one is looking pretty good. 

If yes, then. . . would that make local stores 
even more likely to go out of business? I'm not 
sure — I don't know why they're going out of 
business now. This doesn't seem to affect the 
conclusion one way or the other. 



Ev 



A B C © E 



©M ban CM in city -» help 
loc econ 

if new CM store bnkrpt 
locals t 20% for years 

[any good results?] 



This one could be good, too. If yes, then that 
would be an economic benefit — jobs are 
good! If no, then. . . hmm. . . it's not bad neces- j 
sarily but it's not good either. 



108 



GMAT 



Assumptions 



Chapter 4 



(compare C and E) Ev A6©B£ Wait. For E, if some stores are going out 

©M ban CM in city -> help of 1 ™™*' that ^ans people losing jobs. 

i There's only a benefit to the new CM store 

loc econ J J 

adding jobs if it adds even more jobs than are 
if new CM store -> bnkrpt i 0$t So t /, ere > s no definite benefit given in E, 
locals t 20% for years but there is in C Tricky> C it fc 

[any good results?] 

Common Trap Answers 

Answer choice E in the last question was a very tricky trap. How do they get us to pick wrong answers 
on Evaluate questions? 

Out of Scope: don't make additional assumptions! Answer E presented us with something that seemed 
like a benefit at first, until we realized that we had to make an additional assumption in order to know 
that we definitely had a benefit. We shouldn't have to make additional assumptions; the answer should 
work "as is." 




Irrelevant Distinction or Comparison: we saw this trap for the first time in our Find the Assumption 
example earlier in the chapter. In the above problem, Answer D does discuss something mentioned 
by the argument — bankruptcy — but tries to compare a no-CostMart-in-Metropolis scenario with a 
CostMart-in-other-cities scenario, both of which are not the scenario we want to discuss: the scenario in 
which CostMart does open up in Metropolis. 

Takeaways for Evaluate Questions 

As always, we use the question stem to identify the question type. On Evaluate questions, the question 
stem will likely contain some form of the word "evaluate," "determine," or "useful (or important) to 
know." 

Our goal is to find a "two-path" answer: an answer that can be interpreted in two ways, one of which 
will strengthen the conclusion and the other of which will weaken the conclusion. 

Trap answers will try to get us to make additional assumptions — these answers are actually out of 
scope — or to make an irrelevant distinction or comparison between two things that are not the focus 
of the argument's conclusion. 



MANHATTAN 
GMAT 



Chapter 4 



Assumptions 



■ 1 ! 

(A \ j 



Flaw Questions 



Flaw Questions are the least common of the five Assumption Family question types. The question 
stems will almost always contain some form of the word "flaw," but we have to be careful because 
Weaken the Conclusion questions can also contain the word "flaw" in the question stem. 

Weaken questions will also contain "if true" language; Flaw questions will not contain this language. 



Flaw 


Weaken 


Look for this first: 


contains the word "flaw" but NOT "if true" 
language 


contains the word "flaw" AND the words "if 
true" (or an equivalent synonym) 


If you re still not sure, try this: 


answer choices are a bit more abstract, similar to 
but not as abstract as Structure Family questions 


answer choices represent a new piece of informa- 
tion (see next chapter for more) 


Example 


Which of the following indicates a flaw in the 
reasoning above? 


Which of the following, if true, would indicate a 
flaw in the teacher's plan? 



On occasion, a Flaw question may not contain a synonym of the word "flaw," such as "vulnerable to 
criticism." 



As with the other Assumption Family questions, Flaw questions will contain a conclusion, and we will 
want to brainstorm some assumptions if we can. The correct answer will have something to do with an 
assumption, but rather than articulating the assumption (which would help to strengthen the argu- 
ment), we are looking for wording that indicates why it is flawed thinking to believe that this assump- 
tion is true. 



MANHATTAN 
GMAT 



Assumptions 



Chapter 4 



For example: 

Pierre was recovering from the flu when he visited Shelley last week, and now 
Shelley is showing signs of the flu. If Pierre had waited until he was no longer 
contagious, Shelley would not have become ill. 

The author is assuming that Pierre was the one to infect Shelley. The author is also assuming that there 
is no other way Shelley could have gotten sick. Perhaps it is flu season, and many people with whom 
Shelley comes in contact have the flu! 

The correct answer might be something like: 

The author fails to consider that there are alternate paths by which Shelley could have 
become infected. 



Contrast that language with the assumption itself: the author assumes that only Pierre could have in- 
fected Shelley. If that's true, then that piece of information strengthens the author s argument. When we 
take the same information, though, and flip it around into a flaw, we harm the author s argument: 



Pierre was recovering from the flu when he visited Shelley last week, and now Shelley 
is showing signs of the flu. If Pierre had waited until he was no longer contagious, 
Shelley would not have become ill. 


Assumption 


Flaw 


Only Pierre could have infected Shelley. 


The author fails to consider that there are alter- 
nate paths by which Shelley could have become 
infected. 


The argument is made stronger. 


The argument is made weaker. 



In sum, we can think of Flaw questions as the "reverse" of Assumption questions. The answer still 
hinges on an assumption, but the correct answer will word that assumption in a way that hurts the 
argument. 



In addition, the answer choice language may be a bit more abstract than the answer choices we see on 
other Assumption Family questions. Often, the answer choices will talk about what the author "fails to 
consider (or establish)," "does not specify (or identify)," or something along those lines. 




MANHATTAN 
GMAT 



Chapter 4 



Assumptions 



Lets take a look at a full example. 




Environmentalist: Bando Inc's manufacturing process releases pollution into the 
atmosphere. In order to convince the company to change processes, we will orga- 
nize a boycott of the product that represents its highest sales volume, light bulbs. 
Because Bando sells more light bulbs than any other product, a boycott of light 
bulbs will cause the most damage to the company's profits. 

The environmentalist's reasoning is flawed because it fails to 

(A) allow for the possibility that Bando may not want to change its manufactur- 
ing process 

(B) does not supply information about other possible ways for Bando to reduce 
pollution 

(C) consider that the relative sales volumes of a company's products are not 
necessarily proportional to profits 

(D) identify any alternative methods by which to convince Bando to change its 
manufacturing process 

(E) consider that a boycott may take too long to achieve its purpose 



Step 1: Identify the question. 

The environmentalists reason- 
ing is flawed because it fails to 



Step 2: Deconstruct the argument 



Environmentalist: Bando Incs 
manufacturing process releases 
pollution into the atmosphere. 
In order to convince the com- 
pany to change processes, we 
will organize a boycott of the 
product that represents its high- 
est sales volume, light bulbs. 



Fl A B C D E The word "flawed" tells me this is either 
a Flaw or Weaken question. "If true" does 
not appear, so this is a Flaw question. I'll 
write down "Fl" on my scrap paper. 



Fl A B C D E 
E: B MP -> atmo poll 

Fl A B C D E 

E: B MP atmo poll 

boyc LB (t sales) -> conv B 
A MP 



This is a fact (we'll assume the E 
is telling the truth). 

Okay, here's a plan, so its likely 
a conclusion. They think if they 
boycott something, B might 
change its MP. So they're going to 
boycott LBs because B sells more 
LBs than anything else. 



112 



GMAT 



Assumptions 



Chapter 4 



Because Bando sells more light 
bulbs than any other product, a 
boycott of light bulbs will cause 
the most damage to the com- 
pany's profits. 



Fl A B C D E 
E: B MP -» atmo poll 
©boyc LB (t sales) 



conv 



B 



A MP 



B sells t LBs -» boyc -» t 
damage to prof 



Another claim. Because they sell 
more LBs than anything else, 
the E figures that a boycott of 
LBs will do the most damage to 
profits. Profits? How profitable 
are the LBs? 

Okay, the conclusion was the 
previous sentence, because all of 
this is designed to convince B to 
change its MP. 



Step 3: State the Goal 

Long one. Okay, E doesn't like that B pollutes. B sells more LBs than any other product, so E wants to boycott 
the LBs because that will do the most damage to B's profits (according to E, anyway. . .), and then E hopes this 
will all cause B to change its MP. 

I need to find an answer that will articulate a flaw in that reasoning. I've already thought of one. The E is 
assuming that just because B sells more LBs than anything else, B is also earning the most profits from LB. 
But there's no evidence there to support that. Another might be that consumers might not actually agree to 
boycott B. 



Step 4: Work from wrong to right 

(A) allow for the possibility 
that Bando may not want 
to change its manufacturing 
process 



(B) does not supply informa- 
tion about other possible 
ways for Bando to reduce 
pollution 



Fl A B C D E 

E: B MP -» atmo poll 

)boyc LB (T sales) -> conv 
B A MP 

B sells t LBs -» boyc -* T 

damage to prof 

Fl ABCDE 

E: B MP -> atmo poll 

)boyc LB (T sales) conv 
B AMP 

B sells t LBs -» boyc -» t 
damage to prof 



If anything, you could argue that the E 
is assuming B will not want to change 
— that's why the E thinks he has to 
organize a boycott to change B's mind! 



In the real world, I agree that E should 
explore all possible ways. . . but the 
question asks me to find a flaw in this 
particular plan about the boycott. This 
doesn't apply to that plan. 



GMAT 



113 



Chapter 4 



Assumptions 




(C) consider that the relative 
sales volumes of a company's 
products are not necessarily 
proportional to profits 



(D) identify any alterna- 
tive methods by which to 
convince Bando to change its 
manufacturing process 



(E) consider that a boycott 
may take too long to achieve 
its purpose 



Fl AfiCDE 

E: B MP -> atmo poll 

)boyc LB (t sales) conv 
B AMP 

B sells t LBs -» boyc -* t 

damage to prof 

Fl AfiCBE 

E: B MP atmo poll 

)boyc LB (f sales) -» conv 
BAMP 

B sells t LBs -> boyc t 

damage to prof 

Fl AB©BE 

E: B MP -» atmo poll 

)boyc LB (t sales) conv 
BAMP 

B sells t LBs -» boyc -» T 
damage to prof 



This sounds kind of like what I said be- 
fore. It's a little abstract, so I'm not sure 
I fully understand all of it, but it does 
say that sales aren't necessarily propor- 
tional to profits, so I'll keep this one in. 



This is like answer B. It'd be good in 
general for E to do this. . . but this doesn't 
help us figure out a flaw in the boycott 
plan specifically. 



I think what really matters is whether 
the plan is going to work at all, not how 
long it takes. The argument doesn't have 
any requirements about how long it will 
take to get B to change its process. 



Common Trap Answers 

The most common trap on Flaw questions involves making an irrelevant distinction or comparison: 

• discussing alternate plans or paths when we were asked to comment on the given plan 
(similar to answers B and D in the above example) 

• brings up a detail or distinction that does not actually affect the conclusion; similar to 
choice E in the above problem 

Flaw questions may also occasionally use Reverse Logic, similar to answer choice A in the above ex- 
ample. 



MANHATTAN 
GMAT 



Assumptions 



Chapter 4 



Takeaways for Flaw Questions 



We recognize Flaw Questions by use of the word "flaw" and the absence ofzny "if true" language. On 
occasion, the word "flaw" may be replaced by a synonym, such as "vulnerable to criticism." 

On Flaw Questions, we want to find the conclusion and quickly brainstorm any assumptions we can. 
The correct answer will be tied to an assumption, but it will be worded to highlight the flaw in assum- 
ing something to be true; it will hurt the argument. 

The most common trap answers typically involve making some kind of irrelevant distinction or com- 
parison. The answer might address something in a premise that doesn't affect the conclusion, or it may 
go down a different path entirely when we were asked to comment on a specific plan. 




MANHATTAN 

GMAT 



Assumptions 

Problem Set 



Chapter 4 



Answer each question using the 4-step CR process. Check your answer after each question. As you 
improve, consider timing yourself; critical reasoning questions need to be completed in an average of 2 
minutes. 



1. MTC & Asthma 



Methyltetrachloride (MTC) is a chemical found in some pesticides, glues, and seal- 
ants. Exposure to MTC can cause people to develop asthma. In order to halve the 
nation's asthma rate, the government plans to ban all products containing MTC. 

The government's plan to halve the nation's asthma rate relies on which of the fol- 
lowing assumptions? 

(A) Exposure to MTC is responsible for no less than half of the nation's asthma 
cases. 

(B) Products containing MTC are not necessary to the prosperity of the Ameri- 
can economy. 

(C) Asthma has reached epidemic proportions. 

(D) Exercise and proper nutrition are helpful in maintaining respiratory health. 

(E) Dust mites and pet dander can also cause asthma. 



Recently, the tuition at most elite private colleges has been rising faster than infla- 
tion. Even before these increases, many low and middle income families were un- 
able to afford the full tuition costs for their children at these institutions of higher 
learning. With the new tuition increases, these colleges will soon cater solely to 
students with affluent family backgrounds. 

Which of the following would it be most useful to determine in order to evaluate 
the argument? 

(A) Whether students from affluent families are more likely to prefer public or 
private colleges 

(B) Whether students from low and middle income families are qualified to at- 
tend elite private colleges 

(C) Whether low income families are less likely to be able to afford tuition costs 
than middle income families 

(D) Whether tuition costs at elite public colleges have also been rising faster 
than inflation 

(E) Whether grants or scholarships are earmarked for students from economi- 
cally disadvantaged families 




2. Tuition 




117 



GMAT 



Chapter 4 



Assumptions 



3. Charity 

Studies show that impoverished families give away a larger percentage of their 
income in charitable donations than do wealthy families. As a result, fundraising 
consultants recommend that charities direct their marketing efforts toward indi- 
viduals and families from lower socioeconomic classes in order to maximize the 
dollar value of incoming donations. 

Which of the following best explains why the consultants' reasoning is flawed? 

(A) Marketing efforts are only one way to solicit charitable donations. 

(B) Not all impoverished families donate to charity. 

(C) Some charitable marketing efforts are so expensive that the resulting dona- 
tions fail to cover the costs of the marketing campaign. 

(D) Percentage of income is not necessarily indicative of absolute dollar value. 

(E) People are more likely to donate to the same causes to which their friends 
donate. 

4. Oil and Ethanol 

Country N's oil production is not sufficient to meet its domestic demand. In order 
to sharply reduce its dependence on foreign sources of oil, Country N recently 
embarked on a program requiring all of its automobiles to run on ethanol in ad- 
dition to gasoline. Combined with its oil production, Country N produces enough 
ethanol from agricultural by-products to meet its current demand for energy. 

Which of the following must be assumed in order to conclude that Country N will 
succeed in its plan to reduce its dependence on foreign oil? 

(A) Electric power is not a superior alternative to ethanol in supplementing 
automobile gasoline consumption. 

(B) In Country N, domestic production of ethanol is increasing more quickly 
than domestic oil production. 

(C) Ethanol is suitable for the heating of homes and other applications aside 
from automobiles. 

(D) In Country N, gasoline consumption is not increasing at a substantially 
higher rate than domestic oil and ethanol production. 

(E) Ethanol is as efficient as gasoline in terms of mileage per gallon when used 
as fuel for automobiles. 




MANHATTAN 
GMAT 



Assumptions 



Chapter 4 



5. Exchange Student 

Student Advisor: One of our exchange students faced multiple arguments with 
her parents over the course of the past year. Not surprisingly, her grade point 
average (GPA) over the same period showed a steep decline. This is just one 
example of a general truth: problematic family relationships can cause significant 
academic difficulties for our students. 

The claim by the Student Advisor would be more properly drawn if which of the 
following were inserted into the argument as an additional premise? 

(A) Last year, the exchange student reduced the amount of time spent on aca- 
demic work, resulting in a lower GPA. 

(B) The decline in the GPA of the exchange student was not the reason for the 
student's arguments with her parents. 

(C) School GPA is an accurate measure of a student's intellectual ability. 

(D) If proper measures are not taken, the decline in the student's academic per- 
formance may become irreversible. 

(E) Fluctuations in academic performance are typical for many students. 

6. Food Allergies 

Food allergies account for more than thirty thousand emergency department 
visits each year. Often, victims of these episodes are completely unaware of their 
allergies until they experience a major reaction. Studies show that ninety percent 
of food allergy reactions are caused by only eight distinct foods. For this reason, 
individuals should sample a minuscule portion of each of these foods to deter- 
mine whether a particular food allergy is present. 

Which of the following must be studied in order to evaluate the recommendation 
made in the argument? 

(A) The percentage of allergy victims who were not aware of the allergy before a 
major episode 

(B) The percentage of the population that is at risk for allergic reactions 

(C) Whether some of the eight foods are common ingredients used in cooking 

(D) Whether an allergy to one type of food makes someone more likely to be 
allergic to other types of food 

(E) Whether ingesting a very small amount of an allergen is sufficient to pro- 
voke an allergic reaction in a susceptible individual 




MANHATTAN 
GMAT 



Chapter 4 



Assumptions 



7. News War 

For several years, Nighttime News attracted fewer viewers than World News, 
which broadcasts its show at the same time as Nighttime News. Recently, the pro- 
ducers of Nighttime News added personal interest stories and increased coverage 
of sports and weather. The two programs now have a roughly equal number of 
viewers. Clearly, the recent programming changes persuaded viewers to switch 
from World News to Nighttime News. 

The conclusion above is properly drawn if which of the following is assumed? 

Viewers are more interested in sports and weather than in personal interest 
stories. 

The programming content of Nighttime News is more closely aligned with 
the interests of the overall audience than is the content of World News. 
Some World News viewers liked the new Nighttime News programming bet- 
ter than they liked the World News programming. 
There are other possible causes for an increase in the number of viewers of 
Nighttime News, including a recent ad campaign that aired on many local 
affiliates. 

The quality of World News will remain constant even if Nighttime News 
improves. 

8. Five-Step Process 

Manager: the new manufacturing process should save us time overall, even 
though the first step of the five-step process will take twice as long as it does 
under the old process. Under the new process, far fewer of the components will 
be found defective, and the sole purpose of steps two and three under the old 
process is to weed out defective components. As a result, we should be able to 
eliminate two of the five steps in the existing manufacturing process. 

Which of the following would be most useful in evaluating the claim made in the 
argument? 

(A) Whether factory workers will require training in order to use the new manu- 
facturing process 

(B) Whether the new process is likely to introduce deficiencies or imperfections 
that must be corrected 

(C) Whether defective components can be fixed or must be thrown out 

(D) Whether a third manufacturing process would save even more time than 
both the old and new manufacturing processes 

(E) Whether saving time with the new manufacturing process will ultimately 
lead to cost savings for the company 




MANHATTAN 
GMAT 



Assumptions 



Chapter 4 



9. Genetics 

Two genes, BRCA1 and BRCA2, are linked to hereditary breast cancer. Therefore, in 
order to decrease the annual number of mammogram tests administered across 
a population and to more accurately assess a woman's individual risk of breast 
cancer, all women should be tested for these genes. 

Which of the following is an assumption on which the argument depends? 

(A) Some of the women who are tested for the two genes will subsequently 
undergo mammograms on a less frequent basis than they used to. 

(B) The majority of breast cancer patients have no family history of the disease. 

(C) Researchers may have identified a third breast cancer gene that is linked 
with hereditary breast cancer. 

(D) Women who have these genes have an 80 percent chance of getting breast 
cancer, while women who do not have these genes have only a 10 percent 
chance of getting breast cancer. 

(E) The presence of BRCA1 and BRCA2 can explain up to 50 percent of heredi- 
tary cases. 



MANHATTAN 

GMAT 



Assumptions 



Chapter 4 



Solutions 



A B C D E 



1. MTC & Asthma: The correct answer is A. 

Step 1: Identify the question. 

The government's plan to halve FA 
the nations asthma rate relies on 
which of the following assump- 
tions? 

Step 2: Deconstruct the argument. 



Methyltetrachloride (MTC) is a FA A B C D E 



chemical found in some pesti- 
cides, glues, and sealants. 
Exposure to MTC can cause 
people to develop asthma. 

In order to halve the nations 
asthma rate, the government 
plans to ban all products con- 
taining MTC. 



MTC chem 

FA A B C D E 

MTC chem -» asthma 
FA A B C D E 

MTC chem asthma 

©ban MTC -* Vi asthma rate 



Step 3: State the Goal. 



Asks for the "assumption"; this is i 
Find the Assumption question. 



This is just a fact — background 
or maybe a premise. 

Another fact but it's specifically a 
bad fact. This is likely a premise. 

Okay, the government has a plan 
to ban MTC , and the result will 
be (they claim) that the asthma 
rate will be cut in half There 
are no numbers or anything to 
support that. Are a lot of people 
exposed now? What percentage of 
those who develop asthma were 
exposed? Etc. 



The government claims that it can halve the asthma rate by banning MTC, but it gives absolutely no evi- 
dence or numbers to support halving the rate. 

I need to find an answer that supports the idea that they can halve the asthma rate — maybe that a very large 
percentage of people who develop asthma were exposed to MTC or something like that. 




MANHATTAN 
GMAT 



er 4 



Assumptions 



Step 4: Work from wrong to rig ht. 



(A) Exposure to MTC 
is responsible for no less 
than half of the nations 
asthma cases. 

(B) Products containing 
MTC are not necessary 
to the prosperity of the 
American economy. 

(C) Asthma has reached 
epidemic proportions. 



(D) Exercise and proper 
nutrition are helpful in 
maintaining respiratory 
health. 

(E) Dust mites and pet 
dander can also asthma. 



FA ABCDE 

MTC chem -» asthma 

©ban MTC Vi asthma 
rate 

FA ABCDE 

MTC chem -» asthma 

©ban MTC -» Vi asthma 
rate 

FA ABCDE 

MTC chem asthma 

©ban MTC -» Vi asthma 
rate 

FA ABCDE 

MTC chem asthma 

©ban MTC -» Vi asthma 
rate 

FA ®B € D E 

MTC chem asthma 

©ban MTC -» Vi asthma 
rate 



This sounds similar to what I said. Let's see. 
If MTC actually is responsible for at least half 
of asthma cases, then getting rid of it would 
get rid of all those cases as well. This one looks 
pretty good. 

Prosperity of the economy? They're just trying 
to distract me by making me think of a reason 
why we might want to use MTC. The conclu- 
sion is about halving the asthma rate, and 
this doesn't affect that conclusion. 
If asthma rates are really high, then that sup- 
ports the idea of wanting to lower them. But 
that's not what I'm trying to do — the author 
doesn't HAVE to believe this is true. Plus it 
says nothing about whether MTC is the cause. 
Distraction! Nothing about how or whether 
MTC causes asthma, or whether getting rid of 
I MTC will lower asthma rates. 



Distraction! Nothing about how or whether 
MTC causes asthma, or whether getting rid of 
MTC will lower asthma rates. 



2. Tuition: The correct answer is E. 
Step 1: Identify the question. 

Which of the following would it 
be most useful to determine in 
order to evaluate the argument? 

Step 2: Deconstruct the argument. 

Recently, the tuition at most 
elite private colleges has been 
rising faster than inflation. 



Ev 



ABCDE 



Ev ABCDE 
EPC t tuit>infl 



Contains the words "evaluate" 
and "useful to determine" — this 
is an Evaluate question. 



Fact: tuition at this specific type 
of school has been going up even 
faster than inflation. 



MANHATTAN 
GMAT 



Assumptions 



Chapter 4 



Even before these increases, 
many low and middle income 
families were unable to afford 
the full tuition costs for their 
children at these institutions of 
higher learning. 
With the new tuition increases, 
these colleges will soon cater 
solely to students with affluent 
family backgrounds. 



And people without much money 
already couldn't afford these 
schools, even before the tuition 
went up. Another fact. 



This must be the conclusion 
because the other two were facts. 
Basically, they're saying that only 
wealthy students are going to be 
able to afford these schools now. 



Ev A B C D E 

EPC t tuit > infl 

B4: I + mid inc fams cant 
afford 

Ev A B C D E 

EPC t tuit > infl 

B4: I + mid inc fams cant 
afford 

©EPC will have only rich 
students 



Step 3: State the Goal. 

This is an Evaluate question, so I need to find an answer that will help to determine whether or not the 
conclusion is likely to be valid. The correct answer will have "two paths": one path will make the conclusion a 
little more likely to be valid, and the other will make the conclusion a little less likely to be valid. 

The conclusion is that only wealthy students are going to be able to go to these EPCs. What is the author as- 
suming? Absolutely none of the low or middle income students can afford these schools. Non-wealthy students 
arent going to be taking out loans, or working their way through school, or finding some other way to cover 
the tuition costs. 



Step 4: Work from wrong to right. 



(A) Whether students 
from affluent families 
are more likely to prefer 
public or private colleges 



(B) Whether students 
from low and middle 
income families are 
qualified to attend elite 
private colleges 



Ev A B C D E 

EPC t tuit > infl 

B4: 4 + mid inc fams cant 
afford 

©EPC will have only rich 
students 

Ev AfiCDE 

EPC t tuit > infl 

B4: I + mid inc fams cant 
afford 

©EPC will have only rich 
students 



If affluent students prefer public colleges, that 
doesnt change the fact that the private colleges 
charge a lot of money and poorer students 
cant afford them. If affluent students prefer 
private colleges, that also doesnt change the 
same fact. 

If these students are not qualified to attend 
the EPCs, that doesnt change anything about 
the tuition issue. If these students are quali- 
fied, that also doesnt change the tuition issue 
(though it makes it seem unfair that the EPCs 
charge so much money!). 




GMAT 



125 



Assumptions 



(C) Whether low 
income families are 
less likely to be able to 
afford tuition costs than 
middle income families 



(D) Whether tuition 
costs at elite public col- 
leges have also been ris- 
ing faster than inflation 



(E) Whether grants or 
scholarships are ear- 
marked for students 
from economically 
disadvantaged families 



Ev A B € D E 

EPC t tuit > infl 

B4: I + mid inc fams cant 
afford 

©EPC will have only rich 
students 

Ev A B € © E 

EPC t tuit > infl 

B4: I + mid inc fams can't 
afford 

©EPC will have only rich 
students 

Ev A B € ©(E) 

EPC t tuit > infl 

B4: I + mid inc fams cant 
afford 

©EPC will have only rich 
students 



This answer makes a distinction between 
low and middle income families, but the 
argument doesn't distinguish between these 
| two groups — it combines them. Logically, 
it would make sense that the less money a 
family has, the less likely it could afford the 
tuition... but this doesnt change anything 
about the basic argument that low and mid- 
dle income families cant afford the tuition. 
If they have, then maybe that means lower- 
income students cant afford those schools 
either. . . but it might not mean anything, 
because perhaps the public schools have lower 
tuition fees in the first place. If rates have not 
been rising as fast at public colleges. . . that 
doesnt affect the argument's conclusion at all. 

If there are grants and scholarships for 
lower-income students, then perhaps they can 
afford to attend the EPCs — this hurts the 
argument's conclusion. If there are not grants 
and scholarships for these students, then the 
argument's conclusion is more likely to be 
true: these students won't be able to afford 
these colleges. The "two paths" on this answer 
do lead to strengthening the conclusion on one 
hand and weakening it on the other. 



3. Charity: The correct answer is D. 
Step 1: Identify the question. 



Which of the following best 
explains why the consultants' 
reasoning is flawed? 



A B C D E 



The word " flawed" indicates that 
this is either a Flaw or Weaken 
question. The lack of the words 
"if true" (or an equivalent) means 
that this is a Flaw question. 



MANHATTAN 
GMAT 



Assumptions 



Chapter 4 



Step 2: Deconstruct the argument. 



Studies show that impover- 
ished families give away a larger 
percentage of their income in 
charitable donations than do 
wealthy families. 



As a result, fundraising consul- 
tants recommend that charities 
direct their marketing efforts 
toward individuals and fami- 
lies from lower socioeconomic 
classes in order to maximize the 
dollar value of incoming dona- 
tions. 



F A B C D E 

poor donate > % inc than 
rich 



F A B C D E 

poor donate > % inc than 
rich 

©FC: to get most $, char shld 
focus on I inc ppl 



This is a fact. It's impressive that 
the poor donate anything, but if 
they do donate anything, then this 
fact makes sense because donating 
$100 is a much greater percentage 
of your income if you don't have 
much income. 

This is the conclusion. Based on 
the percentage info, the FCs are 
saying that the charities should 
focus on lower income people. . . 
but the FCs are assuming that 
"greater percentage" equals more 
money. A very rich person might 
donate $10 million, a small 
percentage of income but a very 
large sum. 



Step 3: State the Goal. 



For flaw questions, it's important to find the conclusion and brainstorm any assumptions, if I can. I need to 
find an answer that hurts the argument or shows why the argument is not a good argument. 

In this case, the FCs are recommending that the charities target lower income families in order to maximize 
the number of dollars they get in donations. Vve identified one potential assumption: the FCs assume that 
donating a greater percentage of income also means donating a greater dollar amount collectively. If that's not 
actually the case, then that's a flaw. 



Step 4: Work from wrong to right. 



(A) Marketing efforts 
are only one way to 
solicit charitable dona- 



A B C D E 



tions. 



(B) Not all impover- 
ished families donate to 
charity. 



poor donate > % inc than 
rich 

©FC: to get most $, char 
shld focus on I inc ppl 
F A B C D E 



This might be true, but it just indicates 
that there might be other ways, in addition 
to marketing efforts, to raise money. That 
doesn't affect the FCs recommendation to 
target lower income families in particular. 



poor donate > ( 
rich 



inc 



than 



©FC: to get most $, char 
shld focus on i> inc ppl 



I'm sure this is true, but how does it affect the 
conclusion? It doesn't. The argument never 
claims that ALL impoverished families donate 
to charity — only that, in general, they donate 
a larger percentage of income to charity. 




GMAT 



127 



Chapter 4 



Assumptions 




(C) Some charitable 
marketing efforts are 
so expensive that the 
resulting donations fail 
to cover the costs of the 
marketing campaign. 



(D) Percentage of in- 
come is not necessarily 
indicative of absolute 
dollar value. 



(E) People are more 
likely to donate to the 
same causes to which 
their friends donate. 



F A B € D E 

poor donate > % inc than 
rich 

)FC: to get most $, char 
shld focus on I inc ppl 



F A B € D E 

poor donate > % inc than 
rich 

)FC: to get most $, char 
shld focus on I inc ppl 



F A B €@fi 

poor donate > % inc than 
rich 

)FC: to get most $, char 
shld focus on i inc ppl 



Oh, maybe this is it. If you spend more on the 
marketing than you make from donations, 
that cant be a very successful marketing cam- 
paign. What was the conclusion again? Oh, 
wait, "to maximize the dollar value of dona- 
tions. " Whether ther marketing covered costs 
isn't part of the conclusion — it just depended 
on how much money they get in donations. 
Tricky, but not correct. 
This is what I was saying before about the 
really rich person donating $10 million! You 
can have a bunch of low income people give 
10% of their income and one billionaire give 
9% of her income. . . and the billionaire could 
be giving more in terms of absolute dollars. 
This indicates the flawed assumption made 
by the FCs. 

I can believe that this is true, but the argu- 
ment doesnt address which causes people 
choose for charity. Rather, the argument talks 
about amount of money donated. 



4. Oil and Ethanol: The correct answer is D. 



Step 1; Identify the question. 



Which of the following must be 
assumed in order to conclude 
that Country N will succeed in 
its plan to reduce its dependence 
on foreign oil? 

Step 2: Deconstruct the argument. 

Country N's oil production is 
not sufficient to meet its domes- 
tic demand. 



FA 



A B C D E 



FA A B C D E 
N oil prod * dem 



! Contains the word "as- 
sumed" — this is a Find the As- 
sumption question. 



They produce oil but cant make 
\ enough for their own needs. That 
\ must mean they have to import 
! some oil. 



MANHATTAN 
GMAT 



Assumptions 



Chapter 4 



In order to sharply reduce its 
dependence on foreign sources 
of oil, Country N recently em- 
barked on a program requiring 
all of its automobiles to run on 
ethanol in addition to gasoline. 
Combined with its oil produc- 
tion, Country N produces 
enough ethanol from agricul- 
tural by-products to meet its 
current demand for energy. 



FA A B C D E 

N oil prod ^ dem 

N req cars eth -» I for. oil 

FA A B C D E 
N oil prod * dem 
N req cars eth -» 1 for. oil 
N eth + oil = N s dem now 



They're requiring cars to use etha- 
nol, and they think that'll lead 
to having to use less foreign oil. It 
sounds like the cars can still use 
gas, though. . . 

Okay, so they do make enough 
ethanol PLUS oil combined to 
satisfy their own needs currently. 
The question is whether people are 
actually going to use ethanol for 
their cars or whether they'll want 
to keep using gasoline. And what 
if demand changes in future? 



Step 3: State the Goal. 

Country N thinks it can "sharply reduce" the amount of foreign oil it needs if it starts making people have 
cars that use ethanol. Will the plan really work that way? They're assuming people really will start to use the 
ethanol. They're also assuming they'll continue to produce enough oil and ethanol in the future. 

I need to find an answer that must be true in order to allow the author to draw the above conclusion. 

Step 4: Work from wrong to rig ht. 



(A) Electric power is not 
a superior alternative to 
ethanol in supplement- 
ing automobile gasoline 
consumption. 

(B) In Country N, 
domestic production 
of ethanol is increasing 
more quickly than do- 
mestic oil production. 



FA A B C D E 

N oil prod ^ dem 

N req cars eth -> I for. oil 

N eth + oil = N s dem 
now 

FA A B C D E 

N oil prod * dem 

N req cars eth -» I for. oil 

N eth + oil = N s dem 
now 



Electric power? That seems out of scope. We're 
supposed to find something that goes with the 
plan stated in the argument, and that plan 
mentions nothing about electric power. 



If this is true, then switching stuff to ethanol 
seems like a good call. Does it HAVE to be 
true in order to draw the conclusion? What 
if the two were increasing at the same rate? 
That would be fine, actually. This doesn't 
have to be true — so it isn't a necessary as- 
sumption. 




GMAT 



129 



Assumptions 



(C) Ethanol is suitable 
for the heating of homes 
and other applications 
aside from automobiles. 



(D) In Country N, 
gasoline consumption 
is not increasing at a 
substantially higher rate 
than domestic oil and 
ethanol production. 



FA A B € D E 

N oil prod ^ dem 

N req cars eth I for. oil 

N eth + oil = N s dem 
now 

FA A B € D E 

N oil prod ^ dem 

N req cars eth I for. oil 

N eth + oil = Ns dem 
now 



(E) Ethanol is as ef- 
ficient as gasoline in 
terms of mileage per 
gallon when used as fuel 
for automobiles. 



FA 



A fi €©£ 
N oil prod ^ dem 
N req cars eth i for. oil 
N eth + oil = Ns dem 



now 



5. Exchange Student: The correct answer is B. 
Step 1: Identify the question. 



The claim by the Student 
Advisor would be more properly 
drawn if which of the following 
were inserted into the argument 
as an additional premise? 



FA 



Step 2: Deconstruct the argument. 

Student Advisor: One of our ex- 
change students faced multiple 
arguments with her parents over 
the course of the past year. 



If this is true, then switching stuff to ethanol 
seems like a good call. Does it HAVE to be 
true in order to draw the conclusion? No. The 
argument only talks about a plan to have cars 
start using ethanol. 

Hmm. The argument is assuming in gen- 
eral that the ethanol + oil production can 
keep up with the country's demand. So, yes, 
the author would have to assume that gas 
consumption isn't increasing at a much faster 
rate than production. 

Let's try negating this one: If gas consump- 
tion WERE increasing at a much higher rate, 
what would happen? Oh, they might have to 
get more from foreign sources — bingo! Negat- 
ing this does weaken the conclusion. 
It would be good to know how efficient etha- 
nol is compared to gas. . . but does it HAVE 
to be true that they're equally efficient? No. 
Even if ethanol were less efficient, it's possible 
that the country could still produce enough to 
meet its needs. 



A B C D E 



FA A B C D E 
SA: st arg par lyr 



This is a tough one. They're asking me 
to find the answer that can be "inserted 
into the argument as a premise. "Do- 
ing so makes the claim "more properly 
drawn. " This is an example of unusual 
wording for a Find the Assumption 
question. 



This is a fact — background or a 
premise. 



Assumptions 



Chapter 4 



Not surprisingly, her grade point 
average (GPA) over the same 
period showed a steep decline. 



This is just one example of 
a general truth: problematic 
family relationships can cause 
significant academic difficulties 
for our students. 



FA A B C D E 
SA: st arg par lyr 
GPA 44 

FA A B C D E 
SA: st arg par lyr 
GPA 44 
©fam prob -> acad prob 



Not only did the student's GPA 
go down, but the SA says "not 
surprisingly. " Sounds like the SA 
is going to conclude something 
based on this. 

Here we go: the SA claims that 
this student's family problems 
caused the academic problems. 
Maybe there was a different cause. 



Step 3: State the Goal. 

/ need to find an answer that the author must believe to be true in order to draw this conclusion. The only 
thing I can think of right now is very general: if the SA is assuming the family problems were what caused the 
academic problems, then the SA is also assuming there wasn't something else causing the academic problems. 



Step 4: Work from wrong to right. 



(A) Last year, the ex- 
change student reduced 
the amount of time 
spent on academic work, 
resulting in a lower 
GPA. 

(B) The decline in the 
GPA of the exchange 
student was not the 
reason for the students 
arguments with her 
parents. 



FA A B C D E 
SA: st arg par lyr 
GPA 44 
©fam prob -» acad prob 

FA A B C D E 
SA: st arg par lyr 
GPA 44 
©fam prob acad prob 



(C) School GPA is an 
accurate measure of a 
student s intellectual 
ability. 



FA A B € D E 
SA: st arg par lyr 
GPA 44 
©fam prob ~* acad prob 



This would explain why her GPA went 
down, which means maybe it didrit actu- 
ally have to do with family problems. But 
I'm looking for something the author believes 
will HELP with the claim that it was family 
problems. This answer hurts that claim. 
Let's see. This is kind of what I said be- 
fore — there is NOT a different cause for the 
decline of her GPA. 

Let's try negating this. If the student's GPA 
went down first and then her parents got mad 
at her for that reason, then you can't claim 
that the family problems caused the lower 
GPA. The SA 's argument would fall apart. 
This one looks good. 

This doesn't matter. Either it's accurate or 
inaccurate. Regardless, it used to be higher 
and is now lower, and she and her parents 
have been arguing about something. Whether 
it's accurate doesn't come into consideration 
in the argument. 




MANHATTAN 
GMAT 



Assumptions 



(D) If proper measures 
are not taken, the 
decline in the student s 
academic performance 



FA A B € B E 
SA: st arg par lyr 
GPAU 



may become irreversible. ©f am pro b -> acad prob 
(E) Fluctuations in FA A(B)€ B £ 



academic performance 
are typical for many 
students. 



SA: st arg par lyr 
GPAU 
©fam prob -* acad prob 



/ could see how this might be true in general, 
but this doesnt support the idea that family 
problems can cause academic problems. If it 
doesnt support that idea, it cant possibly be 
the assumption. 

I could see how this might be true in general, 
but this doesnt support the idea that family 
problems can cause academic problems. If it 
doesnt support that idea, it cant possibly be 
the assumption. 



6. Food Allergies: The correct answer is E. 
Step 1: Identify the question. 



Which of the following must be 
studied in order to evaluate the 
recommendation made in the 
argument? 

Step 2: Deconstruct the argument. 

Food allergies account for more 
than thirty thousand emergency 
department visits each year. 
Often, victims of these episodes 
are completely unaware of their 
allergies until they experience a j 
major reaction. 

Studies show that ninety percent 
of food allergy reactions are 
caused by only eight distinct 
foods. 

For this reason, individuals 
should sample a minuscule por- 
tion of each of these foods to 
determine whether a particular 
food allergy is present. 



Ev 



A B C D E 



Ev A B C D E 
FA 30k ER/yr 

Ev A B C D E 

FA -> 30k ER/yr 

ppl don't know till have rx 

Ev A B C D E 

FA ^ 30k ER/yr 

ppl don't know till have rx 

only 8 foods -» 90% FA rx 
Ev A B C D E 

FA -> 30k ER/yr 

ppl don't know till have rx 

only 8 foods -* 90% FA rx 

)ppl shld try tiny bit of 8 
foods to see if FA 



The words "must be studied" and 
"evaluate" indicate that this is an 
Evaluate question. 



This is a fact. 



Fact but more fuzzy. A lot of 
people dont know they're allergic 
till they have a major reaction. 

More facts! That's interesting. 
Only 8 foods cause most allergic 
reactions. 



This is the conclusion. The au- 
thors saying we should all try a 
tiny bit of these 8 foods to see what 
happens. That assumes that we'll 
actually have a reaction from a 
tiny amount. It also assumes we 
wont die from just a tiny amount 
(if we are allergic). 



Assumptions 



Step 3: State the Goal 



This is an Evaluate question, so I need to find an answer that will help to determine whether or not the 
conclusion is likely to be valid. The correct answer will have "two paths": one path will make the conclusion a 
little more likely to be valid and the other will make the conclusion a little less likely to be valid. 

In this case, the author recommends that we all try tiny bits of these 8 foods to see whether we're allergic. The 
authors assuming that we can tell whether we're allergic from trying just a tiny bit — and also that we wont 
die by trying a small amount if we are allergic. 

Step 4: Work from wrong to rig ht. 



(A) The percentage of 
allergy victims who were 
not aware of the allergy 
before a major episode 



(B) The percentage of 
the population that is at 
risk for allergic reactions 



(C) Whether some of 
the eight foods are com- 
mon ingredients used in 
cooking 



(D) Whether an al- 
lergy to one type of food 
makes someone more 
likely to be allergic to 
other types of food 



Ev A B C D E 

FA ^ 30k ER/yr 

ppl don't know till have rx 

only 8 foods -» 90% FA rx 

©ppl shld try tiny bit of 8 foods 
to see if FA 

Ev A B C D E 

FA -> 30k ER/yr 

ppl don't know till have rx 

only 8 foods 90% FA rx 

©ppl shld try tiny bit of 8 foods 
to see if FA 

Ev A fi € D E 

FA -» 30k ER/yr 

ppl don't know till have rx 

only 8 foods -» 90% FA rx 

©ppl shld try tiny bit of 8 foods 
to see if FA 
Ev A B € B E 

FA 30k ER/yr 

ppl don't know till have rx 

only 8 foods -» 90% FA rx 

©ppl shld try tiny bit of 8 foods 
to see if FA 



The argument told us that victims "often" 
aren't aware of the allergy beforehand. 
If I knew that 90% weren't aware, that 
would go along with what the argument 
already says. If I knew that 50% weren't 
aware. . . hmm, that wouldn't change the 
argument. In general, knowing the exact 
percentage doesn't change anything. 
If a really high percentage is at risk for 
allergies, then it's probably important to 
figure out whether people are allergic. . . 
but that doesn't mean that the specific 
recommendation in the conclusion here is 
a good or bad one. Also, this answer choice 
doesn't specifically limit itself to food aller- 
gies; it mentions all allergies in general. 
If yes, then many people may have already 
tried small amounts of these foods. That 
doesn't actually tell us, though, whether 
the recommendation is a good one. If no, 
then it doesn't affect the conclusion at 
all — we still don't know whether it's a 
good recommendation. 

If yes or if no, you'd still want to test 
people to see whether they're allergic to 
anything. This choice doesn't have "two 
paths" that lead to alternate outcomes. 



Assumptions 



(E) Whether ingesting 
a very small amount of 
an allergen is sufficient 
to provoke an allergic 
reaction in a susceptible 
individual 



Ev A B € B(E) 
FA 30k ER/yr 
ppl don't know till have rx 
only 8 foods -» 90% FA rx 



This is one of the things that I said! If yes, 
then the authors plan will work: people 
! will be able to try small amounts and 
determine whether they're allergic. If no, 
then the authors plan is not a good one: 

^ trying small amounts won t actually help 

Oppl shld try tiny bit of 8 foods , , , „ . 

^ rr 33 us tell whether we re allergic. 



to see if FA 



7. News War: The correct answer is C. 



Step 1: Identify the question. 

The conclusion above is properly 
drawn if which of the following | 
is assumed? 

Step 2: Deconstruct the argument. 

For several years, Nighttime 
News attracted fewer viewers 
than World News, which broad- 
casts its show at the same time 
as Nighttime News. 
Recently, the producers of 
Nighttime News added personal 
interest stories and increased 
coverage of sports and weather. 
The two programs now have a 
roughly equal number of view- 
ers. 



Clearly, the recent program- 
ming changes persuaded viewers 
to switch from World News to 
Nighttime News. 



FA 



A B C D E 



FA A B C D E 
past: NN< WN 



FA A B C D E 

past:NN<WN 

rec: NN + pers, sp, weath 

FA ABCDE 

past:NN<WN 

rec: NN + pers, sp, weath 

now:NN = WN 
FA ABCDE 

past: NN < WN 
I rec: NN + pers, sp, weath 

now:NN = WN 
©prog A -» switch 



; The word "assumed" tells me that 
\ this is a Find the Assumption 
question. 



NN and WN are competitors. In 
the past, WN got more viewers. 
Facts = premises. 



NN added certain new things. 



Now, the two are about equal. 
; Interesting. Why? So far, all 
I premises. 



Conclusion! The author is claim- 
ing that the new programming 
\ things actually caused people to 
J switch from one show to the other. 
Hmm — that would mean WNs 
numbers went down — did they? 
Or is it just that NN went up? Or 
maybe there's some other reason 
\fbr the change entirely. 



Assumptions 



Chapter 4 



Step 3: State the Goal. 



The author is claiming specifically that people switched from WN to NN — but there's no evidence for that. 
The author is assuming that, ifNN's numbers went up, then WN's numbers went down and that those 
people switched to NN (and didn't start watching something else or turn off their TVs entirely!). 

The author's also assuming that the reason for the switch was NN's new programming and not something else. 

I need to find an answer that represents something the author must believe to be true. 

Step 4: Work from wrong to right. 



(A) Viewers are more 
interested in sports and 
weather than in per- 
sonal interest stories. 



(B) The programming 
content of Nighttime 
News is more closely 
aligned with the inter- 
ests of the overall audi- 
ence than is the content 
of World News. 

(C) Some World News 
viewers liked the new 
Nighttime News pro- 
gramming better than 
they liked the World 
News programming. 

(D) There are other 
possible causes for an 
increase in the number 
of viewers of Night- 
time News, including 
a recent ad campaign 
that aired on many local 
affiliates. 



FA A B C D E 
past: NN < WN 
rec: NN + pers, sp, weath 
now: NN = WN 
©prog A -» switch 

FA A B C D E 

past: NN < WN 

rec: NN + pers, sp, weath 

now:NN = WN 

©prog A -» switch 

FA ABCDE 

past: NN < WN 

rec: NN + pers, sp, weath 

now: NN = WN 

©prog A -» switch 
FA ABCBE 

past: NN < WN 

rec: NN + pers, sp, weath 

now: NN = WN 

©prog A -> switch 



Hmm. NN added all 3 of these things. Does 
the author need to assume that two are more 
popular than the third? No — it doesn't mat- 
ter as long as the programming in general 
did make people switch. Maybe they're trying 
to get me to think that the choice is compar- 
ing WN and NN — but that's not what this 
choice actually says. 

This basically says that the audience likes 
NNs content better than WN's content. That 
could be a reason to switch. Does it absolutely 
have to be true? It also addresses the program- 
ming issue, so it does seem pretty good — I'll 
leave it in for now. 

This also talks about liking NN better than 
WN. In particular, it says that some WN 
viewers decided they liked the new NN stuff 
better. That also looks really good. Leave it 
in. 



"Other possible causes" — oh, no, are all three 
of these choices good? Wait a second. I'm read- 
ing this backwards. This is saying there are 
other reasons why more people are watching 
NN, so that would actually hurt the author's 
claim that it's because WN viewers switched 
due to the programming. 



GMAT 



135 



Chapter 4 



Assumptions 



(E) The quality of 
World News will remain 
constant even if Night- 
time News improves. 



compare B and C 




FA ABCBE 

past: NN < WN 

rec: NN + pers, sp, weath 

\ now:NN = WN 

©prog A switch 
FA AB©BE 

past: NN < WN 

rec: NN + pers, sp, weath 

now: NN = WN 

©prog A switch 



If this were true, it might help explain why 
some people would switch, but does it HAVE 
to be true in order to claim that people 
already switched due to NNs new program- 
ming? No. 

Lets try negating B and C. 

B: NN content is not more closely aligned 
with audience than WN content. Maybe 
they're about the same? That doesnt really 
hurt the authors argument all that much. 

C: None of the WN viewers liked NN better 
than WN Wait a second. If NONE of them 
liked NN better, why would they switch? 
Negating this definitely hurts the argument. 
Citisl 



8. Five-Step Process: The correct answer is B. 
Step 1: Identify the question. 



Which of the following would 
be most useful in evaluating the 
claim made in the argument? 

Step 2: Deconstruct the argument. 

Manager: the new manufactur- 
ing process should save us time 
overall, even though the first 
step of the five-step process will 
take twice as long as it does 
under the old process. 
Under the new process, far 
fewer of the components will 
be found defective, and the sole 
purpose of steps two and three 
under the old process is to weed 
out defective components. 



Ev 



A B C D E 



Ev A B C D E 

M: New MP faster but SI 
2x longer 



Ev A B C D E 

M: New MP faster but SI 
2x longer 

NMP = I bad parts 
OMP S2+3 for bad parts 



The language "most useful in 
evaluating" indicates that this is 
an Evaluate question. 



This is a claim. It could be the 
conclusion — F 11 have to keep 
reading to tell. 



This seems to be a combo of a 
claim and a fact, but both are 
supporting the first sentence. 



136 



GMAT 



Assumptions 



As a result, we should be able to 
eliminate two of the five steps 
in the existing manufacturing 
process. 



Yes, the first sentence was the con- 
clusion. If the other things are all 
true, then maybe the NMP will 
be faster than the old one. 



Ev A B C D E 

M: New MP faster but SI 
2x longer © 

NMP = 4 bad parts 

OMP S2+3 for bad parts 

so elim 2 steps for NMP 



Step 3: State the Goal. 

This is an Evaluate question, so I need to find an answer that will help to determine whether or not the 
conclusion is likely to be valid. The correct answer will have "two paths": one path will make the conclusion a 
little more likely to be valid, and the other will make the conclusion a little less likely to be valid. 

The manager is claiming that the new process will be faster than the old process. Although the 1st step will 
take twice as long under the new process, the manager claims they "should" be able to drop the second and 
third steps. If dropping the second and third steps saves even more time than is lost during the first step, then 
the manager might be right. . . but the manager is assuming that these other steps will save a lot more time. 



Step 4: Work from wrong to rig ht. 



(A) Whether factory 
workers will require 
training in order to use 
the new manufacturing 
process 



(B) Whether the new 
process is likely to 
introduce deficiencies or 
imperfections that must 
be corrected 



Ev A B C D E 

M: New MP faster but Si 
= 2x longer © 

NMP = I bad parts 

OMP S2+3 for bad parts 

so elim 2 steps for NMP 
Ev ABCDE 

M: New MP faster but SI 
= 2x longer © 

NMP = I bad parts 

OMP S2+3 for bad parts 

so elim 2 steps for NMP 



If they do. . . that may or may not affect how 
much time the process takes. If they dont, I 
still dont know anything more about how 
much time the new process is going to take 
versus the old process. 



If the new process also introduces problems 
that then need to be fixed, then perhaps they 
cant drop steps two and three, or perhaps 
they have to introduce other new steps to fix 
the deficiencies. . . either of which would add 
time to the new process, making it less likely 
that the new process will save time. If the new 
process does not introduce new imperfections 
that need to be fixed, then that increases the 
likelihood that the new process will save time. 



Assumptions 



(C) Whether defective 
components can be 
fixed or must be thrown 
out 



(D) Whether a third 
manufacturing process 
would save even more 
time than both the old 
and new manufacturing 
processes 

(E) Whether saving 
time with the new 
manufacturing process 
will ultimately lead 

to cost savings for the 
company 



Ev ABODE 

M: New MP faster but Si 
= 2x longer © 

NMP = i bad parts 

OMP S2+3 for bad parts 

so elim 2 steps for NMP 

Ev ABODE 

M: New MP faster but SI 
= 2x longer © 

NMP = I bad parts 

OMP S2+3 for bad parts 

so elim 2 steps for NMP 
Ev A(B)€ B E 

M: New MP faster but SI 
= 2x longer © 

NMP = I bad parts 

OMP S2+3 for bad parts 

so elim 2 steps for NMP 



If defective components can be fixed, that 
would add time to the process. If defective 
components must be thrown out, that would 
also add manufacturing time, because they 
would have to make even more. This doesnt 
give us two different paths, one of which helps 
the conclusion and one of which hurts the 
conclusion. 

The conclusion focuses on whether the new 
process is faster than the old process. Introduc- 
ing a third, different process tells us nothing 
about the first two processes or how long they 
are. 



The argument does not address anything 
about cost savings — the focus of the argu- 
ment's conclusion is solely about saving time. 
Whether the company ultimately saves money 
does not tell us whether they 11 save time. 



9. Genetics: The correct answer is A. 



Step 1: Identify the question. 



Which of the following is an 
assumption on which the argu- 
ment depends? 

Step 2: Deconstruct the argument. 

Two genes, BRCA1 and 
BRCA2, are linked to heredi- 
tary breast cancer. 



FA 



ABODE 



FA ABODE 
Gl and G2 = BC 



The word "assumption" indicates 
that this is a Find the Assumption 
question. 



Straight fact. 



Assumptions 



Therefore, in order to decrease 
j the current number of mam- 
mogram tests administered 
j across a population and to more 
accurately zssess a woman's 
individual risk of breast cancer, 
all women should be tested for 
these genes. 

Step 3: State the Goal. 



FA A B C D E 

Gl and G2 = BC 

)test all W to I M and assess 
risk 



Complicated. Okay, the authors 
recommending that all women be 
tested and claims this will do 2 
things: decrease the # of Ms and 
better assess risk. So one assump- 
tion could be that those who test 
negatively wont get an Mas 
frequently. 



The author claims that, if women are all tested for these genes, two things will happen: the number of Ms will 
go down and they'll be able to assess risk more accurately. 

I need to find an answer that the author must believe to be true in drawing this conclusion. That might have 
something to do with the number of Ms or with assessing risk. 



Step 4: Work from wrong to right. 



(A) Some of the women 
who are tested for the 
two genes will sub- 
sequently undergo 
mammograms on a less 
frequent basis than they 
used to. 

(B) The majority of 
breast cancer patients 
have no family history 
of the disease. 



FA A B C D E 

Gl and G2 = BC 

)test all W to I M and as- 
sess risk 

FA AfiCDE 

Gl and G2 = BC 

)test all WtoiM and as- 
sess risk 



(C) Researchers may 
have identified a third 
breast cancer gene that 
is linked with hereditary 
breast cancer. 



FA AB6DE 

Gl and G2 = BC 

)test all W to I M and as- 
sess risk 



If at least some women get tested and then 
get fewer Ms, then that would help to reduce 
the number of Ms. But does this HAVE to be 
true? Actually, I think so. It has to be the case 
that women who otherwise would' ve got- 
ten Ms don't; otherwise, the number cant go 
down. 

I'm not sure how this affects the "number of 
Ms" claim, but I think it actually hurts the 
"better assess risk " claim. It seems like the 
argument assumes that if you don't have the 
gene, you wont get Ms, but then this choice 
says a lot of women who do get BC don't have 
a family history. Also, someone can have a 
gene and not develop BC, so maybe that's why 
there's no family history. Too many "ifs" on 
this one. 

If so, then presumably the author of the argu- 
ment might want to add this third one to 
the list. But that has nothing to do with the 
argument as it stands. 



Chapter 4 



Assumptions 




(D) Women who have 
these genes have an 80 
percent chance of get- 
ting breast cancer, while 
women who do not have 
these genes have only 

a 10 percent chance of 
getting breast cancer. 

(E) The presence of 
BRCA1 and BRCA2 
can explain up to 50 
percent of hereditary 
cases. 



FA A B € B E 

Gl and G2 = BC 

)test all WtolM and as- 
sess risk 



FA (A)B € © E 

Gl and G2 = BC 

)test all W to I M and as- 
sess risk 



If that's true, then it does sound like knowing 
whether you have the gene would help more 
accurately assess your risk. Does this HAVE 
to be true? Not with those specific numbers, 
actually. Tricky. Maybe it's 70 percent or 90 
percent instead of 80%; the message is still the 
same. 

So, of the women who inherit BC, the genes 
account for about half of cases. This is kind 
of like the last one — that specific number 
doesnt have to be true. 



MANHATTAN 
GMAT 



Chapter/5 





Critical Reasoning 



Strengthen and Weaken 



Strengthen and Weaken: The Basics 
Strengthen the Conclusion Questions 
Weaken the Conclusion Questions 

EXCEPT Questions 



Strengthen and Weaken 



In Chapter 4, we introduced the Assumption Family and discussed Assumption, Evaluate, and Flaw 
questions. If you haven't read chapter 4 yet, please do so before reading this chapter; we'll wait! 

To recap briefly: 

• assumptions are something an author must believe to be true in order to draw his or her 
conclusion; these assumptions are NOT stated explicitly in the argument 

• all assumption arguments will contain a "core": a conclusion and the major premise or 
premises that lead to it 

• all assumption arguments will contain at least one (and probably more than one) assump- 
tion 

In this chapter, we'll address the remaining two Assumption Family question types: Strengthen the 
Conclusion and Weaken the Conclusion. 

As we saw with the previous three question types, these last two question types also hinge upon identi- 
fying an assumption; each has certain characteristics, though, that warrant separating them into these 
different categories. 



143 



5 Strengthen and Weaken 
Strengthen and Weaken: The Basics 

Both Strengthen and Weaken questions ask us to find a new piece of information that, if added to the 
existing argument, will make the conclusion either somewhat more likely to be true (Strengthen) or 
somewhat less likely to be true (Weaken). The fact that this information is new, or goes beyond what 
we already know from the argument, is the major difference between Strengthen and Weaken questions 
and the three question types we examined in the last chapter. 

In the case of a Strengthen, the new piece of info will serve as evidence that some assumption is actually 
valid. In the case of a Weaken, the new piece of info will knock down some assumption: it will serve as 
evidence that the assumption is invalid. 

How does that work? Let's look at one of our arguments from the last chapter again, about Thomas's 
football team: 



Last years 2nd place 
team has improved 




Team will win CH 
this year 



(premise) (conclusion 

If we were asked an Assumption question, the answer might be something like: Thomas's team has im- 
proved enough to be better than the defending champions. In order for the author to draw his conclu- 
sion, that must be true. If Thomas's team hasn't improved enough to be better than the first-place team, 
then how could we possibly conclude that Thomas's team will win this year? 

If we're asked a Strengthen question, how does the answer change? A Strengthen answer provides us 
with some new piece of information that does not have to be true, but if true, it does make the conclu- 
sion more likely to be valid. For example: 

The star quarterback on the defending champion team will miss the game due to 
an injury. 

Must it be true that the star quarterback will miss the game in order for Thomas's team to win? No. It's 
just a new piece of data, one we couldn't have anticipated, but if that information is true, then it also 
makes the conclusion more likely to be true. Thomas's team is more likely to win if a star player on the 
opposing team can't play. 

What happens if we're asked a Weaken question? Similarly, a Weaken answer provides us with some 
new piece of information that does not have to be true but, if true, does make the conclusion a bit less 
likely to be valid. For example: 

The players on the defending champion team train more than the players on any 
other team. 



MANHATTAN 
GMAT 



Strengthen and Weaken 



That specific fact does not have to be true in order for us to doubt the claim that Thomas's team will 
win — there are lots of reasons for us to doubt the claim — but if it is, indeed, true, that the defending 
champion team trains more than all the other teams, then the authors conclusion just got weaker. 

Note that all Strengthen and Weaken question stems do include the words "if true" or an equivalent 
variation. In other words, were explicitly told to accept the possibility that the information in the an- 
swer is actually true. 

Finally, there are three possible ways that an answer choice could affect the conclusion on both 
Strengthen and Weaken questions: the answer strengthens the conclusion, the answer weakens the con- 
clusion, or the answer does nothing to the conclusion. As we assess the answers, we'll be keeping these 
three categories in mind; we'll label Strengthen answers with an 5, Weaken answers with a W, and 
"nothing" answers with an n. We'll discuss this in more detail later. 

Strengthen the Conclusion Questions 

As we discussed, Strengthen questions ask us to find a new piece of information that, if added to the 
existing argument, will make the conclusion somewhat more likely to be true. 

Most often, Strengthen questions will contain some form of the words "strengthen" or "support," as 
well as the phrase "if true." Here are some typical examples: 

Which of the following, if true, most strengthens the argument above? 

Which of the following, if true, most strongly supports the mayor's claim? 

Strengthen questions will sometimes use synonyms in place of the strengthen / support language; such 
synonyms might include: 

• provides the best basis or the best reason for 

• provides justification for 

• provides a piece of evidence in favor of (a plan or a conclusion) 

Strengthen questions may occasionally lack the exact phrase "if true" but, if so, some other wording 
will provide a similar meaning. That wording might be something quite similar, such as "if feasible." 
Alternatively, the wording might indicate that the answer can be "effectively achieved" or "successfully 
accomplished" (indicating that the information would become true). 



Chapter 5 



Strengthen and Weaken 



Try out this short example: 




At QuestCorp, many employees have quit recently and taken jobs with a competi- 
tor. Shortly before the employees quit, QuestCorp lost its largest client Clearly, 
the employees were no longer confident in QuestCorp's long-term viability. 

Which of the following, if true, most strengthens the claim that concerns about 
QuestCorp's viability caused the employees to quit? 

(A) Employees at QuestCorp's main competitor recently received a large and 
well-publicized raise. 

(B) QuestCorp's largest client accounted for 40% of sales and nearly 60% of the 
company's profits. 

(C) Many prospective hires who have interviewed with QuestCorp ultimately 
accepted jobs with other companies. 

The question stem indicates that this is a strengthen question. Our core might look similar to this: 



therefo, 



re 



lost client, 
ppl quit 

(premise) 



quit b/c concern 
re: success 

(conclusion) 



Note: you can write the core down as we did above, or you can articulate the core to yourself mentally 
(or both!). Whichever path works best for you is fine. 

That's interesting. The author claims that, because the company lost its largest client, some employees lost 
confidence in the company, so they quit. The author assumes that losing that client will be a significant blow 
to the company. What if the company has many clients and the largest client only represented a very small 
fraction of the business? The author also assumes there aren't other reasons why employees would have quit. 

This is a strengthen question, so I have to find some evidence that actually does support the claim that people 
quit specifically because they lost confidence in the company after it lost its largest client. 



(A) Employees at Quest- 
Corp's main competitor 
recently received a large 
and well-publicized 



raise. 



ABC 

w 



lost client, ppl quit 
©be concern re: success of 



com 



Wouldn't that make QC's employees jeal- 
ous — maybe they'd expect more money? 
That'd make it more likely that they quit 
because of$ issues rather than a loss of 
confidence in the company. If anything, this 
actually weakens the conclusion; I want a 
strengthen answer. 



146 



GMAT 



Strengthen and Weaken 



Chapters 



ABC 

w s 



(B) QuestCorp s largest 
client accounted for 
40% of sales and nearly 
60% of the company's 
profits. 

(C) Many prospective 
hires who have inter- 
viewed with QuestCorp 
ultimately accepted jobs © bc concern re: success of 
with other companies. com 



lost client, ppl quit 

©be concern re: success of 
com 

S A®€ 
lost client, ppl quit 



Ouch. Then losing this client would be a 
pretty serious blow to the company. This is 
a fact that helps make the conclusion a little 
more likely; Fll keep it in. 

Hmm. "Prospective hires" are not employ- 
ees. I was asked to strengthen the part about 
employees losing confidence in QC. We could 
speculate that maybe something is wrong with 
QC if people take other jobs... but the answer 
doesnt even tell us why these people took other 
jobs. Maybe QC rejected them! 



Answer choice A represents one common trap on Strengthen questions: the answer does the opposite of 
what we want. That is, it weakens the conclusion rather than strengthening it. 

Answer choice C represents another common trap: the answer addresses (and sometimes even strength- 
ens) something other than what we were asked to address. In this case, the answer does seem to im- 
ply that there's something not so great about QuestCorp, but it discusses the wrong group of people 
(prospective hires) and doesnt actually provide any information that allows us to zssess what they think 
of QuestCorp's viability. (Again, that last part doesn't matter in the end, because we're already talking 
about the wrong group of people in the first place.) 




Putting It All Together 

Let's try a full problem now. 

Donut Chain, wishing to increase the profitability of its new store, will place a 
coupon in the local newspaper offering a free donut with a cup of coffee at its 
grand opening. Donut Chain calculates that the cost of the advertisement and the 
free donuts will be more than recouped by the new business generated through 
the promotion. 

Which of the following, if true, most strengthens the prediction that Donut 
Chain's promotion will increase the new store's profitability? 

(A Donut Chain has a loyal following in much of the country. 

(B) Donut Chain has found that the vast majority of new visitors to its stores 
become regular customers. 

(C) One donut at Donut Chain costs less than a cup of coffee. 

(D) Most of the copies of the coupon in the local newspaper will not be re- 
deemed for free donuts. 

(E) Donut Chain's stores are generally very profitable. 



MANHATTAN 
GMAT 



Strengthen and Weaken 



Step 1 ; Identify the question. 



Which of the following, if true, S A B C D E 
most strengthens the prediction © DC promo _> f prof 
that Donut Chain s promotion 
will increase the new stores 
profitability? 



Step 2: Deconstruct the argument. 



Donut Chain, wishing to in- 
crease the profitability of its new 
store, will place a coupon in the 
local newspaper offering a free 
donut with a cup of coffee at its 
grand opening. 
Donut Chain calculates that 
the cost of the advertisement 
and the free donuts will be 
more than recouped by the new 
business generated through the 
promotion. 

(brainstorm assumptions) 



S A B C D E 
©DC promo -» t prof 
promo = give free coupon 

S A B C D E 
©DC promo ^ t prof 
promo = give free coupon 
$ spent < $ made 

S A B C D E 
©DC promo -» f prof 
promo = give free coupon 
$ spent < $ made 



Step 3: State the Goal. 



The language "if true" and "most 
| strengthens the prediction that. . . " 
indicates that this is a strengthen 
the conclusion question. Also, the 
question stem tells me the conclu- 
sion I need to address: the plan 
will lead to better profitability. 



DC thinks that giving away a free 
donut will lead to increased profit- 
ability. 



It costs $ to place the ad and give 
away free donuts, but DC thinks it* 11 
get enough new business to offset those 
costs. Still, does that lead to better 
profitability? 

The argument isnt 100% clear that 
the profitability part is the conclu- 
sion, but the question stem also said 
so. I can label that info with my © 
symbol. The author is assuming that 
giving away a free donut once will 
lead to increased revenues over time 
(what if they never come back?), and 
that will then lead to increase profits 
(> revenues dont necessarily equal > 
profitability). 



I need to strengthen the claim that a particular plan is going to lead to increased profitability. The plan is to 
distribute coupons to give away free donuts. 



I need to find an answer that makes it a little more likely that this plan will lead to more profits. 



Strengthen and Weaken 



Chapter 5 



Step 4: Work from wrong to right 



(A) Donut Chain has a 
loyal following in much 
of the country. 



(B) Donut Chain has 
found that the vast ma- 
jority of new visitors to 
its stores become regular 
customers. 

(C) One donut at Donut 
Chain costs less than a 
cup of coffee. 



(D) Most of the copies 
of the coupon in the 
local newspaper will not 
be redeemed for free 
donuts. 

(E) Donut Chains 
stores are generally very 
profitable. 



S A B C D E 

n 

©DC promo -» t prof 

promo = give free coupon 

$ spent < $ made 
S A B C D E 

n S 

©DC promo -> T prof 

promo = give free coupon 

$ spent < $ made 
S A B € D E 

n S ~~ 

©DC promo -» t prof 

promo = give free coupon 

$ spent < $ made 
S A B € © E 

n S n W 

©DC promo t prof 

promo = give free coupon 

$ spent < $ made 
S A B € & E 

n S n W ^ 

©DC promo -> t prof 
promo = give free coupon 
$ spent < $ made 



Common Trap Answers 



This is good for DC. Does that mean it will 
increase profitability though? No. It's already 
an established fact. Plus, it only says that 
DC enjoys a loyal following in "much " of the 
country, not absolutely everywhere. 

So if DC can get people to visit once, they'll 
usually keep coming back. That sounds pretty 
good for DCs plan, which is all about get- 
ting people to visit the first time for that free 
donut. 

This tells me nothing about profits or reve- 
nues or how much they could sell or anything, 
really. This doesn't address the conclusion. 



If this happens, then DCs plan is really un- 
likely to work — it spends money on the ads, 
but never gets the new customers to come in. 
That weakens the conclusion. 

It's good that DC stores are usually profit- 
able; that means this new one is likely to be 
profitable, too. The claim, though, specifi- 
cally asks about increasing the store's profit- 
ability — and it specifically asks whether this 
plan will accomplish that goal. This choice 
looks tempting at first, but it doesn't address 
whether this plan will increase profitability. 



One of the most common traps is the Reverse Logic answer: the question asks us to strengthen, but a 
trap answer choice weakens the conclusion instead. We saw an example of this with answer choice D 
in the last problem. These can be especially tricky if we misread the conclusion or otherwise get turned 
around while evaluating the argument. 

Most of the wrong answers will have No Tie to the Conclusion — they will neither strengthen nor 
weaken the conclusion. Some of these will be more obviously wrong, but these answers can also be quite 
tricky. A No Tie trap might address something in a premise without actually affecting the conclusion, 
for example, as we saw with answer choice E in the last problem. 




GMAT 



149 



Strengthen and Weaken 
Takeaways for Strengthen Questions 

The question stem will contain "if true" or a close synonym, as well as some form of the word 
"strengthen" or "support" (or a synonym). We will write down S to indicate that we have a Strengthen 
question. 

On Strengthen questions, our goal is to find a new piece of information that makes the conclusion at 
least somewhat more likely to be valid. 

The most common trap answers include the Reverse Logic trap (weakening the conclusion rather than 
strengthening it) and the No Tie trap (doesn't affect the specific conclusion). 

Weaken the Conclusion Questions 

As we discussed earlier, Weaken questions ask us to find a new piece of information that, if added to 
the existing argument, will make the conclusion somewhat less likely to be valid. Our goal, then, is to 
attack the conclusion. The correct answer will attack some assumption made by the author. 

Most Weaken question stems contain the phrase "if true" (or an equivalent) and question stems similar 
to these examples: 

• Which of the following, if true, most seriously weakens the conclusion? (variant: which is 
a weakness?) 

• Which of the following, if true, would cast the most serious doubt on the validity of the 
argument? (variant: raise the most serious doubt regarding) 

• Which of the following, if true, most strongly calls into question the authors conclusion? 

• Which of the following, if true, most seriously undermines the mayors claim? 

Sometimes, the question stem will contain more unusual language, such as the words in quotes below: 

• find a "disadvantage" or what is "damaging" to the argument 

• a plan is "ill-suited" or otherwise unlikely to succeed 

• find a "criticism" of the argument 



Strengthen and Weaken 



Chapter 5 



Let s look at the same short example about QuestCorp, but with a different question and answers! 

At QuestCorp, many employees have quit recently and taken jobs with a competi- 
tor. Shortly before the employees quit, QuestCorp lost its largest client. Clearly, 
the employees were no longer confident in QuestCorp's long-term viability. 

Which of the following, if true, most seriously undermines the claim that concerns 
about QuestCorp's viability caused the employees to quit? 

(A) A new competitor in the same town provides health insurance for its em- 
ployees, a benefit that QuestCorp lacks. 

(B) QuestCorp is unlikely to be able to replace the lost revenue via either an 
increase in existing client sales or the attraction of new clients. 

(C) Many prospective hires who have interviewed with QuestCorp ultimately 
accepted jobs with other companies. 

The question stem indicates that this is a Weaken question. Our core might look similar to this: 



lost client, 
ppl quit 

(premise) 



therefore 



quit b/c concern 
re: success 

(conclusion) 




Note: as we discussed on Strengthen question, you can write the core down as we did above, or you can 
articulate the core to yourself mentally (or both!). Whichever path works best for you is fine. 

Hmm. The author claims that losing this client caused employees to lose confidence in QC, which then caused 
them to quit. The author is assuming that losing this one client was serious enough to result in a major prob- 
lem for the company; is that necessarily the case? This is a Weaken question, so I have to find some evidence 
that makes it less likely that people quit for that reason. That could be because it wasn't really a big problem, 
or it could be that there was some other reason that people quit. 



ABC 

w 



lost client, ppl quit 



(A) A new competitor in 
the same town provides 
I health insurance for its 
employees, a benefit that © bc concern re: success of 
QuestCorp lacks. 



com 



The argument claims that people left for one 
reason, but this answer actually provides an 
alternative. Maybe people quit because they 
could get better benefits at the other company. 
This would weaken the claim that people quit 
specifically because of concerns over QCs vi- 
ability as a company. 



MANHATTAN 
GMAT 



151 



Strengthen and Weaken 



(B) QuestCorp is 
unlikely to be able to 
replace the lost revenue 
via either an increase in 
existing client sales or 
the attraction of new 
clients. 

(C) Many prospective 
hires who have inter- 
viewed with QuestCorp 
ultimately accepted jobs 
with other companies. 



S ABC 

w s 

lost client, ppl quit 

©be concern re: success of 
com 



S A B € 

W S n 

lost client, ppl quit 

©be concern re: success of 
com 



So QuestCorp lost its largest client, which 
means a loss of revenue, and the company 
probably cant find a way to make up that 
revenue through other sales. That definitely 
reinforces the problem described in the argu- 
ment. This actually strengthens the conclu- 
sion; that's the opposite of what I want. 
Hmm. "Prospective hires" are not employees. I 
was asked to weaken the part about employees 
losing confidence in QC. We could speculate 
that maybe something is wrong with QC 
if people take other jobs. . . but the answer 
doesnt even tell us why these people took other 
jobs. Maybe QC rejected them! 



Answer B repeats the common Reverse Logic trap we discussed earlier: it strengthens the conclusion but 
we want to weaken the conclusion. Answer C attempts to distract us by talking about a different part 
of the argument — perhaps we'll reason that, if interviewees took different jobs, that means they didn't 
believe QuestCorp was a good company. We have no idea why these prospective hires ended up work- 
ing for another company, though — its entirely possible that QuestCorp didn't extend a job offer to 
these people. 

Note that we used the exact same answer choice C for both the Strengthen and Weaken versions of this 
QuestCorp problem. If a choice is irrelevant to the argument (as choice C was), then it doesn't mat- 
ter whether we're asked to strengthen or weaken the conclusion. An irrelevant choice doesn't affect the 
conclusion at all. 



Strengthen and Weaken 



Chapters 



Let s try a full example: 

The national infrastructure for airport runways and air traffic control requires 
immediate expansion to accommodate the increase in private, smaller planes. To 
help fund this expansion, the Federal Aviation Authority (FAA) has proposed a fee 
for all air travelers. However, this fee would be unfair, as it would impose costs on 
all travelers to benefit only the few who utilize the new private planes. 

Which of the following, if true, would cast the most doubt on the claim that the 
proposed fee would be unfair? 

(A) The existing national airport infrastructure benefits all air travelers. 

(B) The fee, if imposed, will have a negligible impact on the overall volume of 
air travel. 

(C) The expansion would reduce the number of delayed flights resulting from 
small private planes congesting runways. 

(D) Travelers who use small private planes are almost uniformly wealthy or 
traveling on business. 

(E) A substantial fee would need to be imposed in order to pay for the expan- 
sion costs. 




Step 1; Identify the question. 



Which of the following, if true, 
would cast the most doubt on 
the claim that the proposed fee 
would be unfair? 



W A B C D E 
©fee = unfair 



The language "cast the most doubt 
on the claim" tells me that this is 
a weaken question. The specific 
claim Fm attacking is that the 
proposed fee would be unfair. 



Step 2: Deconstruct the argument. 



The national infrastructure for 
airport runways and air traf- 
[ fic control requires immediate 
expansion to accommodate 
the increase in private, smaller 
planes. 

To help fund this expansion, 
! the Federal Aviation Authority 
(EAA) has proposed a fee for all 
air travelers. 



W A B C D E 
©fee = unfair 

t sm pi -> must exp infra 

W A B C D E 
©fee = unfair 

t sm pi must exp infra 
FAA: fee -» fund exp 



This is somewhat claim-like but 
it's written as a fact; I'm guessing 
its just background info, not the 
conclusion, but I'm not 100% 
sure. 

Okay, here's a plan. It could be 
the conclusion. The FAA wants to 
charge a fee to pay for the expan- 
sion. 



MANHATTAN 
GMAT 



Chapter 5 



Strengthen and Weaken 



However, this fee would be un- 



W 



A B C D E 




fair, as it would impose costs on © fce = unfair b/c ^ pay but 



all travelers to benefit only the 
few who utilize the new private 
planes. 



few ben 

t sm pi -* must exp infra 

FAA: fee -» fund exp 
W A B C D E 

©fee = unfair b/c all pay but 
few ben 

t sm pi -> must exp infra 
FAA: fee fund exp 



Change of direction! The author 
disagrees with the fee plan, claim- 
ing its unfair. Her reasoning is 
that everyone would have to pay 
the fee, but only a few people 
would benefit. 

Why wouldn't everyone benefit? 
If there's more space, then all the 
planes will be able to take off 
more quickly. The author is as- 
suming the benefit is only for the 
people flying in small planes. 



Step 3: State the Goal. 



The airports are congested because there are so many small planes, and the FAA wants to charge a fee to ex- 
pand the airports. The author claims that this is unfair because the fee would be paid by all but the expansion 
would only benefit a few. 

I want to weaken the authors conclusion, so I need to find some reason why it really isn't unfair. One pos- 
sibility that I brainstormed: maybe everyone really will benefit, not just the "small plane" people. 



Step 4: Work from wrong to right 



(A) The existing nation- 
al airport infrastructure 
benefits all air travelers. 



(B) The fee, if imposed, 
will have a negligible 
impact on the overall 
volume of air travel. 



W A B C D E 

w 

©fee = unfair b/c all pay but 
few ben 

T sm pi -» must exp infra 

FAA: fee fund exp 

W AfiCDE 

w 

©fee = unfair b/c all pay but 
few ben 

t sm pi must exp infra 
FAA: fee -» fund exp 



This sounds like what I was thinking be- 
fore — everyone benefits, so why is it unfair 
for everyone to pay? Great; Til leave it in. 



A "negligible impact" means it wont really 
change anything. The fee wont change the 
volume of planes trying to fly. . . but that was 
never the plan. The plan was to raise money 
to expand the infrastructure — then they'll 
be able to handle more volume. This answer 
doesn't address the right thing. 



MANHATTAN 
GMAT 



Strengthen and Weaken 



Chapter 5 



(C) The expansion 
would reduce the num- 
ber of delayed flights 
resulting from small 
private planes congest- 
ing runways. 

(D) Travelers who use 
small private planes 
are almost uniformly 
wealthy or traveling on 
business. 



(E) A substantial fee 
would need to be im- 
posed in order to pay for 
the expansion costs. 



W ABCDE 

w w 

©fee = unfair b/c all pay but 
few ben 

t sm pi -» must exp infra 

FAA: fee fund exp 

W ABCDE 

w w 

©fee = unfair b/c all pay but 
few ben 

t sm pi -» must exp infra 
FAA: fee fund exp 

W AfiCfiE 

w w 

©fee = unfair b/c all pay but 
few ben 

t sm pi ^ must exp infra 
FAA: fee -» fund exp 



Examine A and C again W AB^BE 

©fee = unfair b/c all pay but 
few ben 



Hmm. This is another potential benefit for 
everyone — a reduction in the number of 
flight delays. I'll leave this one in, too. 



t sm pi must exp infra 
FAA: fee -» fund exp 



That's nice for them, but what does it have to 
do with this argument? Maybe you could say 
"so they can afford to pay more," but that isn't 
the point of the argument. The point of the 
argument is that it's unfair to make the regu- 
lar travelers pay for something that doesn't 
benefit them (according to the author). 
So the fee would have to be pretty large. If 
anything, doesn't that make it even more 
unfair? Though, actually, Idont think it 
really addresses the fairness at all. Either it is 
fair, in which case the size of the fee doesn't 
matter, or it isn't fair. . . in which case the size 
of the fee still doesn't matter. 
Okay, I have to compare A and C now. 
Both say that this expansion would benefit 
everyone... wait a second. C does explicitly 
mention the expansion, but A says "the exist- 
ing. . . infrastructure. "Existing? Of course the 
existing structure benefits everyone who uses 
it — the argument isn 't about that. It's about 
whether the expansion would benefit every- 
one. Only C actually says that; I missed that 
the first time around. 




Common Trap Answers 

Weaken questions contain the same kind of common trap answers that we see on Strengthen questions. 

One of the most tricky types is the Reverse Logic trap: the question asks us to weaken, but a trap an- 
swer choice strengthens the conclusion instead. We will also again see the No Tie traps — choices that 
might discuss something in a premise but don't affect the conclusion. 

Our most tempting wrong answer in the last problem, answer choice A, is actually a No Tie trap. It was 
so tempting specifically because almost everything in the choice was addressing the right thing but one 
word made it wrong: "existing." The conclusion was about the future infrastructure, after an expansion, so 



MANHATTAN 
GMAT 



Chapter 5 



Strengthen and Weaken 



limiting the answer to the existing infrastructure meant that the information didn't affect the conclu- 
sion after all. 

Takeaways for Weaken Questions 

The question stem will contain "if true" or a close synonym, as well as some form of the words "weak- 
en," "doubt," "undermine," or a synonym. We will write down "W" to indicate that we have a Weaken 
question. 

On Weaken questions, our goal is to attack the conclusion. The correct answer will be a new piece of 
information that makes the conclusion at least somewhat less likely to be valid. 

The most common trap answers include the Reverse Logic trap (strengthening the conclusion rather 
than weakening it) and the No Tie trap (doesn't affect the specific conclusion). 




EXCEPT Questions 



Assumption Family questions may also be presented in a "negative" form that is commonly referred to 
as Except questions. 

A regular Weaken question might read: 

Which of the following, if true, most seriously weakens the conclusion? 
An Except Weaken question might read: 

Each of the following, if true, weakens the conclusion EXCEPT: 

What is the difference in wording between those two questions? 

The first one tells us that one answer choice, and only one, weakens the conclusion. That is the answer 
choice that we want to pick. 

The second one tells us that four answer choices weaken the conclusion. These four are all wrong an- 
swers. What about the fifth answer — what does that one do? 

Many people will assume that the fifth one must do the opposite: strengthen the conclusion. This is 
not necessarily true. Rather, the fifth one cannot weaken the conclusion but it may not strengthen the 
conclusion either. It might have no impact whatsoever on the conclusion. 

For these negatively-worded questions, were going to use the "odd one out" strategy. Four of the an- 
swers will do the same thing; in the case of the above example, four answers will weaken the conclusion. 
The fifth answer will do something else. It doesn't matter whether the fifth one strengthens the conclu- 
sion or does nothing — all that matters is that it is the "odd one out." 




GMAT 



Strengthen and Weaken 



The four answers that do the same thing can be found by using the regular strategy that we would use 
for that question type. On a normal Weaken question, we want to choose the answer that makes the 
conclusion at least a little less valid. On an EXCEPT Weaken question, four answer choices will make 
the conclusion at least a little less valid, and we are going to cross those four choices off. The remaining 
answer will be the answer we pick. 

Here's a full example. 

Supporters of a costly new Defense Advanced Research Projects Agency (DARPA) 
initiative assert that the project will benefit industrial companies as well as the 
military itself. In many instances, military research has resulted in technologies 
that have fueled corporate development and growth, and this pattern can be 
expected to continue. 

Each of the following, if true, serves to weaken the argument above EXCEPT: 

(A) The research initiative will occupy many talented scientists, many of whom 
would otherwise have worked for private corporations. 

(B) In the past decade, DARPA has adopted an increasingly restrictive stance 
regarding the use of intellectual property resulting from its research. 

(C) If the DARPA initiative hadn't been approved, much of the funding would 
instead have been directed toward tax breaks for various businesses. 

(D) At any given time, DARPA is conducting a wide variety of costly research 
projects. 

(E) The research initiative is focused on defense mechanisms that will reduce 
injury to soldiers during combat, a need that is nonexistent for private cor- 
porations. 



Strengthen and Weaken 



Step 1: Identify the question. 



Each of the following, if true, 
serves to weaken the argument 
above EXCEPT: 



WEx 



A B C D E 



The language "serves to weaken* 
tells me that this is a weaken 
question. The word EXCEPT tells 
me that the 4 wrong answers will 
weaken and I want to pick the 
"odd one out" answer. 



Step 2: Deconstruct the argument 

Supporters of a costly new 
Defense Advanced Research 
Projects Agency (DARPA) ini- 
tiative assert that the project will 
benefit industrial companies as 
well as the military itself. 
In many instances, military re- 
search has resulted in technolo- ^ 
gies that have fueled corporate 
development and growth, and 
this pattern can be expected to 
continue. 



WEx A B C D E 
SD: ben ind corns & mil 



WEx A B C D E 

)SD: ben ind corns & mil 

past: mil res -» techs to help 
corns, will com 



The SDs (Supporters of DARPA) 
think that this really costly project 
will be good for companies and 
the military. 



Research has helped corns in the 
past, and the author claims this 
will keep happening in the future. 
That all supports the claim of the 
SDs: that the specific D project 
will be beneficial for corns. 



Step 3: State the Goal 

In the past, military research has helped companies, and the claim is that this D project will also help compa- 



nies. 



I want to find four answers that weaken the conclusion (make it at least a little less likely to be valid). The 
answer that doesnt weaken — the odd one out — is the correct answer. 



Step 4; Work from wrong to right. 



(A) The research initia- 
tive will occupy many 
talented scientists, many 
of whom would other- 
wise have worked for 
private corporations. 



A B C D E 

w 



WEx 

)SD: ben ind corns & mil 



past: mil res -» techs to 
help corns, will cont 



This benefits the military and specifically does 
not benefit the companies. That does weaken 
the idea that companies will benefit. 



Strengthen and Weaken 



Chapters 



(B) In the past decade, 
DARPA has adopted an 
increasingly restrictive 
stance regarding the use 
of intellectual prop- 
erty resulting from its 
research. 

(C) If the DARPA 
initiative hadn't been 
approved, much of the 
funding would instead 
have been directed 
toward tax breaks for 
various businesses- 
CD) At any given time, 
DARPA is conducting 
a wide variety of costly 
research projects. 



(E) The research initia- 
tive is focused on de- 
fense mechanisms that 
will reduce injury to 
soldiers during combat, 
a need that is nonexis- 
tent for private corpora- 
tions. 

Examine B and D again 



WEx A B C D E 

w w? 

)SD: ben ind corns & mil 

past: mil res -» techs to 
help corns, will cont 



WEx A B € D E 

w w? w 

)SD: ben ind corns & mil 

past: mil res techs to 
help corns, will cont 



WEx A B € D E 

W W? W n 

©SD: ben ind corns & mil 

past: mil res -» techs to 
help corns, will cont 

WEx A B € D E 

WW? W n W 

©SD: ben ind corns & mil 

past: mil res -» techs to 
help corns, will cont 



WEx AB€@E 

W W? Wn W 

)SD: ben ind corns & mil 

past: mil res techs to 
help corns, will cont 



Hmm. "Restrictive" makes it sound like D 
doesn't let others use its research as much. If 
that's the case, then that would weaken the 
idea that companies will benefit. I'm not to- 
tally sure that's what this means though — the 
wording is tricky — so I'm not going to cross 
this one off yet. 

A tax break is a good thing, This choice is say- 
ing that the funding for the D project would 
instead have been spent on tax breaks, which 
is a definite benefit. So not giving those tax 
breaks is a bad thing for the companies; this 
does weaken the argument. 

This choice talks about all research projects D 
is conducting. Hmm. The argument makes 
a claim only about one specific project. Does 
this information make that claim more or 
less likely to be valid? I can't really see how it 
affects the argument's conclusion at all. 
The key here is the language "a need that is 
nonexistent for private corporations. " If the 
private companies don't have any need for the 
results of this particular research, then that 
weakens the claim that the D research will 
benefit companies. 



I need to compare B and D. I thought B 
might weaken a little bit, and I thought D 
didn't do anything to the conclusion. Between 
those two, I should choose the one that doesn't 
weaken at all, so I'm going to choose D. 




The biggest "trap answer" on an EXCEPT question is simply to forget halfway through that were 
working on an EXCEPT question. In other words, halfway through the above question, if I forget that 
it's a Weaken EXCEPT, I might accidentally pick a Weaken answer, or pick the answer that I think 
most weakens the conclusion. 



MANHATTAN 
GMAT 



159 



Chapter 5 



"1" 




Strengthen and Weaken 
Takeaways for EXCEPT Questions 

Any of the five Assumption Family question types can be worded as an EXCEPT question. When that 
happens, the four wrong answers will be formulated in whatever way a right answer would have been 
written on a normally-worded question of that type. That is: 



On EXCEPT 
questions 


Wrong answers will 


Right answers will 


Assumption 


be an assumption on which the 
argument depends 


NOT be an assumption on 
which the argument depends 


Evaluate 


help to tell us whether the conclu- 
sion may be valid 


NOT help to tell us whether 
the conclusion may be valid 


Flaw 


represent a flaw in the reasoning 


NOT represent a flaw in the 
reasoning 


Strengthen 


strengthen the conclusion at least 
a little 


NOT strengthen the conclu- 
sion 


Weaken 


weaken the conclusion at least a 
little 


NOT weaken the conclusion 



On EXCEPT questions, we remind ourselves first what the usual goal is for a normal question of that 
type. The four wrong answers will follow that typical goal, and the right answer will be the "odd one 
out" — it will NOT do what we typically expect on a normal question of that type. 

When writing down the question type, add the designation "Ex" to whatever you normally write down 
for that question type. 



MANHATTAN 
GMAT 



Strengthen and Weaken 

Problem Set 



Chapters 



1. Digital Video Recorders 

Advertising Executive: More than 10 million households now own digital video 
recorders that can fast-forward over television commercials; approximately 75% 
of these households fast-forward over at least one commercial per 30-minute 
program. Because television commercials are not as widely watched as they used 
to be, they are much less cost-effective today. 

Which of the following, if true, strengthens the claim that television commercials 
are less cost-effective than they used to be? 

(A) Product placement within television programs is a viable alternative to tradi- 
tional television commercials. 

(B) The television programs preferred by consumers without digital video re- 
corders are similar to those preferred by consumers with the devices. 

(C) Prior to the advent of digital video recorders, very few television viewers 
switched channels or left the room when commercials began. 

(D) The cost-effectiveness of television advertising is based less upon how many 
people watch a particular commercial and more upon the appropriateness 
of the demographic. 

(E) Many companies find it difficult to determine the exact return on invest- 
ment for television commercials. 

2. Smithtown Theatre 

The Smithtown Theatre, which stages old plays, has announced an expansion that 
will double its capacity along with its operating costs. The theatre is only slightly 
profitable at present. In addition, all of the current customers live in Smithtown, 
and the population of the town is not expected to increase in the next several 
years. Thus, the expansion of the Smithtown Theatre will prove unprofitable. 

Which of the following, if true, would most seriously weaken the argument? 

(A) A large movie chain plans to open a new multiplex location in Smithtown 
later this year. 

(B) Concession sales in the Smithtown Theatre comprise a substantial propor- 
tion of the theatre's revenues. 

(C) Many recent arrivals to Smithtown are students that are less likely to attend 
the Smithtown Theatre than are older residents. 

(D) The expansion would allow the Smithtown Theatre to stage larger, more 
popular shows that will attract customers from neighboring towns. 

(E) The Board of the Smithtown Theatre often solicits input from residents of 
the town when choosing which shows to stage. 

MANHATTAN 
GMAT 




Chapters 



Strengthen and Weaken 



3. Books and Coffee 

The owners of a book store and a nearby coffee shop have decided to combine 
their businesses. Both owners believe that this merger will increase the number 
of customers and therefore the gross revenue, because customers who come for 
one reason may also decide to purchase something else. 

Which of the following, if true, most weakens the owners' conclusion that a 
merger will increase revenue? 

Books and drinks can both be considered impulse purchases; often, they are 
purchased by customers without forethought. 

Profit margins at a coffee shop are generally significantly higher than profit 
margins at a book store. 

People who are able to read the first chapter of a book before buying are 
more likely to decide to buy the book. 

A large majority of the book store's current customer base already frequents 
the coffee shop. 

A combination book store and coffee shop that opened in a neighboring 
city last year has already earned higher than expected profits. 

4. Digital Coupons 

The redemption rate for e-mailed coupons is far lower than that for traditionally 
distributed paper coupons. One factor is the "digital divide"— those who might 
benefit the most from using coupons, such as homemakers, the elderly, and those 
in low-income households, often do not have the knowledge or equipment nec- 
essary to go online and receive coupons. 

Which of the following, if true, does the most to support the claim that the digital 
divide is responsible for lower electronic coupon redemption rates? 

(A) Computers are available for free in libraries, schools, and community centers. 

(B) The redemption rate of ordinary coupons is particularly high among elderly 
and low income people that do not know how to use computers. 

(C) Many homes, including those of elderly and low income people, do not have 
high-speed internet connections. 

(D) More homemakers than elderly people would use computers if they had ac- 
cess to them. 

(E) The redemption rate for coupons found on the internet has risen in the last 
five years. 




MANHATTAN 
GK/1AT 



Strengthen and Weaken 



5. Teacher Compensation 

Traditionally, public school instructors have been compensated according to 
seniority. Recently, educational experts have criticized the system as one that 
rewards lackadaisical teaching and reduces motivation to excel. Instead, these 
experts argue that, to retain exceptional teachers and maintain quality instruc- 
tion, teachers should receive salaries or bonuses based on performance rather 
than seniority. 

Which of the following, if true, most weakens the argument of the educational 
experts? 

(A) Some teachers express that financial compensation is not the only factor 
contributing to job satisfaction and teaching performance. 

(B) School districts will develop their own unique compensation structures that 
may differ greatly from those of other school districts. 

(C) Upon leaving the teaching profession, many young, effective teachers cite a 
lack of opportunity for more rapid financial advancement as a primary factor 
in the decision to change careers. 

(D) In school districts that have implemented pay for performance compensa- 
tion structures, standardized test scores have dramatically increased. 

(E) A merit-based system that bases compensation on teacher performance 
reduces collaboration, which is an integral component of quality instruction. 

6. The Gold Standard 

Brand X designs and builds custom sneakers, one sneaker at a time. It recently an- 
nounced plans to sell "The Gold Standard," a sneaker that will cost five times more 
to manufacture than any other sneaker that has ever been created. 

Which of the following, if it occurred, would cast the most serious doubt on the 
claim that The Gold Standard sneaker will be profitable? 

(A) The endorsement of The Gold Standard by a popular celebrity 

(B) The publication of a report indicating that all previous sneaker lines 
launched by Brand X have been profitable 

(C) A significant increase in the cost of the canvas used to construct The Gold 
Standard 

(D) The introduction of another new sneaker line by a rival manufacturer 

(E) An announcement by Brand X that The Gold Standard will be marketed as 
an exclusive offering, available only in limited quantities 



Chapters 



Strengthen and Weaken 



7. Machu Picchu 

In 2001 the Peruvian government began requiring tourists to buy permits to hike 
the Inca Trail to the ancient city of Machu Picchu. Only 500 people per day are 
now allowed to hike the Inca Trail, whereas before 2001 daily visitors numbered in 
the thousands. The Peruvian government claims that this permit program has suc- 
cessfully prevented deterioration of archaeological treasures along the Inca Trail. 

Which of the following, if true, most strengthens the argument above? 

Since 2001, Incan ruins similar to Machu Picchu but without a visitor limit 
have disintegrated at a significantly greater rate than those on the Inca Trail. 
Villages near Machu Picchu have experienced declines in income, as fewer 
tourists buy fewer craft goods and refreshments. 

Many of the funds from the sale of Inca Trail permits are used to hire guards 
for archaeological sites without permit programs. 
Since 2001, tourist guides along the Inca Trail have received 50% to 100% 
increases in take-home pay. 

The total number of tourists in Peru has risen substantially since 2001, even 
as the number of tourists hiking the Inca Trail has remained constant. 

Ethanol, a fuel derived from corn, can be used alone to power cars or along with 
gasoline to reduce the reduce the amount of gas consumed. Unlike gasoline, 
ethanol is easily renewable since it is primarily converted from the sun's energy. 
Moreover, compared with conventional gasoline, pure ethanol is a cleaner- 
burning fuel. To save energy and reduce pollution, many individuals advocate the 
increased usage of ethanol as a primary fuel source in conjunction with or in place 
of gasoline. 

In evaluating the recommendation to increase the use of ethanol, it would be 
important to research all of the following EXCEPT: 

(A) Whether the energy required to grow and process corn used as fuel is 
greater than the amount of energy ultimately produced 

(B) Whether more energy is saved when using ethanol in conjunction with or in 
place of gasoline 

(C) Whether ethanol is as efficient a fuel as gasoline 

(D) Whether it is possible to produce more ethanol than is currently produced 

(E) Whether the process of growing corn for fuel would result in as much pollu- 
tion as does the production of conventional gasoline 



(A) 
(B) 
(C) 




8. Ethanol 



MANHATTAN 
GMAT 



Strengthen and Weaken 



Chapter 5 



CEO: Over the past several years, we have more than doubled our revenues but 
profits have steadily declined because an increasing number of customers have 
failed to pay their balances. In order to compensate for these higher default rates, 
we will increase the interest charged on outstanding balances from an annual 
percentage rate (APR) of 9.5% to an APR of 12%. This increase will be sufficient to 
compensate for the current rate of defaults and allow us to increase our profits. 

Which of the following statements, if true, would most seriously undermine a plan 
to increase interest rates in order to spur profitable growth? 

(A) Many other companies have experienced a similar trend in their default 
rates. 

(B) The company's operating expenses are above the industry average and can 
be substantially reduced, thus increasing margins. 

(C) The increase in default rates was due to a rise in unemployment, but unem- 
ployment rates are expected to drop in the coming months. 

(D) The proposed increase in the APR will, alone, more than double the com- 
pany's profit margins. 

(E) An increase in the APR charged on credit card balances often results in 
higher rates of default. 




MANHATTAN 
GMAT 



165 



Chapter 5 



Strengthen and Weaken 



10. Jupiter vs. Mars 

Scientists suspect that Europa, a moon orbiting Jupiter, may contain living organ- 
isms. However, the government recently scrapped an unmanned science mission 
to Europa and replaced it with a project aimed at landing an astronaut on Mars. 
Polls show that the public is far more fascinated by space travel than by discover- 
ing life elsewhere in the universe. Critics argue that the government's decision- 
making process places a greater emphasis on popularity than it does on the 
importance of scientific research. 

Which of the following, if true, would most strengthen a contention by the gov- 
ernment that the new project is a better use of its funds? 

(A) In the first year of the project, the government will spend 30% of its total 



(C) Some scientists are convinced that a mission to Europa would add immea- 
surably to our understanding of the universe. 

(D) A new telescope that has just become available to scientists promises to 
yield more information than the planned mission to Europa was designed to 
provide. 

(E) Most people feel that a shuttle to Mars is the next logical step in the devel- 
opment of a system that will allow humans to travel even further in the solar 
system. 




(B) 



budget on developing a space shuttle that can travel to Mars; that figure is 
expected to drop to 0% after five years. 

The government cannot be absolutely certain of the chances for success of 
either project. 



166 



HANI* 
GMAT 



ATTAN 



Strengthen and Weaken 



Chapters 



Solutions 



1. Digital Video Recorders: The correct answer is C. 
Step 1: Identify the question. 

Which of the following, if S A B C D E 

true, strengthens the claim that /^w w C r 

I V comm < cost err now 

television commercials are less 
cost-effective than they used to 
be? 



Step 2: Deconstruct the argument. 

Advertising Executive: More 
than 10 million households now 
own digital video recorders that 
can fast-forward over television 
commercials; 

approximately 75% of these 
households fast-forward over 
at least one commercial per 
30-minute program. 

Because television commercials 
are not as widely watched as 
they used to be, they are much 
less cost-effective today. 



S A B C D E 
©TV comm < cost eff now 
AE: 10m hh = DVR 

S A B C D E 

©TV comm < cost eff now 

AE: 10m hh = DVR 

75% FF comms (#) 
S A B C D E 

©TV comm < cost eff now b/c 
ppl watch < 

AE: 10mhh = DVR 

75% FF comms (#) 



Step 3: State the Goal. 



The phrase "strengthens the 
claim" tells me that this is a 
Strengthen question. The question 
stem also tells me the conclusion: 
TV commercials are less cost- 
effective than they used to be. Til 
write that down. 



This is just a fact. 



Another fact. I don't think I need 
to write down the exact numeri- 
cal details right now, but I'll note 
that there are more numerical 
details just to remind myself 

This repeats the conclusion I 
already know from the Qstem, 
with a little more info. 



Okay, theAE claims that TV comms are not as cost-effective specifically because people aren't watching them 
as much, and that is specifically because most people fast-forward over at least some commercials. What as- 
sumptions are being made? 

Let's see. They're assuming that people really did watch TV commercials more, but they don't provide any evi- 
dence of that. Maybe people used to tape programs on VCRs and then still fast-forward. They haven't actually 
told us what people used to do before these DVRs came along. 




MANHATTAN 
GMAT 



Chapter 5 



Strengthen and Weaken 



/ want an answer that will make the claim a little more likely to be valid. 
Step 4: Work from wrong to rig ht. 




(A) Product place- 
ment within television 
programs is a viable 
alternative to traditional 
television commercials. 

(B) The television 
programs preferred 
by consumers without 
digital video record- 
ers are similar to those 
preferred by consumers 
with the devices. 

(C) Prior to the advent 
of digital video record- 
ers, very few televi- 
sion viewers switched 
channels or left the 
room when commercials 
began. 

(D) The cost-effec- 
tiveness of television 
advertising is based less 
upon how many people 
watch a particular com- 
mercial and more upon 
the appropriateness of 
the demographic. 

(E) Many companies 
find it difficult to deter- 
mine the exact return 
on investment for televi- 
sion commercials. 



S A B C D E 

©TV comm < cost eff now 
b/c ppl watch < 

AE: 10m hh = DVR 

75% FF comms (#) 
S AfiCDE 

©TV comm < cost eff now 
b/c ppl watch < 

AE: 10m hh = DVR 

75% FF comms (#) 



S ABCDE 

s 

©TV comm < cost eff now 
b/c ppl watch < 

AE: 10m hh = DVR 

75% FF comms (#) 

S AfiCBE 

s w 

©TV comm < cost eff now 
b/c ppl watch < 

AE: 10m hh = DVR 

75% FF comms (#) 

S AB©BE 

5 w w 

©TV comm < cost eff now 
b/c ppl watch < 

AE: 10mhh = DVR 

75% FF comms (#) 



That's nice for the advertisers who want to 
make money, but it's not talking about how 
to do so by NOT using commercials, so that 
cant affect the conclusion about whether TV 
commercials are less cost-effective now. 



Hmm. The DVR thing was used as evidence 
to show how some people are skipping com- 
mercials. I don't think making a distinction 
about people with or without the DVRs 
watch really tells us anything. The conclusion 
is about commercials, not what programs 
people watch. 

That's interesting. This is about what people 
used to do before DVRs. Oh, look — I didn't 
think of that, but they mention two other 
ways that people could skip commercials: 
changing the channel or leaving the room. 
And this answer says that people really didn't 
used to do that, so maybe they really were 
watching more TV commercials! 
Hmm. They're saying that we should be using 
a different metric to evaluate cost-effective- 
ness, not how many people watch. Yeah, that 
sounds convincing. Wait! My goal is to find 
something that makes the argument MORE 
likely to be valid. If anything, this would 
weaken the argument; this is a Reverse Logic 
trap! 

A lot of companies can't tell how much money 
they earn from people watching TV commer- 
cials. But maybe they can still tell something 
about the relative differences between a few 
years ago and now. Also, if this were actually 
true, if anything, the conclusion would be a 
little less valid, because that would mean we 
couldn't tell that the TV commercials are less 
cost-effective today. 



168 



GMAT 



Strengthen and Weaken 



Chapters 



2. Smithtown Theatre: The correct answer is D. 



Step 1: Identify the question. 



Which of the following, if true, 
would most seriously weaken 
the argument? 

Step 2: Deconstruct the argument. 

The Smithtown Theatre, which 
stages old plays, has announced 
an expansion that will double its 
capacity along with its operating 
costs. 

The theatre is only slightly prof- 
itable at present. 



In addition, all of the current 
customers live in Smithtown, 
and the population of the town 
is not expected to increase in the 
next several years. 
Thus, the expansion of the 
Smithtown Theatre will prove 
unprofitable. 



Step 3: State the Goal. 



W 



A B C D E 



W A B C D E 
ST: exp to tt cap & cost 



W ABODE 

ST: exp to tt cap & cost 

Now: barely prof 
W A B C D E 

ST: exp to tt cap & cost 

Now: barely prof, cust live in 
S, prob won't be more from S 

W A B C D E 

ST: exp to tt cap & cost 

Now: barely prof, cust live in 
S, prob wont be more from S 

)ST exp unprof 



The words "if true" and "weaken" 
tell me that this is a Weaken ques- 
tion. 



They have a plan. It's future, so 
it could be the conclusion, but 
I'm guessing there'll be more of a 
claim like "The ST will (or will 
not) be successful with its plan" or 
something like that. 
This is a fact. I wonder: if ST 
expands, will it get enough new 
business to continue covering costs? 

The first half is a fact; the second 
half is a future predication. So 
far, the case for ST's new plan 
doesn't sound very good. 

Okay, here's the conclusion. The 
author thinks the plan will fail 
and provides some pieces of evi- 
dence to support that claim. 



The ST has a plan to expand but the author claims that the plan will fail because the ST is only barely profit- 
able right now and it doesn't seem like there are a lot more opportunities to get new customers. 

I want something that will weaken the author's claim. I have to be careful here: weaken the idea that 
the plan will fail. I'm not weakening the plan itself — in fact, weakening the author's claim might mean 
strengthening the idea that the plan will work! 



Mil 

'■mm 



MANHATTAN 
GMAT 



169 



Chapters 



Strengthen and Weaken 



Step 4: Work from wrong to right. 



(A) A large movie chain 
plans to open a new 
multiplex location in 
Smithtown later this 
year. 



(B) Concession sales in 
the Smithtown Theatre 
comprise a substantial 
proportion of the the- 
atre's revenues. 



(C) Many recent arrivals 
to Smithtown are stu- 
dents that are less likely 
to attend the Smithtown 
Theatre than are older 
residents. 

(D) The expansion 
would allow the Smith- 
town Theatre to stage 
larger, more popular 
shows that will attract 
patrons from neighbor- 
ing towns. 

(E) The Board of the 
Smithtown Theatre 
often solicits input from 
residents of the town 
when choosing which 
shows to stage. 



W 



A B C D E 



ST: exp to tt cap & cost 

Now: barely prof, cust live 
in S, prob wont be more 
from S 

©ST exp unprof 

W A B C D E 

S n 

ST: exp to tt cap & cost 

Now: barely prof, cust live 
in S, prob won't be more 
from S 

©ST exp unprof 
W A B € D E 

S n S 

ST: exp to tt cap & cost 

Now: barely prof, cust live 
in S, prob won't be more 
from S 

)ST exp unprof 
W A B € D E 

S n S W 

ST: exp to tt cap & cost 

Now: barely prof, cust live 
in S, prob won't be more 
from S 

)ST exp unprof 
W AB€@E 

ST: exp to tt cap & cost 

Now: barely prof, cust live 
in S, prob won't be more 
from S 



Idont think what another business does will 
matter. If anything, you d have to say that 
the new movie theatre would take business 
from ST, which would strengthen the authors 
claim that ST will fail. 



How would this change if ST expanded? That 
still depends upon whether they can get more 
people to come to the theatre, so this doesnt 
really tell me anything new. 



So the new people moving to town are people 
who arent likely to start going to ST. That 
strengthens the authors claim that STs ex- 
pansion is going to fail. Reverse Logic trap! 



Hmm. This basically means that the expan- 
sion would attract a greater audience — that 
helps! If they have more people, they can fill 
the larger theatre and make more money. 
This one is looking good. 



This is how they do things now. Would it stay 
the same, or change when they expand? I have 
no idea. This doesnt tell me that some new 
thing will happen that might make it more 
likely for the plan to succeed; it just talks 
about how things are already done. 



)ST exp unprof 



170 



GMAT 



Strengthen and Weaken 



Chapters 



3. Books and Coffee: The correct answer is D. 



Step 1; Identify the question. 

| Which of the following, if 
true, most weakens the owners' 
conclusion that a merger will 

! increase revenue? 



Step 2: Deconstruct the argument. 



The owners of a book store 



W A B C D E 
) merger -» T rev 



W 



A B C D E 



and a nearby coffee shop have © merger f fey (B + Q 
decided to combine their busi- 



nesses. 

Both owners believe that this 
merger will increase the number 
of customers and therefore the 
gross revenue, 

because customers who come for 
one reason may also decide to 
purchase something else. 



W A B C D E 
) merger t rev(B + C) 

W A B C D E 

)B + C merger -» t cust -» T 
rev 

Cust of each will buy > 



Step 3: State the Goal. 



The words "if true" and "weak- 
ens" tell me that this is a Weaken 
question. Further, I now know 
the conclusion: some merger will 
result in increased revenue. 



This is a fact; they have already 
made this decision, although it 
sounds like they haven't actually 
mergedyet. 

Here's the conclusion. This is the 
same thing the Q_ stem said: the 
merger will increase revenue. 



Okay, so I need to rewrite because 
they just inserted another step 
in the middle. The individual 
customers of each store will end 
up buying both books and coffee, 
so there'll be more customers for 
both, which means more rev- 
enue for both. That's assuming, 
of course, that these customers 
weren't already going to both 
stores to buy stuff. 



The owners think that merging will lead to increased revenue because it'll increase the number of customers 
and the customers will buy more stuff. This assumes that the same customers weren't already going to both 
stores and buying stuff. 

This is a Weaken question so I need to find something that will make the conclusion less likely to be valid. 



GMAT 



171 



Chapters 



Strengthen and Weaken 



Step 4: Work from wron g to rig ht. 




(A) Books and drinks 
can both be considered 
impulse purchases; of- 
ten, they are purchased 
by customers without 
forethought. 

(B) B Profit margins at a 
coffee shop are generally 
significantly higher than 
profit margins at a book 
store. 

(C) People who are able 
to read the first chapter 
of a book before buying 
are more likely to decide 
to buy the book. 

(D) A large majority of 
the book stores current 
customer base already 
frequents the coffee 
shop. 

(E) A combination book 
store and coffee shop 
that opened in a neigh- 
boring city last year has 
already earned higher 
than expected profits. 



W 



A B C D E 



)B + C merger -» t cust 
t rev 

Cust of each will buy > 



W AfiCDE 

S n 

)B + C merger -» t cust -» 
t rev 

Cust of each will buy > 
W A fi € D E 

S n S 

)B + C merger t cust -» 
T rev 

Cust of each will buy > 
W AfiCDE 

S n S W 

)B + C merger t cust -* 
t rev 

Cust of each will buy > 



W Afi€®£ 

S n S TP n 

)B + C merger -» t cust -» 
t rev 

Cust of each will buy > 



j This could be a reason why people would buy 

more. If they normally just buy coffee but 
I see a book they like, maybe they'll be more 
i likely to buy. That would strengthen the plan 
to merge, but I want to weaken the plan. 
Reverse Logic trap! 

1 That might make the coffee shop owner not 
want to merge, but it doesn't address the 
revenue side of the equation at all — and 
the conclusion has to do with revenues, not 
profits. 

\ This helps the owners' argument again! If I 
can sit there and read while having my coffee, 
| then I'm more likely to buy the book, which 
j would increase revenues. 



\ Let's see. Most of the people who shop at the 
book store also already go to the coffee shop. 
\ That's not so good for the owner's plan — it 
i means that they're not going to pick up 
i as many new customers as we might have 
thought before. 



Two problems here. One, we're not talking 
j about the same book store and coffee shop. 
\ Two, this choice talks about profits, not 



revenues. 



4. Digital Coupons: The correct answer is B. 
Step 1: Identify the question. 



Which of the following, if true, 
does the most to support the 
claim that the digital divide is 
responsible for the lower usage 
of electronic coupons? 



S A B C D E 
©DD -> I use e-coup 



| The language <( if true" and "sup- 
port the claim" tell me that this 
\ is a Strengthen question. The 
) question also indicates the conclu- 
| sion: something called the "digital 
j divide" causes electronic coupons 
1 not to be used as much. 



172 



GMAT 



Strengthen and Weaken 



Chapters 



Step 2: Deconstruct the argument. 



The redemption rate for e- 
mailed coupons is far lower than 
that for traditionally distributed 
paper coupons. 
One factor is the "digital di- 
vide" — those who might benefit 
the most from using coupons, 
such as homemakers, the elderly, 
and those in low-income house- 
holds, often do not have the 
knowledge or equipment neces- 
sary to go online and receive 
coupons. 

Step 3: State the Goal. 



S A B C D E 
)DD -» I use e-coup 
e-C ii use than PC 
S A B C D E 

)DD -» I use e-coup 
e-C ii use than PC 



This is a fact. For some reason, 
e-mailed coupons don t get used as 
much as paper coupons. 



Okay, so the people who would 
typically use Cs are less likely to 
be able to get them electronical- 
ly — they have to use the paper Cs 
DD: ppl who use Cs cant get instead. This doesn't really articu- 
them online ^ ate ^ e conclusion that well — the 

question stem did, so I'm going to 
add something here 



The author claims that the "digital divide" (DD) causes lower use of the e-coupons because people who use 
coupons aren't as likely to have access to e-coupons. 

I need to find something that makes this a little more likely to be true. 

Step 4: Work from wrong to right. 



(A) Computers are avail- 
able for free in libraries, 
schools, and community 
centers. 



(B) The redemption rate 
of ordinary coupons is 
particularly high among 
elderly and low income 
people that do not know 
how to use computers. 



S A B C D E 

w 

©DD -* I use e-coup 

e-C i i use than PC 

DD: ppl who use Cs can't 
get them online 

S A B C D E 

w s 

DD I use e-coup 

e-C 1 1 use than PC 

DD: ppl who use Cs cant 
get them online 



If this is true, then people who don't have 
computers can still use them. Maybe they 
could even take classes to learn how to use 
them! If anything, this weakens the author's 
claim. 

At first glance, I thought, "This is just saying 
what the argument already said, which is 
weird because usually they don't do that. " 
Then I realized that there was a gap in the 
argument! The argument only says that these 
people without computers are the ones who 
would "benefit the most" from coupons, but 
it doesn't say that these people actually do use 
coupons more. This choice tells me that; this 
strengthens the conclusion. 



GMAT 



173 



Chapter 5 



Strengthen and Weaken 




(C) Many homes, in- 
cluding those of elderly 
and low income people, 
do not have high-speed 
internet connections. 

(D) More homemak- 
ers than elderly people 
would use computers if 
they had access to them. 



(E) The redemption rate 
for coupons found on 
the internet has risen in 
the last five years. 



A B € D E 

W S n 

DD -> I use e-coup 

e-C H use than PC 

DD: ppl who use Cs can t 
get them online 
S A B € B E 

W S n n 

©DD -* I use e-coup 

e-C H use than PC 

DD: ppl who use Cs cant 
get them online 
1 S A®€ & E 

DD -» I use e-coup 

e-C ii use than PC 

DD: ppl who use Cs cant 
get them online 



The argument doesn 't say that people have to 
have high-speed connections in order to get 
coupons. The issue was whether these groups 
had internet access at all, not how fast the 
internet access is. 

The argument doesn V make any distinction 
between homemakers and the elderly; rather, 
they re both equally part of the group of people I 
without easy access to the internet. This is 
irrelevant. 

This means that more people are using 
electronic coupons today, but the argument 
doesn t claim that people arent. Instead, it 
talks about the fact that paper coupons are 
still in wider use because some people find it 
harder to access the electronic coupons. This 
answer does nothing to affect the conclusion. 



5. Teacher Compensation: The correct answer is E. 
Step 1: Identify the question. 



Which of the following, if true, 
most weakens the argument of 
the educational experts ? 



W 



A B C D E 



The language "if true" and 
"weakens" tells me this is a 
Weaken question. In addition, 
the question tells me that I need 
to look for a reference to "educa- 
tional experts" because whatever 
they claim is the conclusion. 



Step 2: Deconstruct the argument. 

Traditionally, public school 
instructors have been compen- 
sated according to seniority. 
Recently, educational experts 
have criticized the system as 
one that rewards lackadaisical 
teaching and reduces motivation 
to excel. 



W A B C D E 
Old: PST comp by sen. 

W ABCDE 
Old: PST comp by sen. 
EE: i> motiv 



Fact. Teachers were getting paid 
based upon how long they've 
worked. 

I guess the EEs are implying that 
teachers dont have to feel moti- 
vated to work hard because they 
know they'll make more money 
regardless. 



174 



GMAT 



Strengthen and Weaken 



Chapters 



w A b c D E 

Old: PST comp by sen. 

EE: 4 motiv 

)EE: base comp on perf 
keep good Ts, good instr. 



So the EEs want to base com- 
pensation on performance, and 
they claim this will lead to better 
teachers and instruction. 



Instead, these experts argue 
that, to retain exceptional 
teachers and maintain qual- 
ity instruction, teachers should 
receive salaries or bonuses based 
on performance rather than 
seniority. 

Step 3: State the Goal. 



Teachers normally get paid based on seniority, but the EEs want them to be paid based on performance be- 
cause they say that the teachers will then be better. 

I need to find something that weakens this plan at least a little bit. 

Step 4: Work from wron g to rig ht. 



(A) Some teachers 
express that financial 
compensation is not the 
only factor contributing 
to job satisfaction and 
teaching performance. 

(B) School districts 
will develop their own 
unique compensation 
structures that may dif- 
fer greatly from those of 
other school districts. 

(C) Upon leaving the 
teaching profession, 
many young, effective 
teachers cite a lack of 
opportunity for more 
rapid financial advance- 
ment as a primary 
factor in the decision to 
change careers. 



W A B C D E 

n 

Old: PST comp by sen. 

EE: 4 motiv 

©EE: base comp on perf 
keep good Ts, good instr. 

W A B C D E 

n n 

Old: PST comp by sen. 

EE: I motiv 

©EE: base comp on perf 
keep good Ts, good instr. 
W A B € D E 

n n S 

Old: PST comp by sen. 

EE: I motiv 

©EE: base comp on perf 
keep good Ts, good instr. 



So maybe we should also consider other ways 
to reward good teachers too, but as long as 
financial compensation is a factor, then tying 
compensation to performance might be a good 
plan. According to this answer, financial 
compensation is a factor (though not the only 
one). 

The argument isn't claiming that every school 
district has to be identical. It just makes a 
recommendation that compensation be tied to 
performance in general. 



If anything, I think this would strengthen 
the EEs claim! It shows that teachers do care 
about the financial side of things and causes 
some good teachers to leave the profession at a 
young age. 



mm 



MANHATTAN 175 
GMAT 



Chapters 



Strengthen and Weaken 




(D) In school districts 
that have implemented 
pay for performance 
compensation struc- 
tures, standardized test 
scores have dramatically 
increased. 

(E) A merit-based sys- 
tem that bases com- 
pensation on teacher 
performance reduces 
collaboration, which is 
an integral component 
of quality instruction. 



W A B € & E 

n n S S 

Old: PST comp by sen. 

EE: I motiv 

©EE: base comp on perf -» 
keep good Ts, good instr. 

W A B € © 

n n S S 

Old: PST comp by sen. 

EE: I motiv 

©EE: base comp on perf 
keep good Ts, good instr. 



Again, if anything, this makes the EEs' plan 
sound better. Students in the school districts 
that have already followed the EEs' recom- 
mendation are doing better on tests! 



The EE plan has a drawback: it reduces 
something that is considered an <( integral 
component" of good teaching If that's true, it 
could hurt the idea that basing compensation 
on performance will result in maintaining 
good instruction. 



6. The Gold Standard: The correct answer is C. 



Step 1: Identify the question. 

Which of the following, if it 
occurred, would cast the most 
serious doubt on the claim that 
The Gold Standard sneaker will 
be profitable ? 



W A B C D E 
©TGS will be prof 



Step 2: Deconstruct the argument. 

Brand X designs and builds W A B C D E 

custom sneakers, one sneaker at ^ will be prof 
a time. 



It recently announced plans 
to sell "The Gold Standard," a 
sneaker that will cost five times 
more to manufacture than any 
other sneaker that has ever been 
created. 



W A B C D E 

©TGS will be prof 

will sell TGS, 5x more to 
make 



This one is a little unusual. 
Instead of "if true, " it says "if it 
occurred " — but that's a synonym 
of "if true." This language, in ad- 
dition to "serious doubt," indicates 
that this is a Weaken question. 
Further, the claim is that TGS 
will be profitable. 



This is just a fact. I'm not even 
sure I need to write it down — it's 
just telling me what kind of com- 
pany Brand X is. 
It costs five times as much to 
make? Wow. And yet the claim is 
that they'll be profitable. There 
are a LOT of assumptions going 
on here. 



MANHATTAN 
GMAT 



Strengthen and Weaken 



Chapters 



Step 3: State the Goal. 

The claim is that TGS will be profitable and the only piece of "evidence" says that the sneakers are ridiculous- 
ly expensive to make. So there's no real evidence to support the idea that these will be profitable! Profitability 
is based on revenues minus costs, so the author is assuming that they can sell the sneakers for even more than it 
costs to make them. 



I'm looking for an answer that makes the conclusion less likely to be true. 
Step 4: Work from wrong to right. 



(A) The endorsement of W A B C D E 
The Gold Standard by a © TGS J { be prof 
popular celebrity 

will sell TGS, 5x more to 



make 
W 



AfiCDE 

s s 



(B) The publication of a 
report indicating that all q tg$ ^ ^ prof 
previous sneaker lines 
launched by Brand X 
have been profitable 



will sell TGS, 5x more to 
make 



(C) A significant in- 
crease in the cost of the 
canvas used to construct 
The Gold Standard 



(D) The introduction 
of another new sneaker 
line by a rival manufac- 
turer 



AfiCDE 

s s w 



W 

©TGS will be prof 

will sell TGS, 5x more to 
make 



W 

©TGS will be prof 



AfiCBE 

S S W n 



will sell TGS, 5x more to 
make 



Endorsements usually help companies sell 
more of a product, so this would help the 
claim that TGS will be profitable. Reverse 
logic! 

Really, I think this has nothing to do with 
the given conclusion; there's no reason why 
TGS must be profitable just because other 
Brand X sneakers were profitable — especially 
because TGS sneakers cost so much to make. 
If anything, this answer makes it more likely 
that TGS will be profitable, too. 
Lets imagine that TGS is profitable when 
the costs are five times higher. This choice is 
saying that the costs are going to go up even 
more . Profits equal revenues minus costs, so 
increasing the per-product cost makes it less 
likely that the product can be profitable. 
This one is so tempting! If another sneaker be- 
comes popular, perhaps the sales of TGS will 
drop. That doesn't necessarily affect whether 
TGS will be profitable, though. Again, profits 
equal revenues minus costs. But if you make 
fewer sneakers, then costs will go down as 
well — so its not necessarily the case that 
reducing revenues will also impact the level of 
profitability. 




MANHATTAN 
GMAT 



Chapters 



Strengthen and Weaken 




(E) An announcement 
by Brand X that The 
Gold Standard will be 
marketed as an exclusive 
offering, available only 
in limited quantities 



W A B ©B E 

S S W n n 

©TGS will be prof 

will sell TGS, 5x more to 
make 



This doesn't seem to tell me much one way 
I or the other. If anything, I think you could 
speculate that an "exclusive" product could 
| sell for a premium price, so maybe that would 
1 make it more likely that TGS will be profit- 
able. 



7. Machu Picchu: The correct answer is A. 
Step 1: Identify the question. 

Which of the following, if true, S 
most strengthens the argument 
above ? 



Step 2: Deconstruct the argument. 

In 2001 the Peruvian govern- 
ment began requiring tourists 
to buy permits to hike the Inca 
Trail to the ancient city of Ma- 
chu Picchu. 

Only 500 people per day are 
now allowed to hike the Inca 
Trail, whereas before 2001 
daily visitors numbered in the 
thousands. 

The Peruvian government 
claims that this permit pro- 
gram has successfully prevented 
deterioration of archaeological 
treasures along the Inca Trail. ( 

Step 3: State the Goal. 



A B C D E 



I The words "if true" and 
j "strengthens the argument" 
j indicate that this is a Strengthen 
1 question. 



S A B C D E 
01 PG: perm to hike IT 

S A B C D E 
01 PG: perm to hike IT 
now: 500 ppl / d (old = 000 s) 



This is a fact. People now have to 
pay to hike the Inca Trail. 



| More facts. Now, only 500 people 
a day can go; before, there were 
thousands a day. 



S A B C D E 
01 PG: perm to hike IT 



Here's the claim: the PG specifi- 
cally says that the permit program 
j is responsible for preventing 
now: 500 ppl / d (old = 000 s) | deterioration. 



)PG: perm -» A deter of IT 



The PG claims that its permit program has been responsible for preventing deterioration along the IT. The 
only thing we know about the permit program is that it has reduced the number of people who can visit the 
IT. So the government is assuming that reducing the number of visitors was the cause, and that if the permit 
program hadnt been in place, then there would have been deterioration. 



I need to find something that makes this a little more likely to be valid. 



MANHATTAN 
GMAT 



Strengthen and Weaken 



Chapter 5 



Step 4: Work from wrong to right. 



S A B C D E 

s 



(A) Since 2001, Incan 
ruins similar to Machu 
Picchu but without a 
visitor limit have disin- 
tegrated at a significant- 
ly greater rate than those © PG: perm _> j, deter of IT 
on the Inca Trail. 

(B) Villages near Machu 
Picchu have experienced 
declines in income, as 
fewer tourists buy fewer 
craft goods and refresh- 
ments. 



01 PG: perm to hike IT 

now: 500 ppl / d (old = 
000's) 



S AfiCDE 

S n 

01 PG: perm to hike IT 

now: 500 ppl / d (old = 
000's) 



(C) Many of the funds 
from the sale of Inca 
Trail permits are used to 
hire guards for archaeo- 
logical sites without 
permit programs. 

(D) Since 2001, tourist 
guides along the Inca 
Trail have received 50% 
to 100% increases in 
take-home pay. 

(E) The total number of 
tourists in Peru has risen 
substantially since 2001, 
even as the number 

of tourists hiking the 
Inca Trail has remained 
constant. 



)PG: perm -» I deter of IT 
S A B € D E 

S n n 

01 PG: perm to hike IT 

now: 500 ppl / d (old = 
000s) 

)PG: perm I deter of IT 
S A fi € B E 

S n n n 

01 PG: perm to hike IT 

now: 500 ppl / d (old = 
000's) 

)PG: perm-> 4- deter of IT 
S ®B € © £ 

;5 n n n n 

01 PG: perm to hike IT 

now: 500 ppl / d (old = 
000's) 

)PG: perm-» I deter of IT 



This sounds promising. The government s 
assumption was that the visitor limit helped 
prevent deterioration, so showing that other 
sites without limits did experience deterio- 
ration would make it more likely that the 
governments reasoning is valid. V 11 definitely 
keep this one in. 

This sounds bad for the villages, but it doesnt 
impact the specific claim about preventing 
deterioration along the IT 



This sounds like a good use of funds, but it 
has nothing to do with whether the permit 
program really did help prevent deterioration. 
All this tells us is that maybe other sites are 
also better protected due to the guards. 

Thats great for the guides. It doesnt impact 
the actual conclusion at all, though. 



This ones about the number of visitors again, 
so maybe it strengthens. Lets see. A lot more 
people are visiting Peru. . . oh, but the second 
part is what we were already told: visitors to 
the IT are limited. This doesnt add anything 
new that specifically affects the claim about 
deterioration along the IT 




8. Ethanol: The correct answer is B. 



Did this one seem a little different from all of the others? We set a trap for you! This is an Evaluate 
question, not a Strengthen or a Weaken. We discussed Evaluate questions in the previous chapter. (And 
we did warn you at the beginning of this chapter to read the last chapter first!) 

On the real test, you'll never have the luxury of knowing that the next question will be a certain 
type — so be prepared for anything 



MANHATTAN 
GMAT 



Chapter 5 



Strengthen and Weaken 



Step 1: Identify the question. 




In evaluating the recommen- 
dation to increase the use of 
ethanol, it would be important 
to research all of the following 
EXCEPT : 



EvEx 



A B C D E 



Step 2: Deconstruct the argument. 

Ethanol, a fuel derived from 
corn, can be used alone to pow- 
er cars or along with gasoline to 
reduce the reduce the amount of 
gas consumed. 

Unlike gasoline, ethanol is easily 
renewable since it is primarily 
converted from the sun s energy. 



Moreover, compared with con- 
ventional gasoline, pure ethanol 
is a cleaner-burning fuel. 



To save energy and reduce pollu- 
tion, many individuals advocate 
the increased usage of ethanol 
as a primary fuel source in 
conjunction with or in place of 
gasoline. 

Step 3: State the Goal. 



EvEx A B C D E 
E: fuel, use alone or w/gas 
4 use of FF 

EvEx A B C D E 

E: fuel, use alone or w/gas 

I use of FF 

easy to get > E 

EvEx A B C D E 

E: fuel, use alone or w/gas 

I use of FF 

easy to get > E, clean burn 
EvEx A B C D E 

E: fuel, use alone or w/gas 

4 use of FF 

easy to get > E, clean burn 
)use E -> save NRG, I poll 



The word "evaluating" (the 
conclusion) tells me that this is 
an Evaluate question. Its also an 
Except question. The four wrong 
ones WILL be important to evalu- 
ate; the correct answer will NOT 
be important to evaluate. The 
conclusion will have something to 
do with using ethanol. 



All facts. This E stuff is a kind of 
fuel, and it can be used in cars, 
either alone or with gas. 



Interesting. It's easier to get more 
ethanol than more gasoline. 



And ethanol is "cleaner-burning. " 
This E stuff sounds pretty good so 
far. 



Conclusion! People think that us- 
ing E will save NRG and reduce 
pollution. (Note: NRG is an ab- 
breviation for energy.) 



The E fuel has various good qualities, so people say we should use it and we'll save energy and reduce pollu- 
tion. 

On regular Evaluate questions, we try to find an answer that will tell us whether the conclusion is more or 
less valid. The answer can take us down two "paths," one of which will make the conclusion a bit better and 



MANHATTAN 
GMAT 



Strengthen and Weaken 



Chapter 5 



the other of which will make it a bit worse. On this EXCEPT question, all four wrong answers will do this. 
I'm looking for the "odd one out" that does NOT do this. 



Step 4: Work from wrong to right. 



(A) Whether the energy- 
required to grow and 
process corn used as 
fuel is greater than the 
amount of energy ulti- 
mately produced 



(B) Whether more 
energy is saved when us- 
ing ethanol in conjunc- 
tion with or in place of 
gasoline 



(C) Whether ethanol 
is as efficient a fuel as 
gasoline 



EvEx A B C D E 
E: fuel, use alone or w/gas 
I use of FF 

easy to get > E, clean burn 
©use E ^ save NRG, I poll 

EvEx A B C D E 
E: fuel, use alone or w/gas 
I use of FF 

easy to get > E, clean burn 
©use E -> save NRG, i poll 



EvEx A B € D E 
E: fuel, use alone or w/gas 
I use of FF 

easy to get > E, clean burn 
)use E -» save NRG, I poll 



(D) Whether it is pos- 
sible to produce more 
ethanol than is currently 
produced 



EvEx AB6BE 
E: fuel, use alone or w/gas 
i use of FF 

easy to get > E, clean burn 
)use E -» save NRG, I poll 



The conclusion specifically claims that we'll 
save energy. If the amount of energy to pro- 
duce ethanol is MORE than the amount of 
energy produced, then we ar en t saving ener- 
gy. If the amount of energy to produce ethanol 
is LESS than the amount of energy produced, 
then we are saving energy. This answer gives 
me "two paths" so it's wrong (since I want the 
EXCEPT answer). 

This answer choice uses many of the same 
words as the conclusion. But that's a trap! 
The conclusion makes no distinction between 
these two methods of using ethanol; it just 
recommends in general that we do use etha- 
nol. If more energy is saved using ethanol in 
conjunction with gasoline, then the conclusion 
holds. If more energy is saved using ethanol 
in place of gasoline, then the conclusion holds. 
Either way, it's the same thing! There aren't 
"two paths" here. 

If ethanol is as efficient as or more efficient 
than gasoline, then we could use less ethanol 
to get the same amount of power. That would 
save energy, making the conclusion a bit 
stronger. If ethanol is less efficient than gas, 
then we would have to use more ethanol to get 
the same amount of power. That might mean 
it takes more energy for the car to go the same 
distance, making the conclusion weaker. We 
have "two paths" here. 
The conclusion says we should "increase" the 
usage of ethanol. But is more ethanol avail- 
able to use? If we can produce more ethanol, 
then that makes the argument a bit stronger. 
If we cannot produce any more ethanol, then 
how can we increase the usage? That would 
make the argument weaker. 




MANHATTAN 
GMAT 



181 



er 5 



Strengthen and Weaken 



(E) Whether the process 
of growing corn for fuel 
would result in as much 
pollution as does the 
production of conven- 
tional gasoline 



EvEx A(B)€ B £ 
E: fuel, use alone or w/gas 
I use of FF 

easy to get > E, clean burn 
)use E -> save NRG, I poll 



The conclusion claims that using ethanol will 
reduce pollution, but the argument tells us 
only that ethanol burns more cleanly than 
gas. If the process of making ethanol results 
in less pollution, this would be another point 
in favor of the conclusion. If the process of 
making ethanol results in more pollution than 
does the production of gasoline, however, then 
this would weaken the conclusion. 



9. APR: The correct answer is E. 
Step 1: Identify the question. 

Which of the following state- 
ments, if true, would most 
seriously undermine a plan to 
increase interest rates in order to 
spur profitable growth? 



Step 2: Deconstruct the argument. 

CEO: Over the past sev- 
eral years, we have more than 
doubled our revenues but profits 
have steadily declined because 
an increasing number of cus- 
tomers have failed to pay their 
balances. 

In order to compensate for these 
higher default rates, we will 
increase the interest charged on 
outstanding balances from an 
annual percentage rate (APR) of 
9.5% to an APR of 12%. 



W A B C D E 
) t int rate -» t prof growth 



W ABODE 

© t int rate t prof growth 

C: 2x rev but l prof b/c cust 
not pay bills 

W ABODE 

© t int rate -» t prof growth 

C: 2x rev but I prof b/c cust 
not pay bills 

t % to comp 



The "if true" and "undermine" 
language indicate that this is a 
Weaken question. Further, the 
question stem tells me the conclu- 
sion: there's a plan to increase 
interest rates and that is then sup- 
posed to cause profits to grow. 



Several facts here. Revenues have 
gone up but profits have gone 
down because the customers aren't 
paying what they owe. 



Okay, here's the plan. They'll 
charge more interest to everyone 
to compensate for the people who 
aren't paying their bills. 



MANHATTAN 
GMAT 



Strengthen and Weaken 



This increase will be sufficient 
to compensate for the current 
j rate of defaults and allow us to 
increase our profits. 



W A B C D E Hmm. They're claiming that 12% 

. _^ ^ r i will be enough to compensate for 

T int rate -» T prof growth & r j 

the current rate of people who 
C: 2x rev but I prof b/c cust don't pay so that they can increase 
not pay bills profits (which is the conclusion I 

T % to comp; 12% is suff for already wrote down). They're as- 
now default suming that the current rate isn't 

going to get worse in future. 



Step 3: State the Goal. 



The company plans to charge higher interest rates in order to become profitable again. The evidence shows 
only that the higher interest rate will be sufficient for today's default rate; that could change over time. 

This is a Weaken question so I need to find something that makes the CEO's conclusion a bit less likely to be 
valid. 



Step 4: Work from wrong to right. 



(A) Many other compa- 
nies have experienced 
a similar trend in their 
default rates. 



W 



A B C D E 



(B) The company's 
operating expenses 
are above the industry 
average and can be sub- 
stantially reduced, thus 
increasing profits. 



© t int rate -» T prof 
growth 

C: 2x rev but 4 prof b/c 
cust not pay bills 

t % to comp; 12% is suff 

for now default 

W ABCDE 

n n 

© t int rate -» t prof 
growth 

C: 2x rev but I prof b/c 
cust not pay bills 

t % to comp; 12% is suff 
for now default 



This doesn't address the company's plan to fix 
the problem: increasing the interest rate. This 
doesn't impact the conclusion at all. 



Hmm. If the company does this, it could in- 
crease profits, which is the company's goal. . . 
but the conclusion is that the plan to increase 
interest rates will improve profits, and this 
choice doesn't address that plan. Plus, if any- 
thing, this choice makes it more likely that the 
company will increase profits, but we want to 
weaken the conclusion. 



Chapter 5 



Strengthen and Weaken 




(C) The increase in 
default rates was due to 
a rise in unemployment, 
but unemployment rates 
are expected to drop in 
the coming months. 



(D) The proposed in- 
crease in the APR will, 
alone, more than double 
the company's profit 
margins. 



(E) An increase in the 
APR charged on credit 
card balances often 
results in higher rates of 
default. 



W A fi € D E 

n n n 

) t int rate -» t prof 
growth 

C: 2x rev but I prof b/c 
cust not pay bills 

t % to comp; 12% is suff 

for now default 

W A fi € B E 

n n n S 

) t int rate -* t prof 
growth 

C: 2x rev but 4 prof b/c 
cust not pay bills 

t % to comp; 12% is suff 
for now default 
W A fi € B 

n n n S 

) t int rate t prof 
growth 

C: 2x rev but 4 prof b/c 
cust not pay bills 

t % to comp; 12% is suff 
for now default 



10. Jupiter vs. Mars: The correct answer is D. 
Step 1: Identify the question. 



| If unemployment caused people not to pay 
| their bills, and fewer people are going to be 
j unemployed, then maybe more will pay their 
bills? That would help the company, but we 
want something that will weaken the conclu- 
sion. And the conclusion is specifically about 
the plan. This choice doesnt address the 
specific plan about interest rates. 

This supports the company s claim that 
increasing the interest rate will help raise 
| profits. I want something that weakens that 
claim. 



Okay, if they do increase the APR, then more 
people may stop paying their bills as a result! 
The conclusion specifically said that raising 
the APR would compensate for the "current 
rate of defaults," so if the rate goes up, then 
the company is less likely to increase its profits. 
This does weaken the conclusion. 



Which of the following, if true, 
would most strengthen a con- 
tention by the government that 
the new project is a better use of 
its funds? 



S A B C D E 
)G: new proj = better 



j The words "if true" and 

I "strengthen a contention" indicate 
that this is a Strengthen question. 
Further, the question stem tells 
me the conclusion: the government \ 
claims that the new project is a 
better use of funds. 



MANHATTAN 
GMAT 



Strengthen and Weaken 



Step 2: Deconstruct the argument. 

Scientists suspect that Europa, 
a moon orbiting Jupiter, may 
contain living organisms. 

However, the government 
recently scrapped an unmanned 
science mission to Europa and 
replaced it with a project aimed 
at landing an astronaut on 
Mars. 

Polls show that the public is far 
more fascinated by space travel 
than by discovering life else- 
where in the universe. 



Critics argue that the govern- 
ment s decision-making process 
places a greater emphasis on 
popularity than it does on the 
importance of scientific re- 
search. 



S A B C D E 
) G: new proj = better 
S: E may have life 
S A B C D E 

)G: new proj = better 

S: E may have life 

cncl E project, rplc w/M 
project 

S A B C D E 

)G: new proj = better 

S: E may have life 

cncl E project, rplc w/M 
project 

ppl like spc trvl > 
S A B C D E 

) G: new proj = better 

S: E may have life 

cncl E project, rplc w/M 
project 

ppl like spc trvl 

C: govt cares popul > rsrch 



There is a fact: scientists sus- 
pect something is true. We dorit 
actually know whether its true, 
though. 

There was a project to send an 
unmanned mission to E, but that 
was replaced by another project to 
send a person to Mars. More facts. 



More facts — a survey showed that 
people like space travel more. 



This is a counter-conclusion. The 
critics say that the government is 
just paying attention to popular- 
ity of projects, but the question 
stem told me that the government 
claims that the new project is a 
better use of funds. 



Step 3: State the Goal. 

There are two opposing points of view, the government and the critics. The government claims that the new 
project is a better use of funds. The critics claim that the government is paying more attention to popularity 
than to scientific research. The critics are assuming that, just because the public finds the Mars project more 
interesting, there arent also good scientific reasons for replacing the Europa project with the Mars project. 

I need to strengthen the government s claim. I need to be really careful that I don t mistakenly strengthen the 
critics* claim. 



Strengthen and Weaken 



Step 4: Work from wrong to rig ht. 



(A) In the first year of 
the project, the govern- 
ment will spend 30% 
of its total budget on de- 
veloping a space shuttle 
that can travel to Mars; 
that figure is expected 
to drop to 0% after five 
years. 

(B) The government 
cannot be absolutely 
certain of the chances 
for success of either 
project. 



(C) Some scientists are 
convinced that a mis- 
sion to Europa would 
add immeasurably to 
our understanding of 
the universe. 



(D) A new telescope 
that has just become 
available to scientists 
promises to yield more 
information than the 
planned mission to 
Europa was designed to 
provide. 



S A B C D E 

n 

^)G: new proj = better 

S: E may have life 

cncl E project, rplc w/M 
project 

ppl like spc trvl 

C: govt cares popul > 
rsrch 

S ABC D E 

n n 

©G: new proj = better 

S: E may have life 

cncl E project, rplc w/M 
project 

ppl like spc trvl 

C: govt cares popul > 
rsrch 

t S A B € D E 

n n W 

©G : new proj = better 

S: E may have life 

cncl E project, rplc w/M 
project 

ppl like spc trvl 

C: govt cares popul > 
rsrch 

S A B € D E 

n n W S 

©G: new proj = better 

S: E may have life 

cncl E project, rplc w/M 
project 

ppl like spc trvl 

C: govt cares popul > 
rsrch 



This doesn't give us any additional informa- 
tion as to why the Mars project is better than 
the Europa project. We don t know whether 
i they'd be spending more or less on the E 
project, nor do we know what kind of good 
research they'll expect to get in return. 



Was there anything in the argument that 

hinged on being absolutely certain of success? 
| No. If they told us that the Mars project has 

a greater chance for success, that would be 
\ good — but knowing that we don't know the 

chances for either project. . . that doesn't add 

anything. 



This is a good reason to continue funding the 
mission to E. But that would support the crit- 
ics, not the government. This is a trap. 



Now they have a new telescope that they can 
use to get even more research than they would 
have if they sent an unmanned mission? 
That's a good reason to cancel the unmanned 
mission. If that's true, then pretty much any 
other decent project would be a better use of 
funds! 



Strengthen and Weaken 



Chapter 5 



(E) Most people feel 
that a shuttle to Mars 
is the next logical step 
in the development of 
a system that will allow 
humans to travel even 
further in the solar 
system. 



S A fi €® £ 

)G: new proj = better 

S: E may have life 

cncl E project, rplc w/M 
project 

ppl like spc trvl 

C: govt cares popul > 
rsrch 



That's interesting, but it doesn't tell us 
anything new about why spending the money 
on the Mars project is better than spending 
money on the Europa project. We already 
know that people are more interested in space 
travel. This answer is a tangent that's trying 
to get us to think more about that and make 
us forget about the conclusion, which centered 
on a comparison of the two projects. 



MANHATTAN 
GMAT 



Chapter 




Critical Reasoning 



Evidence Family 



-pm 

C/f BBS 



What Are Inferences? 
Percentages vs. Real Numbers 
Inference Questions 
Explain a Discrepancy 
EXCEPT Questions 



Evidence Family 



The Evidence Family of questions is our third main family. Here's a recap of what we discussed in 
Chapter 2: 

• No Conclusions! These are made up entirely of premises 

• No Assumptions either! 

• Two main question types: Inference and Explain a Discrepancy 

Inference questions require us to find an answer that must be true according to the premises given in the 
argument. 

Explain a Discrepancy questions require us to identify some kind of paradox or puzzling result in an 
argument and find an answer that explains, or resolves, the puzzling part of the argument. Before we 
delve further into each type, let's talk about what inferences are in general. 

What Are Inferences? 

In order to answer inference questions accurately, we first need to understand what the GMAT test 
writers mean when they ask us to infer something. In the GMAT World, an inference is something that 
must be true according to the evidence given in the argument. In the Real World, by contrast, we don't 
think of inferences in this way; rather, in the Real World, inferences are likely to be true based on the 
available evidence, but they dont absolutely have to be true. 

For example, if a friend tells you that chocolate is her favorite flavor of ice cream, what kind of real- 
world inferences might you make? 

You might infer that she likes chocolate in general and that she likes ice cream in general. Maybe she 
likes all desserts in general — perhaps she has a sweet tooth. All of these things are perfectly reasonable 
to infer in a "likely to be true" situation, but not a single one has to be true. Its possible that she really 

191 



Evidence Family 



likes chocolate only when it's in the form of ice cream, and she really likes ice cream only when its 
chocolate. The kinds of answers discussed in this paragraph would be tempting incorrect answers on the 
GMAT. 

What does have to be true? She can't like vanilla ice cream better than she likes chocolate ice cream — if 
chocolate is her favorite flavor of ice cream, then by definition she doesn't like any other flavor better. 
She has to have tried at least one other flavor of ice cream at some point in her life — she has to have 
had the ability to compare with at least one other flavor in order to decide that chocolate is her favorite 
flavor. These kinds of answers would be correct answers on the GMAT. 

From now on, when we discuss how to infer something, we'll be referring to the GMAT's definition: 
something that must be true based on the available evidence. 

Percentages vs. Real Numbers 

Let's say that we're discussing a company that sells only vanilla and chocolate ice cream. We're told 
that 55% of the company's profits last year were derived from chocolate ice cream sales and 40% of the 
revenues last year were derived from vanilla ice cream sales. What can we infer? 

Because we know that the company sells only these two products, we can figure out two additional 
numbers. If 55% of profits came from chocolate, then 45% of profits came from vanilla. If 40% of 
revenues came from vanilla, then 60% of revenues came from chocolate. These things must be true, but 
these inferences are probably too easy for any GMAT question. What else can we infer? 

The company earned 60% of its revenues, but only 55% of its profits, from chocolate. By contrast, the 
company earned 40% of its revenues and a higher percentage of its profits, 45%, from vanilla. That's 
interesting. We made more money on vanilla than we would have expected based on the percentage of 
revenues, and we made less money on chocolate than we would have expected based on the revenues. 
Profitability is a measure of profits versus revenues. The vanilla ice cream product is more profitable 
than the chocolate ice cream product. That must be true. 

What doesn't have to be true? It doesn't have to be true that vanilla will continue to be more profitable 
in the future. The trend might not continue in the future. 

Let's say there are two ice cream companies, X and Y. Chocolate ice cream represents 60% of Company 
X s sales and 50% of Company Y's sales. Clearly, then, Company Y sells more chocolate ice cream, 
right? 

Not necessarily. We have no information about real numbers here, only percentages, and we don't even 
know how those percentages relate to each other. What if company Y has $1 million in annual revenues 
and company X has only $10,000 in annual revenues? We can't conclude anything about actual dollar 
amounts from this information about percentages. 



Evidence Family 

Inference Questions 



Chapter 6 



Inference questions require us to find an answer that must be true according to the information in the 
argument. Most of the time, we will need to use only some of the information in the argument, though 
we may use all of it. 

Most Inference question stems contain some form of the word "conclude" or some form of the word 
"infer," though there are some variations that don't include those specific words. Examples of words or 
phrases contained in Inference questions include: 

• which answer can be " logically concluded" or the "statements above most strongly 
support which of the following conclusions"? 

• which answer can be "properly inferred"? 

• the statements above "best support" which of the following "assertions"? 

• which answer "must be true" based upon the above statements? 

Note that Inference question stems can contain the language "most strongly support." We also saw this 
language on Strengthen questions, so it is critical to ensure that we don't mix up the two question types. 

The below diagram shows the "direction" of the support. On Inference questions, the argument (above) 
is used to support the correct answer (below). On Strengthen questions, the correct answer (below) is 
used to support the conclusion of the argument (above): 

Argument 



Strengthen 




Strongly 
Supports 



Answers 




Inference 




Inference questions ask us to use the argument to support an answer (the answer concludes some- 
thing from the argument). By contrast, Strengthen questions ask us to use an answer to support the 

argument (strengthen the argument / conclusion). Strengthen questions will contain a conclusion in 
the argument or question stem; Inference arguments will not contain a conclusion in the argument or 
question stem. 



MANHATTAN 
GMAT 



Chapter 6 



Evidence Family 



Let's take a look at a short example. 

Both enrollment and total tuition revenue at Brownsville University have in- 
creased during each of the last four years. During the same period, enrollment 
at Canterbury University has steadily decreased, while total tuition revenue has 
remained constant. 

Which of the following hypotheses is best supported by the statement given? 

(A) Brownsville University now collects more total tuition revenue than does 
Canterbury University. 

(B) The per-student tuition at Canterbury University has risen over the last four 
years. 

(C) Brownsville University will continue to increase its revenues as long as it 
continues to increase enrollment. 

The question stem is one of the slightly different variations. It uses the word "hypotheses" instead of 
"conclusions," but it means the same thing: an inference question. Our core might look like this: 



Note that, on all inference questions, the right-hand side of the core will always look like the above. 
We'll only be given premises. 




4 yrs: 
BU: enrol, tuit t 
CU: enrol I, tuit = 
(premise) 




therefore 



[what must be true?] 



(inference) 



Hmm. There are two schools but different trends are happening. BUs enrollment and tuition revenues are 
both going up. CUs enrollment is going down, but tuition revenues are the same. 



This is an Inference question, so I have to find an answer that must be true according to my premises. 



(A) Brownsville University 
now collects more total tuition 
revenue than does Canterbury 
University. 



4 yrs: 



BU: enrol, tuit T 



ABC 



Things have certainly been looking up 
for BU lately, but I know absolutely 
nothing about the actual dollar values 
that the schools are collecting. It's en- 



CU: enrol i>, tuit = 



tirely possible that CU still collects more 
money than BU. 



MANHATTAN 
GMAT 



Evidence Family 



Chapter 6 



(B) The per-student tuition at 
Canterbury University has risen 
over the last four years. 



4 yrs: 



ABC 



Let's see. "Per-student tuition' '= rev- 
enues I # of students. CU has the same 
revenues today, so the numerator stays 
the same, but fewer students, so the 



BU: enrol, tuit t 



CU: enrol 4, tuit = 



denominator gets smaller. Dividing by 
a smaller number = a larger number. 
This must be true! Til check C, butFm 
feeling good about this one. 



(C) Brownsville University will 
continue to increase its rev- 
enues as long as it continues to 
increase enrollment. 



In A(B)€ 

4 yrs: 



This might be reasonable to believe in 
the real world, but it doesnt have to 
be true. A trend never absolutely has to 
continue in the future. 



BU: enrol, tuit t 



CU: enrol I, tuit = 



The argument provides us with several fact-based premises. (It is also possible to have premises that are 
somewhat more claim-based.) The correct answer is something that must be true based on those prem- 
ises, though note that, in this case, we only needed to use the information about Canterbury in order to 
draw the correct conclusion. Answer B didn't use the Brownsville data at all. 

Answer A tried to trap us by getting us to make a conclusion based on information we don't have (ac- 
tual dollar values). Answer C is a classic "real-world inference" trap — it might be reasonable to believe 
that the trend will continue, but nothing says that a trend must continue in the future. 

Let's try a full example; set your timer for 2 minutes: 

Curbing government spending has been demonstrated to raise the value of a 
country's currency over time. However, many economists no longer recommend 
this policy. A currency of lesser value causes a country's exports to be more com- 
petitive in the international market, encouraging domestic industries and making 
the economy more attractive to foreign investment. 

The statements above most strongly support which of the following inferences? 

(A) Limited government spending can also lead to a reduction in the national 
deficit. 

(B) Curbing government spending can make a country's exports less competi- 
tive. 

(C) Many economists now recommend higher levels of government spending. 

(D) An increase in the value of a currency will result in reduced government 
spending. 

(E) Competitive exports indicate a weak currency. 




MANHATTAN 
GMAT 



Evidence Family 



Step 1 : Identify the question. 



The statements above most 
strongly support which of the 
following inferences? 

Step 2: Deconstruct the argument 



In 



A B C D E 



; They re asking me to support 
something below (in the answers), \ 
and they use the word "inference. " j 
This is an Inference question. 



Curbing government spending 
has been demonstrated to raise 
the value of a country's currency 
over time. 

However, many economists no 
longer recommend this policy. 



A currency of lesser value causes 
a country's exports to be more 
competitive in the international 
market, encouraging domes- 
tic industries and making the 
economy more attractive to 
foreign investment. 



In A B C D E 
1 gov sp -» t val curr 

In A B C D E 
I gov sp -* t val curr 
BUT E don't rec 

In A B C D E 

I gov sp -* T val curr 

BUTE don't rec 

I val curr -> exp > comp 
benefits 



This is a fact. Fairly straight- 
forward — one thing leads to 
another. 

Hmm. According to the first 
sentence, raising the value of cur- 
rency sounds like a good thing, so 
why wouldn't the Es want us to 

j do that? 
Oh, okay, so there are some good 
reasons to have a lower currency 
value. I guess those economists 

\ think these benefits outweigh the 

j lower value. 



Step 3: State the Goal. 

Reducing government spending will increase currency value. It seems like it would be good to have a high 
currency value, but some Es say that we shouldn't do that, because there are other benefits involved in having 
a lower currency value. 

I need to find an answer that must be true given the information in the argument. I don't need to use all of 
the info in the argument, though I may. 



Step 4: Work from wrong to right. 

(A) Limited government In A B C D E 



spending can also lead 
to a reduction in the 
national deficit. 



i gov sp t val curr 

BUT E don't rec 

4 val curr exp > comp 
-> benefits 



Deficit? This might be reasonable to believe 
in the real world, but there was nothing 
! about the deficit in the argument — I don't 
have any evidence to support this statement. 



Evidence Family 



(B) Curbing government 
spending can make a 
country's exports less 
competitive. 



(C) Many economists 
now recommend higher 
levels of government 
spending. 



(D) An increase in the 
value of a currency will 
result in reduced gov- 
ernment spending. 



(E) Competitive exports 
indicate a weak cur- 
rency. 



In ABCDE 

4 gov sp -» t val curr 

BUT E don't rec 

4 val curr -» exp > comp 
-> benefits 

In ABCDE 

4 gov sp -» t val curr 

BUT E dont rec 

4 val curr exp > comp 
benefits 

In A B C B E 

4 gov sp 1 s val curr 

BUT E don t rec 

4 val curr -» exp > comp 
-» benefits 

In A B € & E 

4 gov sp t val curr 

BUT E don't rec 

4 val curr exp > comp 
-» benefits 



Z^i see. The author said that curbing spend- 
ing leads to a higher currency value. And 
then the Es said that a lower currency value 
makes exports more competitive. If that's true, 
then a higher currency value could make ex- 
ports less competitive. . . so, hey, it is actually 
the case that curbing spending might lead to 
less-competitive exports! Keep this one in. 
The argument says "many economists," and 
the answer says "many economists," so that 
part is okay. If you tell someone not to lower 
their spending, is that the same thing as 
telling them to increase their spending? No. 
You could also recommend spending the same 
amount. Tricky! This one isn't a "must be 
true" statement. 

This one feels similar to B — language pretty 
similar to the argument, and I have to figure 
out what leads to what. The author said that 
X (curbing spending) will lead to Y(a higher 
currency value). This answer reverses the 
direction: Ywill lead to X. That's not what 
the author said! 

The Es said that M (a lower currency value) 
leads to N (more competitive exports). The 
answer reverses the direction but changes the 
description a bit: N indicates that M is true 
or has happened. IfM leads to N, then isn't it 
the case that having N could indicate that M 
happened? Yes. Leave this one in. 



Now I need to compare B and E. I checked the logic on B and E, and they both seem good — in both cases, 
the causal direction could be true. Let me check the wording of the answers to make sure I'm reading them 
correctly. Oh, I see. B says that curbing spending "can make" exports less competitive, which is true, while E 
says that competitive exports "indicate" a weak currency. Answer E is missing the "can" or "could" part. It 
might be the case that competitive exports indicate a weak currency, but the argument never says that this is 
definitely the case. 



The correct answer is B. 



er 6 



Evidence Family 



Common Trap Answers 

The most tempting wrong answers on Inference questions tend to revolve around making Real World 
Inferences — things that we would reasonably assume to be true in the real world but don't absolutely 
have to be true. Some of these wrong answers may go way too far and be quite obviously out of scope, 
but the trickiest ones will seem very reasonable. . . until we ask ourselves whether that answer MUST be 
true. 

Answer choice E in our last problem did this, and so did answer choice C. The argument said merely 
that economists "no longer recommend" a policy to reduce spending. The trap answer said that the 
economists recommend higher spending. Many people in the real world might assume or infer this, but 
it doesn't have to be true! There's also a third option: maintaining the same level of spending. 

Other wrong answers will use language very similar to the language in the argument but will Reverse 
Direction or Switch Terms somehow. If we're told that eating honey causes people to hiccup, then a 
wrong answer might say that hiccupping causes people to eat honey! Alternatively, if we're told that the 
flu causes higher temperatures, then a wrong answer might say that the flu causes a fever. Higher tem- 
peratures and fevers are not interchangeable, but may seem to be if you're not reading carefully. Term 
switching is only acceptable if both terms are synonyms. 

Takeaways for Inference Questions 

Most of the time, inference questions will contain some form of the word "conclude" or "infer," though 
other variations are possible. Common synonyms include "assertion" or "hypothesis" in place of "con- 
clusion" or "must be true" in place of "infer." 

We have to be careful not to mix up Inference and Strengthen questions. Inference questions ask us to 
use the argument to support an answer. Strengthen questions ask us to use an answer to support the 
argument. 

Inference arguments will not contain conclusions or assumptions, so don't waste time trying to find 
conclusions or brainstorm assumptions! (And that lack of a conclusion is another way by which we can 
distinguish between Inference and Strengthen questions — Inference arguments never have conclusions, 
and Strengthen arguments always do.) 

The correct answer to an inference question must be true according to the information given in the 
argument. The correct answer does not have to use all of the information given in the argument. 

Trap answers will include Real World inferences — they're reasonable and could he true, but they don't 
have to be true. Inference questions also often include Reverse Direction or Switch Terms traps. These 
traps will often contain language that is very similar to the language in the argument, but the trap will 
mix up the order of what the argument actually said. 



MANHATTAN 
GMAT 



Evidence Family Chapter 6 

Explain a Discrepancy 

As was the case with Inference questions, Discrepancy questions consist only of premises, mostly on the 
fact-based side (though it is possibly to have more claim-like premises). There are no conclusions. Most 
of the time, two sets of premises will be presented, and those premises will be contradictory in some 
way. They won't "make sense" together. Sometimes, the argument will include indicator words such as 
surprisingly or yet. 

Most discrepancy question stems will include some form of the words "explain" or "resolve" and the vast 
majority will also contain the words "if true." Here are two typical examples: 

Which of the following, if true, most helps to resolve the paradox described 
above? 

Which of the following, if true, best explains the fact that many economists no 
longer recommend curbing spending in order to increase currency values? 

Our task on Discrepancy questions is to find an answer that resolves or fixes the discrepancy — that 
makes all of the information make sense together. If we leave the argument as is, people should say, 
"Wait. That doesn't make sense." If we insert the correct answer into the argument, people should say, 
"Oh, I see. That makes sense now." 

Take a look at this short example. 

According to researchers, low dosages of aspirin taken daily can significantly 
reduce the risk of heart attack or stroke. Yet doctors have stopped recommending 
daily aspirin for most patients. 

Which of the following, if true, most helps to explain why doctors no longer rec- 
ommend daily low dosages of aspirin? 

(A) Only a small percentage of patients have already experienced a heart attack 
or stroke. 

(B) Patients who are at low risk for heart attack or stroke are less likely to comply 
with a doctor's recommendation to take aspirin daily. 

(C) Aspirin acts as a blood thinner, which can lead to internal bleeding, particu- 
larly in the stomach or brain. 

The question stem asks us to "explain" something that doesn't make sense: the aspirin is apparently 
beneficial but "doctors have stopped recommending" its use for most people (implying that they did 
used to recommend it more). Why would they do that? We might sketch or think of our info visually in 
this way: 



MANHATTAN 
GMAT 



Chapter 6 



Evidence Family 



daily asp I 
hrt att / strk 




BUT 
WHY? 



Drs stop for most 



Note that, for Discrepancy questions, we didn't set up a core. Not only don't we have a conclusion, but 
were not even trying to find a conclusion (as we were on Inference questions). Were trying to find a 
third premise that will help these two facts to make sense together. In this case, what we want to do is 
highlight the apparent discrepancy between the two facts: on the one hand, daily aspirin is beneficial, 
and, on the other, doctors have stopped recommending it. 

Back to step 3: what's our goal? 

So far, they've told me something really good about taking aspirin daily: it significantly reduces the risk of 
some pretty bad things. The fact that the doctors have stopped recommending it means that they used to 
recommend it, so why would they stop doing so? Maybe there's something else that's bad about taking aspirin 
daily. 



(A) Only a small percent- 
age of patients have already 
experienced a heart attack 
or stroke. 



ED ABC 
daily asp 4 hrt att, strk 
BUT drs stop rec for most 



(B) Patients who are at 
low risk for heart attack 
or stroke are less likely to 
comply with a doctor's 
recommendation to take 
aspirin daily. 

(C) Aspirin acts as a blood 
thinner, which can lead to 
internal bleeding, par- 
ticularly in the stomach or 
brain. 



ED ABC 
daily asp 4 hrt att, strk 
BUT drs stop rec for most 



ED A fi© 
daily asp I hrt att, strk 
BUT drs stop rec for most 



So maybe this means the doctors think 
it wont help that many people? Wait. 
The purpose of taking the aspirin is to 
try to prevent a heart attack or stroke. If 
most people haven t had a heart attack or 
stroke, you'd want them to do something 
that would help lower the risk. 
I can believe this is true in the real world, 
but that doesn't affect a doctor's behavior. 
They don't say, "Oh, I know a lot of people 
won't take the life-saving medication prop- 
erly, so I just won't bother to prescribe it." 
Plus, why would they recommend aspirin 
to people who are at low risk? 
Oh, this is a bad thing about aspirin — it 
can cause you to bleed internally. The 
brain? Yeah, if it could make your brain 
start bleeding, I can imagine that doctors 
would want to avoid prescribing it unless 
there were a really good reason to do so. 



The fact that doctors once prescribed daily aspirin but mostly stopped is perplexing when all we're told 
is that daily aspirin does something good. Possibly that benefit doesn't apply to most people so the doc- 
tors don't waste time recommending the daily treatment. Alternatively, maybe there's some other bad 



200 



GMAT 



Evidence Family 



thing going on to make the doctors stop prescribing daily aspirin. In this case, answer C gives us that 



Answer A tries to get us to think about the first possibility: maybe it doesn't really help very many 
people, so the doctors don't bother. However, answer A limits the group to those who have already had 
a heart attack or stroke — but the argument is not limited to that group. Answer A does nothing to 
explain why the doctors stopped prescribing the treatment to most people. 

Answer B again tries to distract us: patients wouldn't benefit if they didn't actually take the medication. 
That's true, but this doesn't explain why doctors would stop recommending aspirin. In addition, this 
choice limits itself to those who are at low risk for heart attack or stroke — why would doctors need to 
recommend daily aspirin for a group that doesn't have the risk factors? 

Let's try this again, this time with a full example: 

In a recent poll, 71% of respondents reported that they cast votes in the most 
recent national election. Voting records show, however, that only 60% of eligible 
voters actually voted in that election. 

Which of the following pieces of evidence, if true, would provide the best expla- 
nation for the discrepancy? 

(A) The margin of error for the survey was plus or minus five percentage points. 

(B) Fifteen percent of the survey's respondents were living overseas at the time 
of the election. 

(C) Prior research has shown that people who actually do vote are also more 
likely to respond to polls than those who do not vote. 

(D) Some people who intend to vote are prevented from doing so by last-min- 
ute conflicts on election day or other complications. 

(E) People are less likely to respond to a voting poll on the same day that they 



bad thing. 



voted. 



Step 1: Identify the question. 



Which of the following pieces of ED A B C D E 
evidence, if true, would provide 
the best explanation for the 
discrepancy ? 



The question stem uses the word 
"explanation" and explicitly men- 
tions a " discrepancy," so this is an 
Explain the Discrepancy question. 



Chapter 6 



Evidence Family 



Step 2: Deconstruct the argument. 



In a recent poll, 71% of respon- 
dents reported that they cast 
votes in the most recent national 
election. 

Voting records show, however, 
that only 60% of eligible voters 
actually voted in that election. 



ED A B C D E 
poll: 71% voted in elec 

ED A B C D E 

poll: 71% voted in elec 

records: 60% of elig voters 
voted 



j Pure fact. There was a poll, and 
71% of the people who responded 
said they voted in the last election. 

Okay, that's strange. Records show 
that only 60% of people who were 
\ allowed to vote actually voted. 




Step 3: State the Goal, 

How can it be the case that, when asked, 71% of the people said they voted, but records show only 60% 
of those who were allowed to vote actually voted? I don't think it would be because some people voted who 
weren't allowed to — I guess that would technically resolve the discrepancy, but I doubt the GMATis going 
to say that! So what could it have been? Maybe some people are remembering incorrectly or mixed up the elec- 
tion in question. Oh, I know! Polls always have a margin of error, so maybe the margin of error accounts for 
the discrepancy. 

Okay, I need to find something that will make the whole thing make sense — it'll explain why 71% said they 
voted but records showed that only 60% actually voted. 



Step 4: Work from wrong to right 



(A) The margin of error 
for the survey was plus 
or minus five percentage 
points. 



(B) Fifteen percent of 
the survey s respondents 
were living overseas at 
the time of the election. 



ED A B C D E 

poll: 71% voted in elec 

records: 60% of elig voters 
voted 



ED A B C D E 

poll: 71% voted in elec 

records: 60% of elig voters 
voted 



Margin of error, bingo! Excellent. So the real 
\ percentage could've been anywhere from. . . 
\ 71% + 5% to 71%- 5%.. . which is still 
66%. This doesn't go far enough. Still, it's 
about margin of error. I'm going to mark this 
one and come back to it later. 
This percentage is larger than the 11% 
discrepancy mentioned in the argument. But 
what group are they talking about? Are these 
the people who did vote, or didn't vote, or 
some mix of the two? And what does "living 
overseas" imply? This country might allow 
people to vote by absentee ballot. This doesn't 
resolve any thing 



202 



GMAT 



Evidence Family 



Chapter 6 



(C) Prior research has 
shown that people who 
actually do vote are also 
more likely to respond 
to polls than those who 
do not vote. 



ED 



AfiCDE 



(D) Some people who 
intend to vote are pre- 
vented from doing so 
by last-minute conflicts 
on election day or other 
complications. 

(E) People are less likely 
to respond to a voting 
poll on the same day 
that they voted. 



poll: 71% voted in elec 

records: 60% of elig voters 
voted 



ED 



A ft C B E 



poll: 71% voted in elec 

records: 60% of elig voters 
voted 



ED A ft (QBE 
poll: 71% voted in elec 



records: 60% of elig voters 
voted 



People who vote are also more likely to re- 
spond to a survey. What does that mean? Of 
the people who responded, more were likely 
to have been voters than is represented in the 
overall population. Oh, I see — the survey 
group was skewed towards those who voted. 
That's why 71% of that sub-group could have 
voted while only 60% of the overall popula- 
tion of eligible voters voted. That's better than 
A — F 11 get rid of A. 

Vm sure this is true in the real world. How 
does it affect this argument? The survey 
took place after the election; it asked people 
whether they had voted in the past. It doesnt 
address what people intended to do before the 
election. 

We have no idea when the poll was taken, 
sol cant do much with this. Even if the 
poll were done the same day as the election, 
this just highlights the discrepancy — its 
even more puzzling now. I would expect the 
percentage of people who said they voted to be 
lower than the real percentage because those 
who didnt vote that day would be more likely 
to agree to participate in the poll. 




Common Trap Answers 

One common wrong answer trap will seem to be on topic because it will address one of the premises, 
but it won't actually resolve the discrepancy between the two premises. This trap answer is actually 
Out Of Scope because it doesn't address the discrepancy between the premises. Some of these will be 
more obviously out of scope, such as answer D, while others will be trickier because they just don't go 
quite far enough, such as answer A. If answer A had said that the margin of error was plus or minus 15 
percentage points, that could have been the correct answer. 

We can also see Reverse Logic traps, where the answer choice actually highlights or points out the dis- 
crepancy — that is, the answer tells us that there is a discrepancy rather than providing new information 
to show that there really isn't a discrepancy. 



MANHATTAN 
GMAT 



Chapter 6 



Evidence Family 




Takeaways for Explain a Discrepancy Questions 

Discrepancy question stems will usually contain the words "explain" or "resolve" as well as the words "if 
true" (or synonyms). 

The argument will consist of premises only; it will not contain a conclusion. The premises will not 
make sense together. Upon reading the argument, we should think, "Hmm, why would those two 
things both happen? That doesn't make sense!" 

The correct answer will resolve the discrepancy — that is, the correct answer will show that there really 
isn't any discrepancy at all. If we insert the correct answer into the argument, we should be able to say, 
"Oh, okay, that makes sense now." 

The most common trap answers will try to address something in the argument but will be Out Of 
Scope in some way. Perhaps the answer will address only one premise and not the other. Perhaps the 
answer will discuss a group that isn't at issue or a circumstance that occurred at the wrong time. The 
trickiest wrong answers of this type will address both premises but won't go far enough to resolve the 
discrepancy. 

We may also see Reverse Logic trap answers, which will highlight or point out the discrepancy rather 
than fix it. These can be tricky if we forget that our task is to fix the discrepancy, not point out what 
the discrepancy is. 



EXCEPT Questions 



As we saw with Assumption Family questions, Evidence Family questions can also be presented in the 
negative "Except" format. These are more likely to occur on Discrepancy questions than on Inference 
questions. 

A regular Discrepancy question might read: 

Which of the following, if true, would best help to explain the surprising finding? 
An EXCEPT Discrepancy question might read: 

Each of the following, if true, could help to explain the surprising finding EXCEPT: 

What is the difference in wording between those two questions? 

The first one tells us that one answer choice, and only one, explains the discrepancy. That is the answer 
choice that we want to pick. 



MANHATTAN 
GMAT 



Evidence Family 



Chapter 6 



The second one tells us that four answer choices explain the discrepancy. These four are all wrong an- 
swers. The fifth answer will NOT explain or resolve the discrepancy. This is the "odd one out" and, as 
we saw in the Strengthen and Weaken chapter, its the answer that we want to pick. 

Similarly, on an Inference EXCEPT question, four answer choices would represent things that must be 
true according to the argument, and we will eliminate these four. One answer will represent something 
that does not have to be true; this is our "odd one out" and the correct answer. 




MANHATTAN 
GMAT 



Evidence Family 

Problem Set 



Chapter 6 



1 . Nitrogen Triiodide 

Nitrogen triiodide is a highly explosive chemical that is easy to make from only 
two ingredients: ammonia and concentrated iodine. However, nitrogen triiodide 
has never been known to be used in a terrorist or criminal attack. 

Which of the following, if true, is the most likely explanation for the discrepancy 
described above? 

(A) Ammonia can be bought in a grocery store, but concentrated iodine must 
be obtained from somewhat more restricted sources, such as chemical sup- 
ply houses. 

(B) Nitrogen triiodide is only one of several powerful explosives that can be 
made from ammonia. 

(C) Many terrorists and criminals have used other chemical explosives such as 
TNT or PETN. 

(D) Airport security devices are typically calibrated to detect nitrogen com- 
pounds, such as ammonia and ammonium compounds. 

(E) Nitrogen triiodide is extremely shock sensitive and can detonate as a result 
of even slight movement. 

2. Mycenaean Vase 

Museum A will display only undamaged objects of proven authenticity. Doubts 
have been raised about the origins of a supposedly Mycenaean vase currently on 
display in the museum's antiquities wing. The only way to establish this vase's au- 
thenticity would be to pulverize it, then subject the dust to spectroscopic analysis. 

The claims above, if true, most strongly support which of the following conclu- 
sions? 

(A) Authentic Mycenaean vases are valuable and rare. 

(B) Museum A has been beset with questions about the provenance of many of 
the items in its antiquities wing. 

(C) The vase in question will no longer be displayed in Museum A. 

(D) Spectroscopic analysis has revolutionized the forensic investigation of art 
forgery. 

(E) Knowingly or not, many of the world's museums display some forgeries. 




MAN 



LJ 

n 




207 



GMAT 



er 6 



Evidence Family 



3. Gas Mileage 

The average fuel efficiency of vehicles sold nationwide during the period 2000- 
2004 was 25 miles per gallon; the corresponding figure during the period 1995- 
1999 was 20 miles per gallon. The national average price of gasoline during the 
period 2000-2004 was $2 per gallon; the corresponding figure during the period 
1995-1999 was $1.60 per gallon. 

The statements above, if true, best support which of the following conclusions? 

(A) The average fuel efficiency of vehicles sold nationwide should reach 30 
miles per gallon for the period 2005-2009. 

(B) The national average price of gasoline during 1997 was lower than the cor- 
responding price during 2003. 

(C) Rising gasoline prices led consumers to purchase more fuel-efficient cars. 

(D) Between the two described time periods, the national average fuel effi- 
ciency and the national average gasoline price both increased at roughly the 
same rate. 

(E) Consumers spent more money on gasoline during the period 2000-2004 
than during the period 1995-1999. 

4. CarStore 

CarStore's sales personnel have an average of fifteen years' experience selling 
automobiles, and they regularly sell more cars than other local dealers. Despite 
this, CarStore has recently implemented a mandatory training program for all 
sales personnel. 

Which of the following, if true, best explains the facts given above? 

(A) The sales personnel in CarStore have historically specialized in aggressively 
selling automobiles and add-on features. 

(B) Salespeople at other local dealers average 10 years' experience. 

(C) It is common for new or less experienced employees to participate in train- 
ing programs. 

(D) Pricing information, which used to be confidential, has recently been re- 
leased on the internet, and many customers try to negotiate lower prices 
using this data. 

(E) Several retailers that compete directly with CarStore use "customer-cen- 
tered" sales approaches. 



MANHATTAN 
GMAT 



Evidence Family 



5. Stem Cell Research 

Government restrictions have severely limited the amount of stem cell research 
United States companies can conduct. Because of these restrictions, many United 
States scientists who specialize in the field of stem cell research have signed long- 
term contracts to work for foreign companies. Recently, Congress has proposed 
lifting all restrictions on stem cell research. 

Which of the following statements can most properly be inferred from the infor- 
mation above? 

(A) Some foreign companies that conduct stem cell research work under fewer 
restrictions than some United States companies do. 

(B) Because United States scientists are under long-term contracts to foreign 
companies, there will be a significant influx of foreign professionals into the 
United States. 

(C) In all parts of the world, stem cell research is dependent on the financial 
backing of local government. 

(D) In the near future, United States companies will no longer be at the forefront 
of stem cell research. 

(E) If restrictions on stem cell research are lifted, many of the United States sci- 
entists will break their contracts to return to United States companies. 

6. Hunting Season 

In an effort to reduce the number of deer, and therefore decrease the number of 
automobile accidents caused by deer, the government lengthened the deer hunt- 
ing season earlier this year. Surprisingly, the number of accidents caused by deer 
has increased substantially since the introduction of the longer hunting season. 

All of the following, if true, help to explain the increase in traffic accidents caused 
by deer EXCEPT: 

(A) The presence of humans in the woods causes the deer to move to new areas, 
which causes the deer to cross roads more frequently than normal. 

(B) In the area where the deer live, traffic has increased substantially precisely 
because of the lengthened hunting season. 

(C) Most automobile accidents involving deer result from cars swerving to avoid 
deer, and leave the deer in question unharmed. 

(D) Deer tend to bolt when hearing gunshots or other loud sounds and are 
more likely to run across a road without warning. 

(E) A new highway was recently built directly through the state's largest forest, 
which is the primary habitat of the state's deer population. 



Chapter 6 



Evidence Family 



7. World Bank 



In 2010, China comprised about 10 percent of the world's gross domestic product 
(GDP), and its voting share in the World Bank was increased from under 3 percent 
to 4.4 percent. During the same timeframe, France comprised about 4 percent of 
the world's GDP and saw its voting share in the World bank drop from 4.3 percent 
to 3.8 percent. 

Which of the following can be logically concluded from the passage above? 

(A) World Bank voting shares are allocated based upon each country's share of 
the world's GDP. 

(B) The new ratio of voting share to percentage of world GDP is lower for China 
than it is for France. 

(C) Gross domestic product is the most important factor in determining voting 
share at the World Bank. 

(D) China should be upset that its voting share does not match its proportion of 
the world's GDP. 

(E) France lost some of its voting share to China because China comprised a 



Two-dimensional bar codes are omni-directional; that is, unlike one-dimensional 
bar codes, they can be scanned from any direction. Additionally, two-dimensional 
bar codes are smaller and can store more data than their one-dimensional coun- 
terparts. Despite such advantages, two-dimensional bar codes account for a 
much smaller portion of total bar code usage than one-dimensional bar codes. 

Which of the following, if true, most helps to resolve the apparent paradox? 

(A) Many smaller stores do not use bar codes at all because of the expense. 

(B) For some products, the amount of data necessary to be coded is small 
enough to fit fully on a one-dimensional bar code. 

(C) Two-dimensional bar codes are, on average, less expensive than one-dimen- 
sional bar codes. 

(D) Two-dimensional bar codes can also be scanned by consumer devices, such 
as cell phones. 

(E) One-dimensional bar codes last longer and are less prone to error than two- 
dimensional bar codes. 




8. Bar Codes 



larger portion of the world's GDP. 



210 



MANS* 
GMAT 



HATTAN 



Evidence Family 



Chapter 6 



Solutions 



1. Nitrogen Triiodide: The correct answer is E. 
Step 1: Identify the question. 



Which of the following, if true, 
is the most likely explanation 
for the discrepancy described 
above? 

Step 2: Deconstruct the argument. 

Nitrogen triiodide is a highly 
explosive chemical that is easy 
to make from only two ingredi- 
ents: ammonia and concentrated 
iodine. 

However, nitrogen triiodide has 
never been known to be used in 
a terrorist or criminal attack. 



Step 3: State the Goal. 



ED 



A B C D E 



The word "discrepancy" indicates 
that this is a Discrepancy ques- 
tion. 



ED A B C D E 
NT expl, easy to make 

ED A B C D E 

NT expl, easy to make 

BUT never used by terr or 
crims 



This is a fact. I think the main 
point is that it's easy to make, 
not that I need those two specific 
chemicals y so Tm not going to 
write them down. 
That's weird. If it's so easy to 
make, why haven t criminals and 
terrorists used it? Maybe it's hard 
to get one of the ingredients, or 
they're really expensive? 



This is a Discrepancy question, so the argument will provide two seemingly contradictory pieces of informa- 
tion. I need to find something that will make everything make sense. 

In this case, there's an explosive that's easy to make, and yet criminals have never used it. I need to find some- 
thing that explains why. 

Step 4: Work from wrong to right. 



(A) Ammonia can be 
bought in a grocery 
store, but concentrated 
iodine must be obtained 
from somewhat more re- 
stricted sources, such as 
chemical supply houses. 



ED A B C D E 
NT expl, easy to make 



This is kind of like what I said before — it's 
harder to get one of the chemicals. This might 
explain it. . . except it doesn't say that you 
BUT never used by terr or can't get iodine. It just says you have to go to 
cr i ms a special place, but you can still get it. So I'm 

not sure that really explains why no criminals 
have ever used it. I'll leave this in until I find 
something better. 




MANHATTAN 
GMAT 



er 6 



Evidence Family 



(B) Nitrogen triiodide is 
only one of several pow- 
erful explosives that can 
be made from ammonia. 

(C) Many terrorists and 
criminals have used 
other chemical explo- 
sives such as TNT or 
PETN. 

(D) Airport security 
devices are typically 
calibrated to detect ni- 
trogen compounds, such 
as ammonia and ammo- 
nium compounds. 

(E) Nitrogen triiodide is 
extremely shock sensi- 
tive and can detonate 

as a result of even slight 
movement. 



ED ABCDE 

NT expl, easy to make 

BUT never used by terr or 
crims 

ED ABGDE 

NT expl, easy to make 

BUT never used by terr or 
crims 

ED A fi € © E 

NT expl, easy to make 

BUT never used by terr or 
crims 



So you can make even more explosives from 
this chemical? That doesn't explain why the 
criminals have never made it. 



Again, this doesnt explain why they haven t 
used the NT explosive. Maybe if TNT 
or PETN are a lot cheaper or easier to 
make — but this choice doesnt say that. 

This might explain why no one has tried 
to bring these explosives into airports, but 
it doesnt explain why these explosives have 
never been used in any type of attack any- 
where. 



ED AfiG ©(g) 
NT expl, easy to make 



Here we go. If the bomb is so unstable that it 
could go off at any moment, including right 
after you make it, then it makes sense that 
BUT never used by terr or criminals dont want to use these explosives. 
cnms This is better than answer A. 



2. Mycenaean Vase: The correct answer is C. 
Step 1: Identify the question. 

The claims above, if true, most In ABCDE 
strongly support which of the 
following conclusions? 



Step 2: Deconstruct the argument. 

Museum A will display only 
undamaged objects of proven 
authenticity. 



In 



ABCDE 



MA: only perfect, auth 
objects 



The language "strongly support" 
could indicate an Inference or a 
Strengthen question. The question 
stem indicates that the answer 
choice contains the conclusions, 
though (and the argument didnt 
have a conclusion), so this is an 
Inference question. 



This is a fact — all objects have to 
be perfect and authenticated for 
MA to display them. 



MANHATTAN 
GMAT 



Evidence Family 



Doubts have been raised about 
the origins of a supposedly 
Mycenaean vase currently on 
display in the museums antiqui- 
ties wing. 

The only way to establish this 
vases authenticity would be to 
pulverize it, then subject the 
dust to spectroscopic analysis. 



Step 3: Stat e the Goal. 



In A B C D E 

MA: only perfect, auth 
objects 

MV: auth doubtful 
In A B C D E 

MA: only perfect, auth 
objects 

MV: auth doubtful 

to auth M V, must destroy it 



Another fact: they're not sure 
whether this vase is authentic. 



That's interesting. In order to 
prove whether the vase is authen- 
tic, you ve got to destroy it! 



This is an Inference question; I need to find something that must be true according to the info given in the ar- 
gument. In this case, they re not sure whether this vase is authentic, and the only way to establish its authen- 
ticity is to destroy it. But then they cant display it anymore because they'll only display it if it's perfect! 



Step 4: Work from wron g to rig ht. 



(A) Authentic Myce- 
naean vases are valuable 
and rare. 



In A B C D E 

MA: only perfect, auth 
objects 

MV: auth doubtful 

to auth MV, must destroy 



This might be true, but it doesn't have to be 
true. The argument says nothing about value 
or rarity. 



(B) Museum A has been 
beset with questions 
about the provenance of 
many of the items in its 
antiquities wing. 



(C) The vase in ques- 
tion will no longer be 
displayed in Museum A. 



In A © C D E 

MA: only perfect, auth 
objects 

MV: auth doubtful 

to auth MV, must destroy 
it 

In ABCDE 

MA: only perfect, auth 
objects 

MV: auth doubtful 

to auth MV, must destroy 
it 



The argument is only about one particular 
vase. Any other items are out of scope. 



This is exactly what I said before! If they try 
to authenticate it, they'll destroy the vase, in 
which case they can't display it. And if they 
dont try to authenticate it, then they won't 
know whether it's authentic, in which case 
Museum A still won't display it. This has 
to be true (though I'll check the other two 
answers to be sure). 



Chapter 6 



Evidence Family 



(D) Spectroscopic analy- 
sis has revolutionized 
the forensic investiga- 
tion of art forgery. 



In 



ABCBE 



(E) Knowingly or not, 
many of the world's 
museums display some 
forgeries. 



MA: only perfect, auth 
objects 

MV: auth doubtful 

to auth MV, must destroy 
it 

In Afi©©£ 

MA: only perfect, auth 
objects 

MV: auth doubtful 

to auth M V, must destroy 
it 



This might be true, but it doesn't have to be 
true that it "revolutionized" the field. It just 
has to work in general. 



I can believe that this is probably true, but it 
doesn't absolutely have to be true. 



.... .. 



3. Gas Mileage: The correct answer is D. 
Step 1: Identify the question. 



The statements above, if true, 
best support which of the fol- 
lowing conclusions ? 



Step 2: Deconstruct the argument. 

The average fuel efficiency of 
vehicles sold nationwide dur- 
ing the period 2000-2004 was 
25 miles per gallon; the cor- 
responding figure during the 
period 1995-1999 was 20 miles 
per gallon. 



In 



A B C D E 



In 



A B C D E 



95-99 


00-04 


AFE 20 


AFE 25 



The language "best support" 
could indicate an Inference or a 
Strengthen question. The question 
stem indicates that the answer 
choice contains the conclusions, 
though (and the argument didn't 
have a conclusion), so this is an 
Inference question. 



These are all facts, which I'm 
expecting because this is an Infer- 
ence question. They're talking 
about time periods and figures, so 
maybe a table is the best way to 
keep track. 



214 



GMAT 



Evidence Family 



In 



A B C D E 



95-99 


00-04 


AFE 20 


AFE 25 


AG $1.60 


AG $2 



Yep, a table was a good ideal 
More facts and figures for the 
same timeframe. 



The national average price 
of gasoline during the period 
2000-2004 was $2 per gallon; 
the corresponding figure during 
the period 1995-1999 was $1.60 
per gallon. 

Step 3: State the Goal. 



This is an Inference question, so I'm looking for something that must be true based on all this data. I was 
given specific figures for average fuel efficiency and average gas price for two time periods. Both went up over 
time. 



Step 4: Work from wrong to rig ht. 



(A) The average fuel ef- 
ficiency of vehicles sold 
nationwide should reach 
30 miles per gallon for 
the period 2005-2009. 

(B) The national average 
price of gasoline during 
1997 was lower than 
the corresponding price 
during 2003. 

(C) Rising gasoline 
prices lead consumers 
to purchase more fuel- 
efficient cars. 

(D) Between the two 
described time periods, 
the national average fuel 
efficiency and the na- 
tional average gasoline 
price both increased at 
roughly the same rate. 



In 



A B C D E 



95-99 


00-04 


AFE 20 


AFE 25 


AG $1.60 


AG $2 


In AfiCDE 


95-99 


00-04 


AFE 20 


AFE 25 


AG $1.60 


AG $2 ! 


In ABODE 


95-99 


00-04 


AFE 20 


AFE 25 ! 


AG $1.60 


AG $2 


In A B € D E 


95-99 


00-04 


AFE 20 


AFE 25 


AG $1.60 


AG $2 



"Should reach?" That doesnt have to be true. 
Who knows what s going to happen in the 
future? 



The data given is only for the 5-year periods 
95 to 99 and 00 to 04. 1 have no idea what 
the numbers were for 1997 and 2003 specifi- 
cally. 

That might be true, but it doesnt have to 
be true. The argument doesnt say anything 
about why consumers decide to purchase 
certain cars. 

Increased at the same rate? Hmm. I dont 
know, but I can calculate based on the figures 
I was already given. The AFE figure went 
from 20 to 25. The increase, then, was 5 over 
a base (or starting point) of 20. 5/20 = 1/4, 
or a rate of 25%. The AG figure went from 
1.6 to 2, which is an increase of 0.4 over a 
starting point of 1.6. 0.4/1.6= 1/4, or a rate 
of 25% again. Hey, this is true! 



Chapter 6 



Evidence Family 



(E) Consumers spent 
more money on gaso- 
line during the period 
2000-2004 than during 
the period 1995-1999. 



In AB6@E 



95-99 


00-04 


AFE 20 


AFE 25 


AG $1.60 


AG $2 



Tricky! This one seems pretty good at first 
glance, but average price per gallon is not the 
same thing as total amount of money spent. 
Its true that the average price was higher, but 
maybe people bought fewer gallons of gasoline 
(especially because fuel efficiency was better!). 
This one might be true, but it doesnt have to 
be. 




4. CarStore: The correct answer is D. 

Step 1: Identify the question. 

Which of the following, if true, 
best explains the facts given 
above? 

Step 2: Deconstruct the argument. 

CarStore s sales personnel have 
an average of fifteen years' 
experience selling automobiles, 
and they regularly sell more cars 
than other local dealers. 
Despite this, CarStore has 
recently implemented a manda- 
tory training program for all 
sales personnel. 

Step 3: State the Goal. 



ED 



ED 



A B C D E 



A B C D E 



SP: avg 15y exp; sell more 
than comp 



ED A B C D E 

SP: avg 15y exp; sell more 
than comp 

BUT CSreq trngfor all 



The language "best explains the 
facts" is a slightly unusual form for 
a Discrepancy question. 



CarStore s people have 15 years' 
experience on average, and they 
sell more cars than the competi- 
tion. These are facts. 

i Here's the contrast. Why are they 
going to make them all go through 
training! Maybe something has 
changed in the marketplace? 



This is a Discrepancy question, so I need to find an answer that explains why these two facts are actually 
NOT contradictory after all. What would explain why CS is requiring its employees to go through new train- 
ing? Maybe something has changed in the marketplace that would require new training. 



216 



GMAT 



Evidence Family 



Step 4: Work from wrong to rig ht. 



(A) The sales person- 
nel in CarStore have 
historically specialized 
in aggressively selling 
automobiles and add-on 
features. 

(B) Salespeople at other 
local dealers average 10 
years' experience. 

(C) It is common for 
new or less experienced 
employees to participate 
in training programs. 

(D) Pricing informa- 
tion, which used to be 
confidential, has re- 
cently been released on 
the internet, and many 
customers try to negoti- 
ate lower prices using 
this data. 

(E) Several retailers that 
compete directly with 
CarStore use "customer- 
centered" sales ap- 
proaches. 



ED A B C D E 

SP: avg 15y exp; sell more 
than comp 

BUT CS req trng for all 

ED ABCDE 

SP: avg 15y exp; sell more 
than comp 

BUT CS req trng for all 
ED ABCDE 

SP: avg 15y exp; sell more 
than comp 

BUT CS req trng for all 
ED ABCDE 

SP: avg 15y exp; sell more 
than comp 

BUT CS req trng for all 

ED Afi€®E 

SP: avg 15y exp; sell more 
than comp 

BUT CS req trng for all 



IfCS wants to change the way their people 
sell cars, then new training would make 
sense. . . but this choice just talks about what 
they've done in the past, not what they want 
to do in the future. This doesnt explain the 
discrepancy. 

So the CS people are more experienced, on 
average, than other salespeople in the area. 
If anything, this just accentuates the discrep- 
ancy: why do the more experienced people 
need training! 

This makes sense, but again does not explain 
why the employees who average 15 years' 
experience need training. The argument said 
that all sales personnel have to undergo the 
training, not just the new ones. 
Ah, so the situation has changed. Customers 
now know some info that used to be confi- 
dential. That might change negotiations, so 
it makes sense that the salespeople might need 
new training. 



Thats what they already use — the answer 
doesnt indicate that anything has changed. 
Nor does it indicate that CS doesnt use a 
customer-centered approach or that consum- 
ers prefer a customer-centered approach. 
This doesnt explain why the CS people need 
training. 



Chapter 6 



Evidence Family 




5. Stem Cell Research: The correct answer is A. 
Step 1: Identify the question. 



Which of the following state- 
ments can most properly be 
inferred from the information 
above? 

Step 2: Deconstruct the argument. 



Government restrictions have 
severely limited the amount of 
stem cell research United States 
companies can conduct. 
Because of these restrictions, 
many United States scientists 
who specialize in the field of 
stem cell research have signed 
long-term contracts to work for 
foreign companies. 
Recently, Congress has proposed 
lifting all restrictions on stem 
cell research. 



In 



A B C D E 



In A B C D E 
SCR restrict by US govt 

In A B C D E 

SCR restrict by US govt ~< 
US sci work foreign corns 
instead 



In 



A B C D E 



SCR restrict by US govt -» 
US sci work foreign corns 
instead 

US gov: maybe lift restrict? 



The word " inferred" indicates 
that this is an Inference question. 



j This is a fact. The US government 
j restricts this stem cell research 
(SCR). 



"Because of" that — so the first 
sentence leads to the second sen- 
tence. 



Still a fact: the government is 
j considering lifting the restrictions. 
Maybe that* 11 bring the scientists 
back to work for US companies? 



Step 3: Sta te the Goal. 

This is an Inference question, so I need to find something that must be true based on the info given so far. 
The US government restricts a certain kind of research, so many US scientists who do this type of research are 
working for foreign companies instead. Congress might lift the restrictions. 



218 



GMAT 



Evidence Family 



Step 4: Work from wrong to right. 



(A) Some foreign 
companies that conduct 
stem cell research work 
under fewer restrictions 
than some United States 
companies do. 

(B) Because United 
States scientists are un- 
der long-term contracts 
to foreign companies, 
there will be a signifi- 
cant influx of foreign 
professionals into the 
United States. 

(C) In all parts of the 
world, stem cell research 
is dependent on the fi- 
nancial backing of local 
government. 

(D) In the near future, 
United States companies 
will no longer be at the 
forefront of stem cell 
research. 

(E) If restrictions on 
stem cell research are 
lifted, many of the Unit- 
ed States scientists will 
break their contracts to 
return to United States 
companies. 



In ABCDE 

SCR restrict by US govt -* 
US sci work foreign corns 
instead 

US gov: maybe lift restrict? 

In ABCDE 

SCR restrict by US govt -> 
US sci work foreign corns 
instead 

US gov: maybe lift restrict? 

In ABCDE 

SCR restrict by US govt 
US sci work foreign corns 
instead 

US gov: maybe lift restrict? 
In ABCDE 

SCR restrict by US govt 
US sci work foreign corns 
instead 

US gov: maybe lift restrict? 
In ®B € D E 

SCR restrict by US govt -» 
US sci work foreign corns 
instead 

US gov: maybe lift restrict? 



If the researchers decided to work for for- 
eign companies specifically because the U.S. 
companies had restrictions, then that would 
mean that at least some foreign companies did 
have fewer restrictions. Yes, this one must be 
true! I'll check the other answers just in case, 
though. 

This might be true, but it certainly doesn't 
have to be true. The argument doesn't say 
anything about foreign professionals coming 
into the U.S. 



The argument doesn't say anything about how 
this type of research gets its financial backing 
This doesn't have to be true. 



Out of scope. The argument doesn't discuss 
who is or will be at the forefront of this kind 
of research. 



Maybe this will happen, but it doesn't have to 
happen. It isn't easy to break a contract. 



Chapter 6 



Evidence Family 




6. Hunting Season: The correct answer is C. 
Step 1: Identify the question. 



Q stem All of the following, if 
true, help to explain the increase 
in traffic accidents caused by 
deer EXCEPT. 



ED Ex A B C D E 



The language "help to explain" 
indicates that this is a Discrepan- 
cy question. This is also an Except 
question. 



Step 2: Deconstruct the argument. 

In an effort to reduce the 
number of deer, and therefore 
decrease the number of automo- 
bile accidents caused by deer, 
the government lengthened the 
deer hunting season earlier this 
year. 

Surprisingly, the number of 
accidents caused by deer has 
increased substantially since the 
introduction of the longer hunt- 
ing season. 

Step 3: State the Goal. 



ED Ex A B C D E 

G: t HS -> I #D -> I #car 
acc from D 



ED Ex A B C D E 

G: T HS -> I #D -* I #car 
acc from D 

BUT # car acc t 



Multiple levels here. First, the 
| government lengthened hunt- 
\ ing season, which is supposed to 

reduce the number of deer, which 
\ is then supposed to reduce the 

number of car accidents caused by 

deer. 

That's weird. The exact opposite 
has happened: there have been 
more car accidents caused by deer! 



This is a Discrepancy Except question. Normally on discrepancy questions, I'm looking for the answer that 
makes the contradictory evidence make sense. On this one, though, all four wrong answers will fix the discrep 
ancy. The "odd one out" — the one that doesnt fix the discrepancy — will be the right answer. 

So I need to find (and cross off) four things that explain why there have been even more car accidents caused 
by deer. 



Step 4: Work from wrong to rig ht. 



(A) The presence of 
humans in the woods 
causes the deer to move 
to new areas, which 
causes the deer to cross 
roads more frequently 
than normal. 



ED Ex A B C D E 

G: t HS -> i #D -» I 
#car acc from D 

BUT # car acc f 



If hunting season is lengthened, then there 
will be people in the woods for a longer period 
of time. According to this choice, that means 
the deer are going to cross the roads more 
frequently than they otherwise would have. 
That could increase the likelihood of accidents 
due to deer, which explains the discrepancy. 
Cross this one off. 



220 



GMAT 



Evidence Family 



Chapter 6 



(B) In the area where 
the deer live, traffic has 
increased substantially 
precisely because of the 
lengthened hunting 



ED Ex 



AfiCDE 



season. 



(C) Most automobile 
accidents involving deer 
result from cars swerv- 
ing to avoid deer, and 
leave the deer in ques- 
tion unharmed. 

(D) Deer tend to bolt 
when hearing gunshots 
or other loud sounds 
and are more likely to 
run across a road with- 
out warning. 

(E) A new highway was 
recently built directly 
through the states larg- 
est forest, which is the 
primary habitat of the 
state s deer population. 



G: t HS -> A #D -» A 
#car acc from D 

BUT # car acc t 

ED Ex A fi C D E 

G: t HS -> A #D -> A 
#car acc from D 

BUT # car acc t 

ED Ex A B C © E 

G: t HS -> A #D -> A 
#car acc from D 

BUT # car acc t 



ED Ex 



A BO© E 



G: t HS -> A #D -> A 
#car acc from D 

BUT # car acc t 



Oh, this makes sense. The lengthened hunting 
season actually caused more traffic, so there 
are more chances for accidents between cars 
and deer where the deer live. This explains 
the discrepancy, too. 

This one is tricky! It sounded like it explained 
the discrepancy when I first read it, but then 
I realized something: its just explaining how 
the accidents tend to happen, but it doesnt 
address why there are MORE accidents now 
than there used to be. 

Ah, so if there are gunshots for a longer length 
of time, then there are more chances for the 
deer to bolt and cross the road suddenly. . . 
increasing the chances of an accident. 



The situation has changed from the year be- 
fore: a new highway was built right through 
the area where the deer live. So it would 
make sense that there are now more accidents 
caused by deer. 




7. World Bank: The correct answer is B. 



Step 1: Identify the question. 

Which of the following can be 
logically concluded from the 
passage above? 

Step 2: Deconstruct the argument. 

In 2010, China comprised about 
10 percent of the world s gross 
domestic product (GDP), and 
its voting share in the World 
Bank was increased from under 
3 percent to 4.4 percent. 



In A B C D E 



In A B C D E 



2010 


GDP 


VS 


C 


10 


<3 -» 4.4 



The language "logically con- 
cluded" indicates that this is an 
Inference question. 



A bunch of stats about China 
in 2010. I just need to keep this 
straight because, glancing down, I 
can see the next sentence has more 
numbers. 



MANHATTAN 
GMAT 



Evidence Family 



In 



A B C D E 



2010 


GDP 


VS 


C 


10 


<3 -» 4.4 


F 


4 


4.3 -* 3.8 



S# *y/>£ of stats, but about 
France this time. Same time- 
frame. 



During the same timeframe, 
France comprised about 4 
percent of the world's GDP and 
saw its voting share in the World 
bank drop from 4.3 percent to 
3.8 percent. 

Step 3: State the Goal. 

This is an Inference question, so I need to find something that must be true based upon the info given so far. 
There are a lot of numbers to keep straight, but generally, China has a larger share of the world GDP than 
France. China used to have a lower voting share than France, but now it has a higher share. 

Step 4: Work from wrong to rig ht. 



(A) World Bank vot- 
ing shares are allocated 
based upon each coun- 
try's share of the world's 
GDP. 

(B) The new ratio of 
voting share to percent- 
age of world GDP is 
lower for China than it 
is for France. 

(C) Gross domestic 
product is the most 
important factor in de- 
termining voting share 
at the World Bank. 

(D) China should be 
upset that its voting 
share does not match 
its proportion of the 
world's GDP. 

(E) France lost some of 
its voting share to China 
because China com- 
prised a larger portion 
of the world's GDP. 



In 



A B C D E 



2010 


GDP 


VS 


C 


10 


<3 -> 4.4 


F 


4 


4.3 -» 3.8 




In 


ABCDE 


2010 


GDP 


VS 


C 


10 


<3 -> 4.4 


F 


4 


4.3 -» 3.8 


In 


A B € D E 


2010 


GDP 


VS 


C 


10 


<3 -» 4.4 


F 


4 


4.3 -> 3.8 


In 


A B € & E 


2010 


GDP 


VS 


C 


10 


<3 -» 4.4 


F 


4 


4.3 -> 3.8 


In 


A(g)€ & E 


2010 


GDP 


VS ! 


C 


10 


<3 -» 4.4 


F 


4 | 


4.3 -* 3.8 



Maybe. It is the case now that China has a 
larger GDP and a larger voting share. But it 
didn't used to be that way. And we only have 
two data points; I don't know the numbers 
with all of the other countries. This doesn't 
have to be true. 

Let's see. China's ratio is 4.4 1 10. And the 
ratio for France is 3.8 1 4. The first number 
is a lot smaller than the second number: the 
first one is 0.44 and the second one is almost 
1. So, yes, it's true that China's ratio is lower 
than France's. 

"Most important?" The argument didn't say 
anything about how voting share is deter- 
mined or which factor is most important. 



China might be upset but this doesn't have to 
be true — and it doesn't have to be true that 
China "should" be upset. That's a judgment 
call 

Maybe this is true, but they didn't actually 
say why the voting shares were changed. I 
could speculate, but this doesn't have to be 
true. 



Evidence Family 



Chapter 6 



8. Bar Codes: The correct answer is E. 



Step 1: Identify the question. 

Which of the following, if true, 
most helps to resolve the appar- 
ent paradox ? 

Step 2: Deconstruct the argument. 

Two-dimensional bar codes are 
omni-directional; that is, unlike 
one-dimensional bar codes, they 
can be scanned from any direc- 
tion. 

Additionally, two-dimensional 
bar codes are smaller and can 
store more data than their one- 
dimensional counterparts. 

Despite such advantages, two- 
dimensional bar codes account 
for a much smaller portion of 
total bar code usage than one- 
dimensional bar codes. 



ED 



A B C D E 



ED 



A B C D E 



2D BC scan any dir; ID BC 
cant 



ED 



A B C D E 



2D BC scan any dir; ID BC 
can't 

2D BC smaller, more data 
ED A B C D E 

2D BC scan any dir; ID BC 
can't 

2D BC smaller, more data 
BUT ID is used » 



The word "paradox" indicates 
that this is a Discrepancy ques- 
tion. 



Okay, so 2D barcodes have a bet- 
ter feature than ID barcodes. 



Even more advantages for the 2D 
barcodes. 



But the ID barcodes are used a lot 
more — why? There must be some 
advantages to the IDs or disad- 
vantages for the 2Ds. 




Step 3: State the Goal. 

/ need to find something that fixes the discrepancy described in the argument: the 2D barcodes have a bunch 
of advantages, but people mostly still use the ID barcodes. Why? Maybe the 2D ones are super-expensive or 
something like that. 

Step 4: Work from wrong to rig ht. 



(A) Many smaller stores 
do not use bar codes 
at all because of the 
expense. 



ED A B C D E Expense — does this explain why ID barcodes 

are still being used? No, wait — this says the 
stores arent using any type of barcode at all. 
So that doesnt explain why the ones who do 
2D BC smaller, more data use barcodes seem to prefer the ID models. 

BUT ID is used » 



2D BC scan any dir; ID 
BC cant 



GMAT 



223 



Chapter 6 



Evidence Family 



(B) For some products, 
the amount of data 
necessary to be coded is 
small enough to fit fully 
on a one-dimensional 
bar code. 

(C) Two-dimensional 
bar codes are, on aver- 
age, less expensive than 
one-dimensional bar 
codes. 

(D) Two-dimensional 
bar codes can also be 
scanned by consumer 
devices, such as cell 
phones. 

(E) One-dimensional 
bar codes last longer and 
are less prone to error 
than two-dimensional 
bar codes. 



ED 



A © C D E 



2D BC scan any dir; ID 
BC cant 

2D BC smaller, more data 

BUT ID is used » 
ED A B € D E 

2D BC scan any dir; ID 
BC cant 

2D BC smaller, more data 

BUT ID is used » 
ED A B € B E 

2D BC scan any dir; ID 
BC cant 

2D BC smaller, more data 

BUT ID is used » 
ED A B € B(|) 

2D BC scan any dir; ID 
BC can't 

2D BC smaller, more data 
BUT ID is used » 



Okay, so some products might not need the 
2D barcodes. Except, this only mentions 
"some" products, while the argument says that 
the 2D barcodes are a "much smaller" portion 
of total usage. This doesn't fully explain the 
discrepancy. 

Less expensive, this is it! Wait a second. No, 
this says the 2D barcodes are less expen- 
sive — that gives them yet another advantage! 
If they're less expensive, you'd expect people to 
use them more. This isn't it. 

This sounds like yet another advantage for the 
2D barcodes. This isn't it either! 



Ah, here we go. Here are two advantages for 
the ID barcodes. If it's true that they last 
longer and are less prone to error, then that 
would explain why people would want to use 
them rather than the 2D barcodes. 



224 



GMAT 



Chapter/7 





Critical Reasoning 



Complete the Argument 



On This Chapter. , . 

Negatively-Worded Claims 
Alternate Wording 



Complete the Argument 



Complete the Argument (CA) questions don't fall into any one Family of questions. Rather, "Complete 
the Argument" is a structure for writing the argument itself, and any of the question types we've already 
discussed could theoretically be written using this structure. In practice, however, most CA questions 
resemble Assumption or Strengthen questions; most will ask us to find an answer that makes a claim or 
conclusion true or much more likely to be true. 

Lets look at an example: 

Which of the following most logically completes the argument below? 

XYZ Industries sells both a premium line of televisions and a basic line. The 
higher-end line sells at a 20% premium but also costs 30% more to produce and 
market. The company has announced that it will stop producing premium televi- 
sions and sell only the regular line in future. This plan will help to improve profit- 
ability since . 

Right away, you'll notice that we have no question stem after the argument. Most of the time, the ques- 
tion will come before, as in the example above, and the question stem also wont help to tell us anything 
other than that we have a "Complete the Argument" structure. 

In the vast majority of Complete the Argument (CA) problems, the last sentence will contain a conclu- 
sion or claim followed by the word "since" or "because" and a blank. In these cases, the arguments are 
asking us to find some kind of a premise that will support the claim given in the same sentence. 

In our above example, the author claims that "this plan will help to improve profits." Our task is to find 
an answer that will make this claim true or much more likely to be true. 

For the above example, for instance, a correct answer might read: 

basic televisions are more profitable for the company than are premium televisions 



227 



Chapter 7 



Complete the Argument 



If its the case that the basic televisions are more profitable, then getting rid of the less profitable product 
and selling only the more profitable product will likely improve overall profitability — this strengthens 
the author's case. The interesting thing is that the argument itself gave us enough information to de- 
termine that the basic televisions are more profitable. If the company charges 20% more on a premium 
television but has to pay 30% more to produce it, then we can conclude that the premium televisions 
are less profitable than the basic televisions. The answer choice, then, is almost repeating a premise that 
we were already told in the argument. 

Contrast that answer with this alternative for a correct answer: 

cutting the production of premium televisions will allow XYZ Industries to in- 
crease production of its basic television line 

In this case, the correct answer provides us with some new information, similar to a strengthen ques- 
tion. If its the case that the company can produce even more of the more-profitable basic TVs, then 
that again makes it much more likely that this plan will improve overall profitability. 

On CA questions, the correct answer might be a restatement of a premise we were already told. Alter- 
natively, the correct answer may introduce a new premise. Either way, the result will be the same: the 
answer will strongly bolster the author s conclusion. 

Negatively-Worded Claims 

Many Complete the Argument questions introduce a negatively-worded twist. Take a look at this varia- 
tion on our original argument: 

Which of the following most logically completes the argument below? 

XYZ Industries sells both a premium line of televisions and a basic line. The 
higher-end line sells at a 20% premium but also costs 30% more to produce and 
market. Producing more televisions from the basic line, however, will not neces- 
sarily help to improve profitability since . 

These questions will not put the negative wording in capital letters, as we've seen on the "EXCEPT" 
question variations. Rather, the conclusion itself is that last sentence: this will not necessarily help to 
improve profitability. Why? Consider this possible correct answer: 

the market for basic televisions is shrinking 

In other words, producing more TVs doesn't necessarily mean we can sell more TVs, and we have to sell 
them in order to make money. If the market for basic TVs is shrinking, then producing more of those 
TVs wont necessarily be beneficial for the company's profitability. 



MANHATTAN 
GMAT 




Complete the Argument Chapter 7 

Alternate Wording 

The first two variations we discussed represent the most common ways in which Complete the Argu- 
ment questions can be presented. There are a few alternate examples, however, that might pop up. 
Students aiming for 90th percentile or higher on the verbal section may want to be prepared for these 
rare variations. 

The rare variants will still typically include the conclusion or claim in the final sentence with the blank, 
but the "lead in" wording to the blank might be different, and what we need to do to find the answer 
might be different as well. 



"lead in" wording 


answer choice should 


most resembles? 


(something) is "illustrated by" 


provide a concrete example of a 
premise from the argument 


n/a 


if (some claim is true), "it 
should be expected that" 


represent something that must 
be true given the information in 
the argument 


Inference questions 


(in order for some claim to be 
true) "it must be shown that" 


represent something that must 
be true given the information in 
the argument 


Inference questions 



Let's look at a full example of one of the more common forms of the Complete the Argument (CA) 
question type. 

Which of the following best completes the passage below? 

The Farmsley Film and Performing Arts Center was built three years ago in down- 
town Metropolis. A recent study shows that, on average, a person who attends 
a show at the Farmsley Center spends $96 at other downtown businesses on 
the day of the show. This fact, however, does not necessarily indicate that the 
Farmsley Center is a significant driver of the economic revitalization of downtown 
Metropolis, since . 

(A) people who do not attend a Farmsley Center show spend $63 on average 
when shopping in the downtown area 

(B) restaurants near the Farmsley Center tend to be more expensive than restau- 
rants in outlying areas 

(C) the Farmsley center generally earns more from films than from plays or other 
performance art projects 

(D) the Farmsley Center is the only downtown theatre large enough to afford to 
show newly-released major Hollywood films 

(E) most of the people who attend films or performances at the Farmsley Center 
do so because they are already in the area to shop 




MANHATTAN 
GMAT 



Chapter 7 



Complete the Argument 



Step 1 : Identify the question. 



Which of the following best 
completes the passage below? 



CA:S 



A B C D E 




Step 2: Deconstruct the argument 



The Farmsley Film and Per- 
forming Arts Center was built 
three years ago in downtown 
Metropolis. 

A recent study shows that, on 
average, a person who attends 
a show at the Farmsley Center 
spends $96 at other downtown 
businesses on the day of the 
show. 

This fact, however, does not 
necessarily indicate that the 
Farmsley Center is a significant 
driver of the economic revital- 
ization of downtown Metropo- 
lis, since . 



CA:S A B C D E 
3ya: F built 

CA:S A B C D E 
3ya: F built 

RS: F ppl spend $96 avg 
other stuff 

CA:S A B C D E 
3ya: F built 

RS: F ppl spend $96 avg 
other stuff 



©F NOT nec -> M econ + 



Step 3; State the Goal, 



The blank and the fact that the 
\ question appears first both indi- 
cate that this is a "Complete the 
Argument" question. The word 
"since" right before the blank 
indicates that this is likely a clas- 
sic "Strengthen the Conclusion" 
variation. 



Pure fact. Fm not even sure I 
need to write this down. 



It sounds like the other businesses 
in the downtown area should be 
happy that the F place is there. 



Oh, wait — but this is saying that 
the $96 thing is NOT necessar- 
ily evidence that F is one of the 
businesses driving the "economic 

\ revitalization" of the area. That's 

\ interesting 



The conclusion said that F is not necessarily driving Ms economic revitalization (so it could be contributing, 
but it also might not be). "Revitalization" implies that the economy was worse for a while but has been getting 
better lately, and F was just built 3 years ago, so that would be a (very small) point in favor ofF helping drive 
the revitalization. 



Plus, the other piece of evidence sounds pretty good: that people spend nearly a hundred bucks in other stores 
on the day of a performance. So why is that F isn't necessarily driving Ms revitalization? Maybe people ordi- 
narily spend $150 if they don't attend a show, so they spend less if they see a show at F? I was just assuming 
that the $96 figure was a good figure; maybe it's not. 



MANHATTAN 
GMAT 



Complete the Argument 



Chapter? 



/ need to find a piece of evidence that makes the claim at least a little more likely to be true — that F isnt 
necessarily a major driver in revitalizing Ms economy. 



Step 4: Work from wrong to right 



(A) people who do not 
attend a Farmsley Cen- 
ter show spend $63 on 
average when shopping 
in the downtown area 



(B) restaurants near the 
Farmsley Center tend to 
be more expensive than 
restaurants in outlying 
areas 



(C) the Farmsley center 
generally earns more 
from films than from 
plays or other perfor- 
mance art projects 

(D) the Farmsley Center 
is the only downtown 
theatre large enough to 
afford to show newly-re- 
leased major Hollywood 
films 

(E) most of the people 
who attend films or 
performances at the 
Farmsley Center do so 
because they are already 
in the area to shop 



CA:S ABCDE 
3ya: F built 

RS: F ppl spend $96 avg 
other stuff 

©F NOT nec -* M econ + 
CA:S ABCDE 

3ya: F built 

RS: F ppl spend $96 avg 
other stuff 

©F NOT nec -» M econ + 

CA:S ABCDE 
3ya: F built 

RS: F ppl spend $96 avg 
other stuff 

©F NOT nec -> M econ + 
CA:S ABCDE 

3ya: F built 

RS: F ppl spend $96 avg 
other stuff 

©F NOT nec -» M econ + 

CA:S ABC D© 
3ya: F built 

RS: F ppl spend $96 avg 
other stuff 

©F NOT nec -> M econ + 



/ did think of something like this. . . but wait, 
this says people who dont attend a show 
spend less than those who do. If anything, that 
would strengthen the idea that F DOES help 
drive the economic revitalization. That's the 
opposite of what I want. 

Hmm. This would mean that people maybe 
spend more money if they re going to a show 
at F because the restaurants are more expen- 
sive. But I dont even know whether people 
are going to restaurants on the day they go to 
a show at F. Vd have to make a few assump- 
tions here to make this work. 
I dont think this has anything to do with 
the conclusion at all. The conclusion is about 
F overall, not the different kinds of things 
people can watch at F. And what does it 
matter how F earns its money as long as its 
earning money? 

Heres a good reason why people might go to F 
vs. other places: this is the only place they can 
see big Hollywoodfilms. Oh, but this answer 
is like answer A — / want something that will 
support the idea that F is NOT necessarily a 
major driver of Ms revitalization. This one 
makes it sound like F IS an important factor. 
People go downtown to shop. While they're 
there, they think, "oh, hey, lets go see a show." 
I see. So people arent coming downtown 
specifically because F is there; they re already 
downtown and just happen to see F and de- 
cide to stay. That would make it more likely 
that F isnt necessarily a driving factor in Ms 
revitalization. 




GMAT 



231 



Complete the Argument 



Common Trap Answers 

The common trap answers will mirror the trap answers we see on the regular question type. For 
example, if the "Complete the Argument" structure really reflects a Strengthen question, as our last 
problem did, then we should expect to see the same trap answers that we see on regular Strengthen 
questions: Reverse Logic (weakens rather than strengthens, as in answers A and D above) and No Tie to 
the Conclusion (as in answers B and C, above). 

Takeaways for Complete the Argument Questions 

Several question types can be written using a Complete the Argument structure, though most CA ques- 
tions come in the Strengthen or Assumption formats. The CA format will always present an underlined 
blank in the argument, and there will not be a question stem following the argument. 

Our first task is to figure out what kind of question we really have. The presence of the words "since" or 
"because" immediately before the blank indicate a Strengthen / Assumption-type question. On these, 
our task will be to find something that supports the argument in some way. Alternatively, if the lan- 
guage before the blank says something similar to "it should be expected that," then we are looking at 
an Inference-type CA. Finally, we might be asked to "illustrate" or "provide an example of" something 
that was discussed in the argument 

Trap answer types will follow the normal patterns for questions of that type; for example, a Strengthen 
CA will have the usual Strengthen-type traps, and an Inference CA will have the usual Infer-type traps. 



Complete the Argument Chapter 7 

Problem Set 



1. Connecting Flight 

Which of the following most logically completes the argument? 

John was flying from San Francisco to New York with a connecting flight in Chi- 
cago on the same airline. Chicago's airport consists of several small stand-alone 
terminals, and it often takes passengers thirty to forty minutes to move between 
terminals. John's plane into Chicago arrived on time. The flight attendant assured 
John that he would not miss his connecting flight thirty minutes later, because 



(A) John's airline is known for always being on time 

(B) another passenger on John's first flight was also scheduled to take John's 
connecting flight 

(C) at the Chicago airport, airlines always fly in and out of the same terminal 

(D) John knew there was another flight to New York scheduled for one hour 
after the connecting flight he was scheduled to take 

(E) the airline generally closes the doors of a particular flight ten minutes before 
it is scheduled to take off 



2. Motor City 

Which of the following best completes the passage below? 

A nonprofit organization in Motor City has proposed that local college students 
be given the option to buy half-price monthly passes for the city's public trans- 
portation system. The nonprofit claims that this plan will reduce air pollution in 
Motor City while increasing profits for the city's public transportation system. 
However, this plan is unlikely to meet its goals, as , 

(A) most college students in Motor City view public transportation as unsafe 

(B) most college students in Motor City view public transportation as prohibi- 
tively expensive 

(C) college students typically do not have the 9-to-5 schedules of most workers, 
and can thus be expected to ride public transportation at times when there 
are plenty of empty seats 

(D) a bus produces more air pollution per mile than does a car 

(E) a large proportion of the college students in Motor City live off campus 




MANHATTAN 
GMAT 



Chapter 7 



Complete the Argument 



3. Deep-brain Stimulation 

Which of the following most logically completes the argument given below? 

Deep-brain stimulation is a new technique for combating severe depression. In a 
recent experiment, electrodes were implanted into the brains of six patients who 
had not responded to any currently approved treatment for depression. When an 
electrical current to the electrodes was switched on, four of the patients reported 
feeling a dramatic reduction in depressive symptoms. The long-term prospects of 
the new treatment are not promising, however, because , 

(A) other treatments for depression may also be effective 

(B) the other two patients reported only a slight reduction of depressive symp- 
toms during the treatment 

(C) deep-brain stimulation relies on the expertise of highly skilled physicians 

(D) when the electrical current is interrupted, the effects of the treatment are 
reversed 

(E) in a subsequent experiment, a one-hour treatment with the electrodes 
resulted in a sustained remission from depression in the four patients for six 
months 

4. Mutual Funds 

Which of the following most logically completes the argument? 

Many managers of mutual funds proclaim that they have been able to generate 
consistently higher rates of return on their investments than the general stock 
market by buying shares of undervalued companies. Classical economic theory, 
however, proposes the "efficient capital markets hypothesis," which indicates that 
stock prices accurately reflect the value of the underlying investments, incor- 
porating all information available to the public. If the efficient capital markets 
hypothesis is correct, then it should be expected that . 

(A) mutual fund managers, in order to compete with each other, will bid up the 
prices of certain stocks beyond their true values 

(B) mutual fund managers use insider information, an illegal practice, to gener- 
ate higher rates of return than the general stock market 

(C) stock prices will rise over time 

(D) based upon public information alone, companies cannot reliably be labeled 
undervalued or overvalued relative to the general stock market 

(E) some mutual fund managers are better than others at generating a higher 
rate of return on investments 



MANHATTAN 
GMAT 



Warn 



Complete the Argument 



Chapter? 



5. Law of Demand 

Which of the following best completes the passage below? 

The law of demand states that, if all other factors remain equal, the higher the 
price of a good, the less people will consume that good. In other words, the 
higher the price, the lower the quantity demanded. This principle is illustrated 
when , 

(A) Company A has a monopoly over the widget market so an increase in wid- 
get prices has little effect on the quantity demanded 

(B) a manufacturer of luxury cars noticed that its customer base is relatively 
unresponsive to changes in price 

(C) a city experiences an increase in both gasoline prices and the number of 
people taking public transportation 

(D) an increase in the number of computer retailers led to a decrease in the 
average price of computers 

(E) a reduction in the price of oranges from $2 per pound to $1 per pound re- 
sults in 75 pounds of oranges being sold as opposed to 50 pounds 



MANHATTAN 
GMAT 



Complete the Argument 



Chapter 7 



Solutions 



1. Connecting Flight: The correct answer is C. 
Step 1: Identify the question. 

Which of the following most CA A B C D E 

logically completes the argu- 
ment? 



Step 2: Deconstruct the argument. 

John was flying from San 
Francisco to New York with a 
connecting flight in Chicago on 
the same airline. 

Chicago's airport consists of sev- 
eral small stand-alone terminals, 
and it often takes passengers 
thirty to forty minutes to move 
between terminals. 
Johns plane into Chicago ar- 
rived on time. 



The question appears before the 
argument, and the argument con- 
tains a blank at thf end. Both of 
these things indicate that this is a 
Complete the Argument question. 



Straight fact describing his trip. 



Okay, it takes a long time to move 
between terminals in the Chicago 
airport. 



And another fact. . . 



CA A B C D E 
J: SF->C-*NYsame line 

CA A B C D E 

J: SF->C->NYsame line 

C: mult terms, long time to 
move term 

CA A B C D E 

J: SF->C->NYsame line 

C: mult terms, long time to 
move term 

fit on time 

CA A B C D E 

J: SF->C->NYsame line 

C: mult terms, long time to 
move term 

fit on time 

©FA: wont miss next fit 



Step 3: State the Goal. 

The FA claims that J will make his connecting flight in 30 minutes, so she must be assuming it's not going 
to take him more than 30 minutes to get to the gate for his next flight. I need to find an answer choice that 
somehow reflects that — something that makes it more likely that J will get to his next gate in less than 30 
minutes. 



The flight attendant as- 
sured John that he would not 
miss his connecting flight 
thirty minutes later, because 



Here we go: the FA claims that J 
wont miss his next flight. What's 
her evidence for that? That'll be 
the right answer. 



1 -V 




MANHATTAN 
GMAT 



Chapter 7 



Complete the Argument 



Step 4: Work from wrong to right. 



(A) John's airline is 
known for always being 



on time 




(B) another passenger 
on Johns first flight was 
also scheduled to take 
John's connecting flight 



(C) at the Chicago air- 
port, airlines always fly 
in and out of the same 
terminal 



(D) John knew there 
was another flight to 
New York scheduled 
for one hour after the 
connecting flight he was 
scheduled to take 



CA A B C D E 

J: SF-»C->NYsame 
line 

C: mult terms, longtime 
to move term 

fit on time 

©FA: wont miss next fit 
CA AfiCDE 

J: SF^C^NYsame 
line 

C: mult terms, long time 
to move term 

fit on time 

©FA: wont miss next fit 
CA ABCDE 

J: SF->C-»NYsame 
line 

C: mult terms, long time 
to move term 

fit on time 

©FA: won't miss next fit 

CA ABCDE 

J:SF->C-*NYsame 
line 

C: mult terms, long time 
to move term 

fit on time 

©FA: won't miss next fit 



We already know that Js plane to Chicago is 
on time, so that doesnt change anything for 
this first flight. For the connecting flight, the 
only thing that would make the situation bet- 
ter is if that flight were late — because then 
John would have more time to get there. This 
one isnt it. 



Does that mean they can somehow get to the 
next gate faster because there are two of them? 
I dont think so. If they said they were going to 
hold the plane because there were two people 
coming, that would help. . . but they didnt 
say that. 



Ah — / isnt going to have to change termi- 
nals! We dont know how long it takes to 
move around the same terminal, but the 
argument does say that it typically takes 30 
to 40 minutes to change terminals, so it likely 
takes less time when you re staying in the same 
terminal. This increases the likelihood that J 
will make his connecting flight. This might 
be it. 

This is only relevant if J misses his flight. . . 
but the FA claimed that J would make his 
flight. 



MANHATTAN 
GMAT 



Complete the Argument 



(E) the airline gener- 
ally closes the doors 
of a particular flight 
ten minutes before it is 
scheduled to take off 



CA 



A BOB E 



J: SF-»C->NYsame 
line 

C: mult terms, long time 
to move term 

fit on time 

©FA: won't miss next fit 



This hurts J's chances; now, he only has 20 
minutes to make his next flight. Definitely 
not. 



2. Motor City: The correct answer is A. 
Step 1: Identify the question. 



Which of the following best 
completes the passage below ? 



Step 2: Deconstruct the argument. 

A nonprofit organization in Mo- 
tor City has proposed that local 
college students be given the 
option to buy half-price month- 
ly passes for the city's public 
transportation system. 
The nonprofit claims that this 
plan will reduce air pollution 
in Motor City while increas- 
ing profits for the city's public 
transportation system. 
However, this plan is un- 
likely to meet its goals, as 



CA A B C D E The question appears before the 

argument, and the argument con- 
tains a blank at the end. Both of 
these things indicate that this is a 
Complete the Argument question. 



CA A B C D E This is a fact — the organization 

NPMC: give coll stud 1/2 off has proposed this plan. 
pub trans 



CA 



A B C D E 



NP: give coll stud 1/2 off pub 
trans -» I air poll, t prof 



CA 



A B C D E 



Step 3: State the Goal. 



NP: give coll stud 1/2 off pub 
trans 4 air poll, t prof 

©BUT won't work 



Okay, the NP claims something, 
but Tm not labeling this the 
conclusion, because the conclu- 
sion is supposed to be in the final 
sentence ofCA questions. 
This is the conclusion. The author 
thinks the plan wont work. Why? 



The author believes that the nonprofits plan is not going to work, and I need to find a reason why. The plan 
is to let college students buy public transportation passes for half-price in order to reduce air pollution and 
increase profits. 



Chapter? 



Complete the Argument 



Step 4: Work from wrong to rig ht. 



II 



(A) most college stu- 
dents in Motor City 
view public transporta- 
tion as unsafe 



(B) most college stu- 
dents in Motor City 
view public transporta- 
tion as prohibitively 
expensive 

(C) college students 
typically do not have 
the 9-to-5 schedules of 
most workers, and can 
thus be expected to ride 
public transportation 

at times when there are 
plenty of empty seats 

(D) a bus produces more 
air pollution per mile 
than does a car 



CA ABCDE 

NP: give coll stud 1/2 off 
pub trans -> 4 air poll, t 
prof 

©BUT wont work 
CA ABCDE 

NP: give coll stud 1/2 off 
pub trans -» 4 air poll, t 
prof 

©BUT won't work 
CA ABCDE 

NP: give coll stud 1/2 off 
pub trans 4 air poll, t 
prof 

©BUT wont work 



CA 



ABCDE 



(E) a large proportion 
of the college students 
in Motor City live off 
campus 



NP: give coll stud 1/2 off 
pub trans -> 4 air poll, t 
prof 

©BUT won't work 



CA ®B € D E 

NP: give coll stud 1/2 off 
pub trans -» I air poll, t 
prof 

©BUT won't work 



If this is the case, then the students wouldn't 
want to use public transport at all, even if 
they were given a discount. That would make 
the plan unlikely to succeed. This might be it! 



If they don't use public transport specifically 
because it's too expensive, then giving the 
students a discount is likely to make them use 
public transport more. This makes the plan 
more likely to succeed, not less likely. 

If this were true, it'd be good news for the 
public transport's profits — the students 
would be filling what are currently empty 
seats. 



At first, this sounds good — if a bus produces 
more air pollution than a car, then using 
more buses would create more air pollution, 
which would hurt the plan. But the plan 
isn't to use more buses; it's to put more people 
on the already-running buses. Plus, a car 
typically holds only 1 or 2 people; if 10 people 
stop using cars and take 1 bus instead, air 
pollution may indeed be decreased. 
This makes it likely that the students need 
some method of transportation to get to 
school — if they're using cars now and switch 
to buses, then the plan just might work. 



240 



GMAT 



Complete the Argument 



Chapter 7 



3. Deep-brain Stimulation: The correct answer is D. 
Step 1: Identify the question. 



Which of the following most 
logically completes the argu- 
ment given below? 



Step 2: Deconstruct the argument. 

Deep-brain stimulation is a new 
technique for combating severe 
depression. 

In a recent experiment, elec- 
trodes were implanted into the 
brains of six patients who had 
not responded to any currently 
approved treatment for depres- 
sion. 

When an electrical current to 
\ the electrodes was switched on, 
! four of the patients reported 
I feeling a dramatic reduction in 
depressive symptoms. 
The long-term prospects of 
the new treatment are not 
promising, however, because 



CA A B C D E 



CA A B C D E 
DBS combat depr 

CA A B C D E 
DBS combat depr 
Tested on 6 ppl 

CA A B C D E 
DBS combat depr 
Tested on 6 ppl, 4 better 

CA A B C D E 
DBS combat depr 
Tested on 6 ppl, 4 better 
©BUT won't work 



Step 3: State the Goal. 



The question appears before the 
argument, and the argument con- 
tains a blank at the end. Both of 
these things indicate that this is a 
Complete the Argument question. 



Straight fact. 



This tells me how it works and 
that they tested it on 6 people. 



And four of the people got a lot 
better. 



Oh, but the author thinks the 
treatment's not really going to 
work long-term. Why? 



The author describes a new medical treatment but says it's probably not going to be good long-term; I need 
to find a reason why. So far, the only evidence they've given makes DBS sound promising, so I've got to find 
something that shows a flaw or weakness in the treatment. 



GMAT 



241 



Chapter 7 



Complete the Argument 



Step 4: Work from wrong to right. 



(A) other treatments for 
depression may also be 
effective 



(B) the other two 
patients reported only 
a slight reduction of 
depressive symptoms 
during the treatment 



CA A B C D E 

DBS combat depr 

Tested on 6 ppl, 4 better 

©BUT won't work 
CA A B C D E 

DBS combat depr 

Tested on 6 ppl, 4 better 

©BUT won't work 



(C) deep-brain stimula- 
tion relies on the ex- 
pertise of highly skilled 
physicians 

(D) when the electrical 




current is interrupted, 
the effects of the treat- 
ment are reversed 



CA A B £ D E 

DBS combat depr 

Tested on 6 ppl, 4 better 

©BUT won't work 
CA A B € D E 

DBS combat depr 

Tested on 6 ppl, 4 better 

©BUT won't work 



(E) in a subsequent 
experiment, a one- 
hour treatment with 
the electrodes resulted 
in a sustained remis- 
sion from depression in 
the four patients for six 
months 



CA AB۩E 
DBS combat depr 
Tested on 6 ppl, 4 better 
©BUT won't work 



This is probably true in the real world, but 
talking about other treatments doesn't explain 
why DBS won 't be a good treatment long- 
term. 

When I saw the word "only, " I was expecting 
them to say they had a bad result, but actu- 
ally having even a slight reduction is better 
than nothing, especially for people who have 
tried other treatments that haven 't worked. 
So, if anything, this is a plus for DBS. That's 
not what I want. 

I can believe this is true, but we would expect 
any major medical treatment to be performed 
by skilled physicians, so why would this make 
DBS not work long-term? 

That's interesting. So, when the current is on, 
the symptoms go away, but when the current 
is off, the depression comes back. That means 
they'd have to be connected to some machine 
all the time — they couldn't just get a treat- 
ment once a week or once a month. That 
definitely makes the treatment less practical 
and promising. Unless E is better, this might 
be it. 

This is almost the opposite ofD. If you get a 
one-hour treatment, then the symptoms go 
away for 6 months — that's great for DBS! 
This can't be the right answer. 



MANHATTAN 
GMAT 



Complete the Argument 



4. Mutual Funds: The correct answer is D. 



Step 1: Identify the question. 

Which of the following most 
logically completes the argu- 
ment? 



Step 2: Deconstruct the argument. 

Many managers of mutual 
funds proclaim that they have 
been able to generate consistent- 
ly higher rates of return on their 
investments than the general 
stock market by buying shares 
of undervalued companies. 
Classical economic theory, 
however, proposes the "efficient 
capital markets hypothesis," 
which indicates that stock prices 
accurately reflect the value of 
the underlying investments, 
incorporating all information 
available to the public. 
If the efficient capital markets 
hypothesis is correct, then 
it should be expected that 



CA A B C D E 



CA A B C D E 

MF: buy I val corns -» > 
return than SM 



CA A B C D E 

MF: buy 4 val corns > 
return than SM 

BUT CET: stock $ is right 
based on public info 

CA A B C D E 

MF: buy 1 val corns -» > 
return than SM 

BUT CET: stock $ is right 
based on public info 



The question appears before the 
argument, and the argument con- 
tains a blank at the end. Both of 
these things indicate that this is a 
Complete the Argument question. 



These managers claim something, 
but Vm betting the argument 
will disagree. Usually, when the 
author says someone else claims 
something, then the author dis- 
agrees. Lets see. 

Okay, the author is disagreeing. 
This sentence is a little hard to 
understand, but it sounds like its 
saying that there arent "under- 
valued" companies because the 
stock price should generally reflect 
the accurate value of the company. 

Hmm. Vm not sure how to write 
that down. It's just saying that if 
my second line is true, then some- 
thing else should be true, too. 



Step 3: State the Goal. 

This is one of the more rare variations of the complete the argument type. Rather than asking me to strength- 
en something or find an assumption, they're asking me to find something that follows, or must be true based 
on the given info. In other words, this is really an Inference question. 

The CET says that stock prices generally accurately reflect the company's actual value. If that s true, then the 
MF managers must be wrong. 



Chapter 7 



Complete the Argument 



Step 4: Work from wrong to right. 




(A) mutual fund man- 
agers, in order to com- 
pete with each other, 
will bid up the prices of 
certain stocks beyond 
their true values 

(B) mutual fund manag- 
ers use insider informa- 
tion, an illegal practice, 
to generate higher rates 
of return than the gen- 
eral stock market 

(C) stock prices will rise 
over time 



(D) based upon public 
information alone, com- 
panies cannot reliably 
be labeled undervalued 
or overvalued relative to 
the general stock market 

(E) some mutual fund 
managers are better 
than others at generat- 
ing a higher rate of 
return on investments 



CA A B C D E j They're supposed to be buying "undervalued" 

stocks, so bidding up the price doesn't make 
any sense. This doesnt follow from the argu- 
ment. 



MF: buy I val corns > 
return than SM 

BUT CET: stock $ is right 
based on public info 

CA A B C D E 

MF: buy I val corns -> > 
return than SM 

BUT CET: stock $ is right 
based on public info 

CA A B € D E 

MF: buy I val corns -» > 
return than SM 

BUT CET: stock $ is right 
based on public info 
CA A B € D E 

MF: buy I val corns > 
return than SM 

BUT CET: stock $ is right 
based on public info 

CA AB€@E 

MF: buy 4 val corns > 
return than SM 

BUT CET: stock $ is right 
based on public info 



I suppose this could be true of some people, 
but I don't think this is something that abso- 
lutely has to follow from the argument. 



This might be true for many stocks, but there 
are also companies that go down and even go 
out of business. 



This one's a little difficult to understand. If 
we only know public info, then companies 
aren't under- or overvalued. . . so that would 
mean they're correctly valued. . . oh, wait, 
that's similar to what the theory said. It said 
that prices do accurately reflect the value of 
the companies. Okay, this might be it. 
Again, I can believe this is true in general, 
but the argument, doesn't talk about whether 
some MF managers are better than others. 
Rather, it's talking about this CET thing and 
I how it goes against what the MF managers 
say. 



5. Law of Demand: Correct answer is E. 



Step 1: Identify the question. 

Which of the following best 
completes the passage below? 



CA A B C D E The question appears before the 

argument and the argument con- 
tains a blank at the end. Both of 
these things indicate that this is a 
Complete the Argument question. 



MANHATTAN 
GMAT 



Complete the Argument 



Chapter 7 



Step 2: Deconstruct the argument. 



The law of demand states that, 
if all other factors remain equal, 
the higher the price of a good, 
the less people will consume 
that good. 

In other words, the higher the 
price, the lower the quantity 
demanded. 



CA A B C D E 

LD: = factors, t $ -» I 
consume 



CA 



A B C D E 



This principle is illustrated 
when . 



LD: = factors, t $ -» I 
consume 

CA A B C D E 

LD: = factors, T $ -» I 
consume 



They're giving me a "general law, " 
which is essentially a fact. 



This basically says what I already 
wrote y so Fm not going to write 
anything else down. 

This is another one of those weird 
forms. They re not asking me to 
give a reason why or to find a 
conclusion. They re asking me to 
find an example that illustrates 
the "general law" they gave above. 



Step 3: State the Goal. 



This is an interesting one. I need to find an example that illustrates this general principle: when some- 
thing costs more, then people don't want to buy it as much. The opposite would be true, too: when 
something costs less, then people do buy it more. 



Step 4: Work from wrong to right. 



(A) Company A has 
a monopoly over the 
widget market so an 
increase in widget prices 
has little effect on the 
quantity demanded 

(B) a manufacturer of 
luxury cars noticed 
that its customer base is 
relatively unresponsive 
to changes in price 



CA A B C D E 

LD: = factors, t $ -> I 
consume 



CA 



A B C D E 



LD: = factors, t $ -» I 
consume 



This says that an increase in price does not 
change demand, but the LD said that an 
increase in price should lower demand, so this 
cant be an example ofLD. 



Again, this is saying that a change in price 
doesnt really affect demand, which is not 
what the LD theory said. 




MANHATTAN 
GMAT 



245 



Chapter 7 



Complete the Argument 



(C) a city experiences an 
increase in both gasoline 
prices and the number 
of people taking public 
transportation 



CA AfiGDE 

LD: = factors, t $ -» I 
consume 



(D) an increase in the 
number of computer 
retailers led to a decrease 
in the average price of 
computers 



CA AfiGDE 

LD: = factors, f $ -> I 
consume 



consume 



(E) a reduction in the CA A fi € B(g) 

price of oranges from LD: = factors, t $ -> I 

$2 per pound to $1 per 
pound results in 75 
pounds of oranges being 
sold as opposed to 50 
pounds 



Hmm. So maybe an increase in the gaso- 
line price is causing people not to want to 
use as much gas? They didn't actually say 
that directly, though — they just said more 
people are taking public transport. Maybe 
the population is growing It could be the 
case that people are still buying just as much 
gasoline even though the price went up. This 
one doesn't work. 

Increase and decrease — that's good! Oh wait. 
Its not an increase in price vs. a decrease in 
number sold. Its an increase in number of 
stores selling computers to a decrease in price. 
That's not quite the same, but it's still closer 
than the first three answers. IfE isn't better, I 
guess I'll choose this one. 
Tricky! Okay, they're giving me the other side 
of the rule. If it's true that higher price leads 
to lower consumption, then it's also true that 
higher consumption means there were lower 
prices. And that's what this one says — the 
price goes down and people buy more. This 
one's better than D! 



MANHATTAN 
GMAT 



Critical Reasoning 



Wrong Answer Analysis 



On This Chapter. . , 

Out of Scope 
Reverse Logic 
The Mix Up 



Wrong Answer Analysis 



In previous chapters, we have examined a number of question types along with their common traps, or 
wrong answer types. This chapter is a summary of the "wrong answer" information scattered through- 
out the question-types chapters, and it also contains additional examples to illustrate the characteristics 
of these common traps. 

We've talked about many different types, so we're going to group them into three big categories: 

1. Out of Scope 

2. Reverse Logic 

3. The Mix Up 



Out of Scope 

Generally speaking, "out of scope" answers miss or go beyond the scope of the argument in some way. 
The "scope" refers to what the argument covers. If the author claims that women over five feet ten 
inches tall all make good basketball players, then the scope is limited to women (as opposed to men) 
with a certain physical characteristic (over five feet ten inches tall) and to the sport of basketball (as 
opposed to, say, hockey). If an answer focuses on men (the wrong group) and doesn't mention women at 
all, that answer is out of scope. 

There are several different ways in which the test writers will try to take an answer out of the scope of 
the argument. On Assumption Family and Evaluate a Discrepancy questions, it's quite common to find 
answers that talk about the wrong detail (group, activity, action, characteristic, or other detail). For our 
mini-argument in the last paragraph, men would be the wrong group, hockey would be a wrong activ- 
ity, less than five feet ten inches tall would be a wrong characteristic, and so on. The answer may sound 
as though it is related to the argument, but the details will take that answer out of scope. 



249 



Wrong Answer Analysis 



A variation on the last trap is the no tie to the conclusion trap, which we see on Strengthen and Weak- 
en questions, and the no tie to the discrepancy trap, which we see on Discrepancy questions. In all of 
these cases, we are asked to do something to the conclusion or discrepancy, and in all of these cases, the 
trap answer does not affect the conclusion or discrepancy at all. 



Take a look at this example: 



Question 


"No Tie to the Conclusion" Wrong Answer 


Which of the following, if true, best supports 
the claim that women who are under five feet 
ten inches tall cannot have successful careers 
as basketball players? 


Women who are over five feet ten inches tall 
are more likely to excel at basketball. 



In many ways, the wrong answer seems relevant: it's talking about women and basketball; it mentions 
the "five feet ten inch" height threshold. It does not, however, provide any information about women 
who are under five feet ten inches tall. The conclusion claimed something about this specific group of 
women. If the answer on a Strengthen the Conclusion question does not actually address the given 
conclusion, then it is out of scope. The same is true for Weaken questions. 



Discrepancy questions provide us with some sort of discrepancy, and it is also possible for a wrong an- 
swer not to address the discrepancy. This is the same type of trap answer as a "no tie to the conclusion." 
For instance, an argument might tell us: 

Amy loves basketball, yet she's not attending tonight's game even though she has 
tickets. 

It sounds like Amy would normally go to the game. Why isn't she? That's the discrepancy. A "no tie" 
wrong answer might say something like: 

Amy's friends also love basketball and plan to go to the game. 

Why is this wrong answer not tied to our discrepancy at all? The argument mentions only Amy — what 
Amy thinks and what Amy plans to do. The answer choice talks about the wrong group — Amy's 
friends, rather than Amy. Unless the argument told us that Amy was influenced by her friends in some 
way (thereby making her friends part of the argument), we don't care what her friends think or do. 
That's out of the scope of the argument. 

We also often see wrong answer choices that make an irrelevant distinction or comparison; these 
tend to appear primarily on Assumption Family questions. For instance, consider this argument: 

Students who earn A and B grades are more likely to participate in sports than are 
students who earn C grades. Therefore, participation in sports helps students to 
achieve higher grades. 



Wrong Answer Analysis 



Chapter 8 



Lets say that we re asked to find an assumption. An incorrect answer might say something like: 

Students who earn A grades participate in sports even more frequently than do 
those who earn B grades. 

The argument grouped together the A and B students and treated them in the same way. The answer 
separates, or makes a distinction between, the A and B students. On Find the Assumption questions, 
our task is to find something that the author must believe to be true in order to draw his conclusion. 
Is it absolutely necessary to believe that the A-students participate in sports even more frequently than 
the B -students in order to believe the conclusion that sports participation in general helps students to 
achieve higher grades? 

No, it's not absolutely necessary to believe that. If it were true, then that would help to strengthen the 
conclusion — but we weren't asked to strengthen the conclusion! The distinction between A and B 
students is irrelevant, since the argument puts them in the same category. In other words, the argument 
itself makes absolutely no distinction between A and B students, so why would it be necessary to make 
a distinction in order to accept the conclusion? 

This example also illustrates another type of trap answer: the real world distraction. This type of 
answer sounds reasonable to assume in everyday, real-world conversations, but the information does 
not actually fulfill whatever we're supposed to be doing for that question type. For instance, on Find 
the Assumption (FA) questions, as we just discussed, we're trying to find something that must be true. 
It could be true that A students participate even more then B students, and we might even reasonably 
speculate that it is true in the real world, but it doesn't absolutely have to be true in order to draw the 
conclusion. We have to hold ourselves to the "must be true" standard on an FA question, so a "real- 
world" could-be-true answer is incorrect (though often very tempting!). 



In sum, "out of scope" answers can take multiple forms. 



Name 


Why it's tempting 


Why it's wrong 


Most likely found in 


Wrong detail 


May use the same or 
similar words from the 
argument 


It's not the right group, 
activity, action, charac- 
teristic, or other detail. 


Find Assumption, 
Evaluate, Strengthen, 
Weaken, Inference, 
Discrepancy 


No tie to the 
conclusion (or 
discrepancy) 


Likely to use the same or 
similar words from the 
argument 


The question asks us 
to address the conclu- 
sion or discrepancy; 
this answer does not 
affect the conclusion or 
discrepancy. 


Strengthen, Weaken, 
Discrepancy 




MANHATTAN 
GMAT 



Chapter 8 



Wrong Answer Analysis 



Irrelevant 
distinction or 
comparison 


Does use specific groups, 
actions, or other details 
from the argument 


Tries to separate two 
things that the argu- 
ment places into the 
same category. 


Find Assumption, 
Evaluate, Strengthen, 
Weaken 


Real world 
distraction 


Is the kind of thing that 
people might conclude 
or assume in the real 
world; could actually be 
true in the real world 


1 he question asks us 
to find something that 
must be true and this 
answer doesn't have to 
be true. 


Inference 



Reverse Logic 

One of the easier traps to fall into is the "reverse logic" trap, when we accidentally pick the opposite of 
what we really want, such as an answer that strengthens on a Weaken question. Reverse logic traps oc- 
cur most frequently on Assumption Family and Structure Family questions. 

One of the most common ways in which we fall into this trap is to misidentify the conclusion, particu- 
larly when the argument contains two "sides," or points of view. Consider this example. 

Some companies tie bonuses to company performance as well as personal perfor- 
mance, on the theory that individual performance is only valuable as far as it ben- 
efits the company as a whole in some way. This is counter-productive, however, 
because the highest-performing employees are essentially penalized by receiving 
a bonus commensurate only with the average performance of the overall compa- 
ny, thereby leading to a lack of motivation to continue to outperform their peers. 

What are the claims here? Some companies think that "individual performance is only valuable if it 
benefits the company as a whole" and set up their bonus plans accordingly. Some unknown person, on 
the other hand, thinks that this viewpoint is "counter-productive" and will "[lead] to a lack of motiva- 
tion" on the part of the best employees. Which is the main conclusion? 

The authors point of view is always the main conclusion. In this case, the "unknown person" is the 
author. If a claim is attributed to a particular person or group, that claim is likely not the authors claim. 
A claim that is simply asserted, with no commentary as to who is doing the asserting, is likely to be the 
author s claim. 

We can see how easy it would be to mix up the claims, though, and that in turn would make it easy to 
pick a "Reverse Logic" answer, since the two claims are on opposing sides of the fence. 




MANHATTAN 
GMAT 



Wrong Answer Analysis 



Chapter 8 



Lets say that we have this question: 

Some companies tie bonuses to company performance as well as personal per- 
formance, on the theory that individual performance is only valuable as far 
as it benefits the company as a whole in some way. This is counter-productive, 
however, because the highest-performing employees are essentially penalized 
by receiving a bonus commensurate only with the average performance of the 
overall company, thereby leading to a lack of motivation to continue to outper- 
form their peers. 

In the argument given, the portion in boldface plays which of the following roles? 

And here are our two answer choices: 



(A) It is the main conclusion of the argument 

(B) It is a judgment that the argument opposes. 

If we identify the boldface statement as the conclusion (of "some companies"), then we'd pick answer A. 
But if we identify the "lack of motivation" comment as the true conclusion, then the boldface statement 
goes against the conclusion and the answer is clearly B. 

Alternatively, what if we were asked this question for the same argument? 



Question 


Reverse Logic trap answer 


Correct Answer 


Which of the following, if true, 
would most seriously under- 
mine the argument above? 


The performance of employees 
who feel they aren't appropri- 
ately compensated for their 
efforts often drops. 


High-performing employees 
typically state that their pri- 
mary motivation is the satisfac- 
tion of a job well done. 




Strengthens 


Weakens 



In this example, the Reverse Logic trap strengthens the conclusion instead of weakening it (and it is 
even easier to fall into this trap if you misidentify the conclusion!). The trap answer, above, reinforces 
(or strengthens) the authors conclusion: people whose pay is below their performance may lose motiva- 
tion to work hard. The correct answer, on the other hand, does weaken the author s conclusion by offer- 
ing a reason why employees might continue to work hard regardless of compensation levels. 

In general, make sure to check the logical "direction" of the answers; if something fits one of the follow- 
ing categories, its a trap! 




MANHATTAN 
GMAT 



Chapter 8 



Wrong Answer Analysis 



Question Type 


Reverse Trap answer will 


Role 


assign the opposite role of the correct 
role 


Find Assumption 


actually hurt the argument if it is true 


Strengthen 


weaken the conclusion 


Weaken 


strengthen the conclusion 



The Mix Up 

Our final major wrong answer category is "The Mix Up." These can appear in a few different varieties. 

The one word off variety is simple, in the sense that only a single word can make the answer wrong, 
but also quite difficult and tempting. . . because only a single word makes the difference! (Note: it could 
also be two or three words.) These wrong answers most often show up in Describe the Role, Describe 
the Argument, and Inference questions. 

For example, what's the difference between the below two answer choices? 

The first is a prediction that supports a position that the argument concludes. 

The first is a prediction that supports a position that the argument opposes. 

Only one word is different — the very last word — and yet that one word changes everything. The first 
sample answer is describing a premise: something that supports the author's conclusion. The second, 
on the other hand, is describing a counter-premise: something that goes against the author's conclusion. 
If were reading too quickly or skim over a word, that can be the difference between picking the right 
answer and falling for a tempting trap. 

We also have to be on the alert for the switching terms trap, which occurs most often on Inference and 
Find the Assumption questions. The answer choice will use actual wording or terminology from the 
argument, but it will switch terms around or pair things that weren't actually paired in the argument. 
For instance, what if we were asked to infer something about the following argument excerpt? 



Argument excerpt 


Switching Terms Trap Answer 


Studies have shown that holding a blood drive 
tends to stimulate the participation of members 
of an organization and increase the number of 
donations. 


(B) Holding a blood drive helps an organization 
to increase the number of members. 




MANHATTAN 
GMAT 



Wrong Answer Analysis 



Chapter 8 



See what answer choice B did there? Its certainly possible that answer choice B is true, but it doesn't 
accurately reflect what the argument actually said. Answer B contains many of the same words found in 
the argument, but in a mixed-up way. The argument said that the number of donations would increase, 
not the number of members. On an inference question, that is sufficient to eliminate this answer, be- 
cause on inference questions, we are looking for an answer that must be true. 





255 



GMAT 



Wrong Answer Analysis Chapter 8 

Problem Set 

The problem set consists of problems that you have already seen in earlier chapters of this book. Note: 
if you have not yet done these problems, then do them normally under the 2-minute time constraint for 
the first time before doing the exercise described below. 

For each of the following problems, identify the right answer, and try to articulate why each wrong 
answer is wrong. If you spot a particular category of wrong answer, write that down as well, but remem- 
ber that the real test won't ask us to classify. Rather, our goal is to train ourselves to be able to identify 
wrong answers accurately and efficiently; the wrong answer categories are just a tool to help us practice 
this. Also note that some wrong answers may not fit into any of the common categories listed in this 
chapter. 

1. Gray Wolf Population 

From Chapter 3, Structure Family 

Government representative: Between 1996 and 2005, the gray wolf popula- 
tion in Minnesota grew nearly 50 percent; the gray wolf population in Montana 
increased by only 13 percent during the same period. Clearly, the Minnesota gray 
wolf population is more likely to survive and thrive long term. 

Environmentalist: But the gray wolf population in Montana is nearly 8 times the 
population in Minnesota; above a certain critical breeding number, the popula- 
tion is stable and does not require growth in order to survive. 

The environmentalist challenges the government representative's argument by 
doing which of the following? 

introducing additional evidence that undermines an assumption made by 
the representative 

challenging the representative's definition of a critical breeding number 
demonstrating that the critical breeding number of the two wolf popula- 
tions differs significantly 

implying that the two populations of wolves could be combined in order to 
preserve the species 

suggesting that the Montana wolf population grew at a faster rate than 
stated in the representative's argument 




MANHATTAN 
GMAT 



Chapter 8 



Wrong Answer Analysis 



2. Malaria 

From Chapter 3, Structure Family 

In an attempt to explain the cause of malaria, a deadly infectious disease, early 
European settlers in Hong Kong attributed the malady to poisonous gases sup- 
posedly emanating from low-lying swampland. In the 1880s, however, doctors 
determined that Anopheles mosquitoes were responsible for transmitting the 
disease to humans after observing that the female of the species can carry a 
parasitic protozoan that is passed on to unsuspecting humans when a mos- 
quito feasts on a person's blood. 

What function does the statement in boldface fulfill with respect to the argument 
presented above? 

(A) It provides support for the explanation of a particular phenomenon. 

(B) It presents evidence that contradicts an established fact. 

(C) It offers confirmation of a contested assumption. 

(D) It identifies the cause of an erroneous conclusion. 

(E) It proposes a new conclusion in place of an earlier conjecture. 

3. Oil and Ethanol 

From Chapter 4, Assumption Family 

Country N's oil production is not sufficient to meet its domestic demand. In order 
to sharply reduce its dependence on foreign sources of oil, Country N recently 
embarked on a program requiring all of its automobiles to run on ethanol in ad- 
dition to gasoline. Combined with its oil production, Country N produces enough 
ethanol from agricultural by-products to meet its current demand for energy. 

Which of the following must be assumed in order to conclude that Country N will 
succeed in its plan to reduce its dependence on foreign oil? 

(A) Electric power is not a superior alternative to ethanol in supplementing 
automobile gasoline consumption. 

(B) In Country N, domestic production of ethanol is increasing more quickly 
than domestic oil production. 

(C) Ethanol is suitable for the heating of homes and other applications aside 
from automobiles. 

(D) In Country N, gasoline consumption is not increasing at a substantially 
higher rate than domestic oil and ethanol production. 

(E) Ethanol is as efficient as gasoline in terms of mileage per gallon when used 
as fuel for automobiles. 




MANHATTAN 
GMAT 



Wrong Answer Analysis 



Chapter 8 



4. Charity 

From Chapter 4, Assumption Family 

Studies show that impoverished families give away a larger percentage of their 
income in charitable donations than do wealthy families. As a result, fundraising 
consultants recommend that charities direct their marketing efforts toward indi- 
viduals and families from lower socioeconomic classes in order to maximize the 
dollar value of incoming donations. 

Which of the following best explains why the consultants' reasoning is flawed? 

(A) Marketing efforts are only one way to solicit charitable donations. 

(B) Not all impoverished families donate to charity. 

(C) Some charitable marketing efforts are so expensive that the resulting dona- 
tions fail to cover the costs of the marketing campaign. 

(D) Percentage of income is not necessarily indicative of absolute dollar value. 

(E) People are more likely to donate to the same causes to which their friends 
donate. 

5. Food Allergies 

From Chapter 4, Assumption Family 

Food allergies account for more than thirty thousand emergency department 
visits each year. Often, victims of these episodes are completely unaware of their 
allergies until they experience a major reaction. Studies show that 90 percent of 
food allergy reactions are caused by only eight distinct foods. For this reason, in- 
dividuals should sample a minuscule portion of each of these foods to determine 
whether a particular food allergy is present. 

Which of the following must be studied in order to evaluate the recommendation 
made in the argument? 

(A) The percentage of allergy victims who were not aware of the allergy before a 
major episode 

(B) The percentage of the population that is at risk for allergic reactions 

(C) Whether some of the eight foods are common ingredients used in cooking 

(D) Whether an allergy to one type of food makes someone more likely to be 
allergic to other types of food 

(E) Whether ingesting a very small amount of an allergen is sufficient to pro- 
voke an allergic reaction in a susceptible individual 




MANHATTAN 
GMAT 



Chapter 8 



Wrong Answer Analysis 



6. Smithtown Theatre 

From Chapter 5, Assumption Family 

The Smithtown Theatre, which stages old plays, has announced an expansion that 
will double its capacity along with its operating costs. The theatre is only slightly 
profitable at present. In addition, all of the current customers live in Smithtown, 
and the population of the town is not expected to increase in the next several 
years. Thus, the expansion of the Smithtown Theatre will prove unprofitable. 

Which of the following, if true, would most seriously weaken the argument? 

(A) A large movie chain plans to open a new multiplex location in Smithtown 
later this year. 

(B) Concession sales in the Smithtown Theatre comprise a substantial propor- 
tion of the theatre's revenues. 

(C) Many recent arrivals to Smithtown are students that are less likely to attend 
the Smithtown Theatre than are older residents. 

(D) The expansion would allow the Smithtown Theatre to stage larger, more 
popular shows that will attract customers from neighboring towns. 

(E) The Board of the Smithtown Theatre often solicits input from residents of 
the town when choosing which shows to stage. 

7. Digital Coupons 

From Chapter 5, Assumption Family 

The redemption rate for e-mailed coupons is far lower than that for traditionally 
distributed paper coupons. One factor is the "digital divide" — those who might 
benefit the most from using coupons, such as homemakers, the elderly, and those 
in low-income households, often do not have the knowledge or equipment nec- 
essary to go online and receive coupons. 

Which of the following, if true, does the most to support the claim that the digital 
divide is responsible for lower electronic coupon redemption rates? 

(A) Computers are available for free in libraries, schools, and community centers. 

(B) The redemption rate of ordinary coupons is particularly high among elderly 
and low income people that do not know how to use computers. 

(C) Many homes, including those of elderly and low income people, do not have 
high-speed internet connections. 

(D) More homemakers than elderly people would use computers if they had ac- 
cess to them. 

(E) The redemption rate for coupons found on the internet has risen in the last 
five years. 




MANHATTAN 
GMAT 



Wrong Answer Analysis 



Chapter 8 



8. World Bank 

From Chapter 5, Evidence Family 

In 2010, China comprised about 10 percent of the world's gross domestic product 
(GDP), and its voting share in the World Bank was increased from under 3 percent 
to 4.4 percent. During the same timeframe, France comprised about 4 percent of 
the world's GDP and saw its voting share in the World bank drop from 4.3 percent 
to 3.8 percent. 

Which of the following can be logically concluded from the passage above? 

(A) World Bank voting shares are allocated based upon each country's share of 
the world's GDP. 

(B) The new ratio of voting share to percentage of world GDP is lower for China 
than it is for France. 

(C) Gross domestic product is the most important factor in determining voting 
share at the World Bank. 

(D) China should be upset that its voting share does not match its proportion of 
the world's GDP. 

(E) France lost some of its voting share to China because China comprised a 
larger portion of the world's GDP. 

9. Bar Codes 

Two-dimensional bar codes are omni-directional; that is, unlike one-dimensional 
bar codes, they can be scanned from any direction. Additionally, two-dimensional 
bar codes are smaller and can store more data than their one-dimensional coun- 
terparts. Despite such advantages, two-dimensional bar codes account for a 
much smaller portion of total bar code usage than one-dimensional bar codes. 

Which of the following, if true, most helps to resolve the apparent paradox? 

(A) Many smaller stores do not use bar codes at all because of the expense. 

(B) For some products, the amount of data necessary to be coded is small 
enough to fit fully on a one-dimensional bar code. 

(C) Two-dimensional bar codes are, on average, less expensive than one-dimen- 
sional bar codes. 

(D) Two-dimensional bar codes can also be scanned by consumer devices, such 
as cell phones. 

(E) One-dimensional bar codes last longer and are less prone to error than two- 
dimensional bar codes. 




MANHATTAN 
GMAT 



Wrong Answer Analysis 
Solutions 

1. Gray Wolf Population 

(A) introducing additional evidence that undermines an assumption made by the representative 

This is the correct answer. 

(B) challenging the representatives definition of a critical breeding number 

This is a Mix-Up answer. The environmentalist discusses critical breeding number, not the 
representative. 

(C) demonstrating that the critical breeding number of the two wolf populations differs significantly 

This doesntfit into one of the standard trap categories. The environmentalist does mention 
the term "critical breeding number' but does not say that this number differs significantly. 
Rather, the environmentalist says that the population size differs. 

(D) implying that the two populations of wolves could be combined in order to preserve the species 

This is a Real World Distraction answer. It might be an interesting strategy in the real 
world, but the argument doesn't mention it. 

(E) suggesting that the Montana wolf population grew at a faster rate than stated in the representative's 
argument 

This is a Mix-Up answer. The environmentalist does mention a number, but that number 
does not represent a rate of growth. 

2. Malaria 

(A) It provides support for the explanation of a particular phenomenon. 

This is the correct answer. 

(B) It presents evidence which contradicts an established fact. 

This doesnt fit into one of the standard trap categories. The boldface text does contradict 
what people once thought about malaria, but what they once thought was not an established 
fact. 

(C) It offers confirmation of a contested assumption. 

This is a "one word off" trap — nothing was contested in the argument. 



Chapter 8 



Wrong Answer Analysis 



(D) It identifies the cause of an erroneous conclusion. 

This could be a Reverse Logic trap; were looking for something that supports the conclusion. 

(E) It proposes a new conclusion in place of an earlier conjecture. 

This is a general Mix-Up answer; the argument does do this in general, but not the state- 
ment in boldface. 

3. Oil and Ethanol 

(A) Electric power is not a superior alternative to ethanol in supplementing automobile gasoline con- 
sumption. 

This Out Of Scope answer is the Wrong Detail. The argument is about oil and ethanol, not 
electric power. 

(B) In Country N, domestic production of ethanol is increasing more quickly than domestic oil produc- 
tion. 

This doesn't fit into one of the standard trap categories. It sounds pretty good at first glance, 
but isn't actually necessary (which is a requirement for a correct answer on an assumption 
question). 

(C) Ethanol is suitable for the heating of homes and other applications aside from automobiles. 

This seems somewhat Out Of Scope. What does the heating of homes have to do with the 
argument? 

(D) In Country N, gasoline consumption is not increasing at a substantially higher rate than domestic 
oil and ethanol production. 

This is the correct answer. 

(E) Ethanol is as efficient as gasoline in terms of mileage per gallon when used as fuel for automobiles. 

This Out of Scope answer is too specific on the detail, so we can call this a Wrong Detail. 
Knowing how efficient the two are generally might help, but they don't necessarily have to 
be equally efficient. 

4. Charity 

(A) Marketing efforts are only one way to solicit charitable donations. 




MANHATTAN 
GMAT 



Wrong Answer Analysis 



Chapter 8 



This Out Of Scope answer discusses an Irrelevant Distinction. It may be true that there are 
other ways to solicit donations besides marketing efforts, but the argument itself is about 
marketing efforts. 

(B) Not all impoverished families donate to charity. 

This answer is One Word Off. It makes a statement about "air impoverished families, but 
the argument never says that all of these families act in the same way. (Note: many people 
will eliminate this answer because the word "air is extreme. Its true that this argument 
does not provide support for the extreme word "all," but extreme words can appear in cor- 
rect CR answers — if the argument provides support for the extreme word.) 

(C) Some charitable marketing efforts are so expensive that the resulting donations fail to cover the 
costs of the marketing campaign. 

This is an especially tricky Wrong Detail answer. The argument never talks about whether 
the marketing campaign will be "profitable" (that is, make more money than was spent on 
the marketing campaign). It might seem like this should be the goal of any charitable mar- 
keting campaign. . . but the argument doesn't address this. 

(D) Percentage of income is not necessarily indicative of absolute dollar value. 

This is the correct answer. 

(E) People are more likely to donate to the same causes to which their friends donate. 

This sounds plausible in the Real World, but its just a distraction here — the argument 
doesnt address this issue. 

5. Food Allergies 

(A) The percentage of allergy victims who were not aware of the allergy before a major episode 

This answer makes an Irrelevant Distinction. Knowing the exact percentage doesnt actually 
tell us anything. 

(B) The percentage of the population that is at risk for allergic reactions 

This answer is Out Of Scope because it talks about all allergies in general, not just food 




allergies. 



(C) Whether some of the eight foods are common ingredients used in cooking 



This doesnt fit into one of the standard trap categories. The argument does not hinge on 
how commonly used the foods must be in order to warrant testing. Further, the argument 
does not limit itself to foods that must be cooked. 



MAN 



H 




265 



GMAT 



Chapter 8 



Wrong Answer Analysis 



(D) Whether an allergy to one type of food makes someone more likely to be allergic to other types of 
food 

This answer makes an Irrelevant Distinction; the argument doesn't address whether some- 
one is allergic to multiple types of food. 

(E) Whether ingesting a very small amount of an allergen is sufficient to provoke an allergic reaction in 
a susceptible individual 

This is the correct answer. 

6. Smithtown Theatre 

(A) A large movie chain plans to open a new multiplex location in Smithtown later this year. 

This one can be considered either Out of Scope (a different movie chain doesnt matter to 
this conclusion) or Reverse Logic (if anything, the new movie theatre might take some busi- 
ness from Smithtown Theatre, strengthening the authors claim). 

(B) Concession sales in the Smithtown Theatre comprise a substantial proportion of the theatres rev- 
enues. 

This one is Out of Scope because it has No Tie to the Conclusion. Knowing this information 
about concession sales tells us nothing new about the Theatres plans to expand. 

(C) Many recent arrivals to Smithtown are students that are less likely to attend the Smithtown Theatre 
than are older residents. 

This is a Reverse Logic trap because it strengthens the authors claim (and this is a weaken 
question). 

(D) The expansion would allow the Smithtown Theatre to stage larger, more popular shows that will 
attract customers from neighboring towns. 

This is the correct answer. 

(E) The Board of the Smithtown Theatre often solicits input from residents of the town when choosing 
which shows to stage. 

This sounds good in the Real World, but it really has No Tie to the Conclusion. Two traps 
for the price of one! 

7. Digital Coupons 

(A) Computers are available for free in libraries, schools, and community centers. 




MANHATTAN 
GMAT 



Wrong Answer Analysis 

If anything, this answer choice weakens the authors claim, and this is a strengthen ques- 
tion. This is a Reverse Logic trap. 

(B) The redemption rate of ordinary coupons is particularly high among elderly and low income people 
that do not know how to use computers. 

This is the correct answer. 

(C) Many homes, including those of elderly and low income people, do not have high-speed internet 
connections. 

This argument focuses on the Wrong Detail. The argument says nothing about having to 
have high-speed internet connections. 

(D) More homemakers than elderly people would use computers if they had access to them. 

This answer is making an Irrelevant Distinction between two groups that are treated the 
same in the argument. 

(E) The redemption rate for coupons found on the internet has risen in the last five years. 

This answer focuses on the Wrong Detail. The argument claims that paper coupons are in 
wider use because some people have difficulty accessing electronic coupons. 

8. World Bank 

(A) World Bank voting shares are allocated based upon each country's share of the worlds GDP. 

This sounds as though it could be reasonable in the Real World, but they didnt provide 
enough data points to say that this is definitely true. 

(B) The new ratio of voting share to percentage of world GDP is lower for China than it is for France. 

This is the correct answer. 

(C) Gross domestic product is the most important factor in determining voting share at the World 
Bank. 

We can think of this as an Irrelevant Comparison because it says that something is the "most 
important factor 7 when the argument doesnt actually say that at all. 

(D) China should be upset that its voting share does not match its proportion of the worlds GDP. 

This might be reasonable to believe in the Real World, but the argument mentions nothing 
about how China "should" feel about anything. 



Chapter 8 



Wrong Answer Analysis 



(E) France lost some of its voting share to China because China comprised a larger portion of the 
world s GDP. 

We can consider this a Mix-Up answer because it includes many words and terms from the 
argument. . . but this answer imposes a cause-effect relationship that wasnt given in the 
argument. 

9. Bar Codes 

(A) Many smaller stores do not use bar codes at all because of the expense. 

This choice makes an Irrelevant Distinction. The argument talks about stores that do use 
bar codes, not stores that dont. 

(B) For some products, the amount of data necessary to be coded is small enough to fit fully on a one- 
dimensional bar code. 

This one is very tempting, but its also a One Word Off trap. The choice addresses only 
"some" products — not enough to affect the conclusion. 




(C) Two-dimensional bar codes are, on average, less expensive than one-dimensional bar codes. 

This is a Reverse Logic trap. If this choice were true, it would make the discrepancy even 
more strange, because it offers another reason why people would want to use 2D bar codes. 

(D) Two-dimensional bar codes can also be scanned by consumer devices, such as cell phones. 

This can be considered a Reverse Logic trap (because it makes 2D bar codes more attractive) 
or a No Tie to the conclusion trap (because scanning with consumer devices isnt part of the 
scope of the argument). 

(E) One-dimensional bar codes last longer and are less prone to error than two-dimensional bar codes. 

This is the correct answer. 



268 



GMAT 



Append ix/A 

Critical Reasoning 



Official Guide Problem Sets 



On This Charter. . , 



Official Guide Problem Sets 



Official Guide Problem Sets 
Official Guide Problem Sets 



Appendix A 



Now that you have completed Critical Reasoning, it is time to test your skills on problems that have 
actually appeared on real GMAT exams over the past several years. 

The problem set that follows is composed of questions from two books published by the Graduate 
Management Admission Council® (the organization that develops the official GMAT exam): 

The Official Guide for GMAT Review, 13th Edition (pages 33-39 & 500-538) 
The Official Guide for GMAT Verbal Review, 2nd Edition (pages 116-152). 

These books contain Verbal questions that have appeared on past official GMAT exams. (The ques- 
tions contained therein are the property of The Graduate Management Admission Council, which is 
not affiliated in any way with Manhattan GMAT.) 

Although the questions in The Official Guides have been "retired" (they will not appear on future of- 
ficial GMAT exams), they are great practice questions. 

Solve each of the following problems in a notebook, making sure to demonstrate how you arrived at 
each answer by showing all of your work. If you get stuck on a problem, look back at the Critical Rea- 
soning strategies and content contained in this guide to assist you. 

Note: Problem numbers preceded by "D" refer to questions in the Diagnostic Test chapter of 
The Official Guide for GMAT Review, 13th Edition (pages 33-39). 

Describe the Argument: 

13th Edition: 34, 84, 85, 123 
Verbal Review: 79 

Describe the Role: 

13th Edition: 18, 28, 63, 76, 78, 89, 98, 116 
Verbal Review: 48, 74 

Find the Assumption: 

13th Edition: 21, 41, 46, 48, 75, 77, 83, 93, 96, 106, 109, 113, D28 
Verbal Review: 7, 34, 44, 52, 56, 63, 67, 76 

Evaluate the Argument: 

13th Edition: 7, 10, 15, 27, 36, 42, 47, 53, 68, 70, 72, 110, 114, 124, D21, D22, D29 
Verbal Review: 3, 28, 40, 42, 54, 66, 70 

MANHATTAN 
GMAT 



Appendix A 



Official Guide Problem Sets 



Flaw: 



13th Edition: 8, 100 

Strengthen the Argument: 

13th Edition: 1, 5, 11, 14, 16, 19, 23, 29, 30, 31, 35, 40, 45, 50, 52, 56, 64, 67, 95, 101, 

102, 108, 111, 118, 120, 121, D25, D27, D32 
Verbal Review: 1, 2, 6, 9, 13, 17, 21, 23, 25, 29, 30, 32, 33, 35, 37, 45, 51, 55, 58, 62, 
65, 68, 69, 77, 78, 82 

Weaken the Argument: 

13th Edition: 2, 4, 20, 25, 32, 37, 43, 51, 58, 62, 71, 73, 79, 80, 82, 87, 88, 90, 97, 107, 

112, 115, 117, 119, 122, D18, D20, D23, D26, D30, D34 
Verbal Review: 4, 5, 11, 15, 16, 18, 20, 22, 24, 26, 27, 31, 36, 39, 41, 46, 47, 49, 50, 71, 
80, 81, 83 



Explain the Discrepancy: 

13th Edition: 3, 6, 9, 13, 17, 22, 24, 44, 49, 57, 61, 86, 92, 94, 99, D19, D33 
Verbal Review: 8, 59, 60, 61, 72, 73 

Inference: 



13th Edition: 26, 38, 54, 55, 60, 66, 91, 103, 104, 105, D24, D31 
Verbal Review: 12, 14, 19, 43, 53, 57, 64, 75 

Complete the Argument: 

13th Edition: 12, 33, 39, 59, 65, 69, 74, 81 
Verbal Review: 10, 38 



MANHATTAN 
GMAT 



ALL TEST PREP IS NOT THE SAME 




MANHATTAN MANHATTAN MANHATTAN 

GMAT GRE" LSAT 

Elite test preparation from 99th percentile instructors. 

Find out how we're different. 



M 



^9 Hr 



www.manhattanprep.com 



Did you know you CANNOT use 
any paper on the actual GMAT? 

When taking the GMAT, you can only use a laminated booklet 
with a felt-tip pen to take notes and work out problems. 

Don't be caught off-guard on test day! 



MANHATTAN 

GMAT 



I 



TEST SIMULATION BOOKLET 



Identical to laminated scratch pad 
and marker provided at the official GMAT 



Visit out website for free resources & expert advice. Including: 
& Strategist for using this ten simulation booklet 
ST Fiee computer-adaptive practice mm 
ST fiee GMAT flash caids Hill] I 

& Top ten tips for lei! day 




Practice with a Test Simulation Booklet 



Offered Exclusively By 



MANHATTAN GMAT 



A GMAT Prep 
Essential! 



Only $21.00 USD, and it 
includes the felt-tip pen 




Now Available 

Get one today at 
www.manhattangmat.com 



FREE with any Complete 
Prep Set purchase or any 
ManhattanGMAT Course 




' GMAT and GMAC are registered trademarks of the Graduate Management Admission Council which neither sponsors nor endorses this product. 



/ \ 

Mission 



EVERY CANDIDATE HAS A STORY TO TELL. 

We have the creative experience to help you tell yours. 



We are mbaMission p a professional MBA admissions consulting firm, specializing 
in helping business school applicants identify and showcase the strongest aspects 
of their candidacy in their applications. Our dedicated senior consultants— all 
published authors with elite MBA experience— will work one-on-one with you to 
help you discover, select and articulate your unique stories and force the 
admissions committees to take notice. 




Every Manhattan GMAT student receives 

• Free 30-minute consultation with an mbaMission senior consultant - Sign up 
at www.mbamission.com/consult.php 

• Free copy of our 250-page book. The Complete Start-to-Finish MBA Admissions 
Guide, loaded with application advice as well as sample essays, 
recommendations, resumes and more 

• One free Insider s Guide on one of 16 top business schools (available in the 
Manhattan GMAT Student Center) 



mbaMission Services 

• Complete Start-to-Finish Package offers unlimited service for a flat fee 
and guides you through the entire MBA application process, from 
brainstorming and outlining to interviews and beyond 

• A la Carte Hourly Services focus on specific application needs, such as 
perfecting a single essay, reviewing your resume or analyzing a 
recommendation 

• Mock Interview Sessions simulate a real MBA interview with feedback 

• MBA Application Boot Camp demonstrates how to create a standout 
application in a live, classroom "workshop" environment 



www.mbamission.com/manhattangmat I info@mbamission.com I (646) 485-8846 



STRATEGY GUIDE SERIES 0 1 1 \z\ IcbI kl |5 



Critical Reasoning 




8 



The Critical Reasoning Strategy Guide strengthens your logic skills for this tough GMAT question type. Learn how to 
simplify arguments, classify questions, and eliminate wrong answers efficiently and confidently through clear explanations 
and step-by-step instructions. Practice the logic skills tested by the GMAT and master proven methods for solving all 
Critical Reasoning problems. 

Used by itself or with other Manhattan GMAT Strategy Guides, Critical Reasoning will help you develop all the knowledge, 
skills, and strategic thinking necessary for success on the GMAT! 



Sample Problem 



9. CEO: Over the past several years, we have more 
than doubled our revenues but profits have steadily 
declined because an increasing number of customers 
have failed to pay their balances. In order to compen- 
sate for these higher default rates, we will increase the 
interest charged on outstanding balances from an 
annual percentage rate (APR) of 9.5% to an APR of 
1 2%. This increase will be sufficient to compensate for 
the current rate of defaults and allow us to increase 
our profits. 

Which of the following statements, if true, would most 
seriously undermine a plan to increase interest rates in 
order to spur profitable growth? 



(A) Many other companies have experienced a similar 
trend in their default rates. 

(B) The company's operating expenses are above the 
industry average and can be substantially reduced, thus 
increasing margins. 

(C) The increase in default rates was due to a rise in 
unemployment, but unemployment rates are expected 
to drop in the coming months. 

(D) The proposed increase in the APR will, alone, more 
than double the company's profit margins. 

(E) An increase in the APR charged on credit card 
balances often results in higher rates of default. 



The answer is on page 182. 



MANHATTAN GMAT 

Manhattan GMAT has perfected 
the art of studying for the GMAT." 



• STUDENT AT TOP FIVE B-SCHOOL 



How This Book is Different 

More pages per topic than all-in-one tomes 
Real content written by real GMAT instructors 
Advanced online tools to help you study smarter 



US $26.00 CAN $29.99 



ifl\ SUSTAINABLE 
\5C FORESTRY 
yr INITIATIVE 
Certified Chain of Custody 

Promoting Sustainable Forestry 
www.sfiprGgram.Grg 

^ 5F1-00756 J 



How Manhattan GMAT is Different 

99th percentile instructors with unparalleled GMAT expertise 
Focus on content-based curriculum, not empty tips & tricks 
Commitment to excellence in the classroom and beyond 



As part of Manhattan Prep, Manhattan GMAT provides intensive courses, private tutoring, and 
free seminars in addition to comprehensive curricular materials. 



www.manhattangmat.com US & Canada: 1.800.576.4628 International: 001.212.721.7400